22-23 Dahiliye Teorik

Download as pdf or txt
Download as pdf or txt
You are on page 1of 137

22-23 Internal Medicine Theoric Questions

E1
1- Which of the following is correct for renal osteodystrophy in chronic kidney disease?
A) Increased phosphate excretion, decreased parathyroid hormone levels, reduced 1,25 OH
vitamin D, increased FGF-23
B) Reduced phosphate excretion, decreased parathyroid hormone levels, increased 1,25 OH
vitamin D, decreased FGF-23
OC) Reduced phosphate excretion, increased parathyroid hormone levels, reduced 1,25 OH
vitamin D, increased FGF-23
D) Reduced phosphate excretion, decreased parathyroid hormone levels, reduced 1,25 OH
vitamin D, decreased FGF-23
E) Increased phosphate excretion, increased parathyroid hormone levels, reduced 1,25 OH
vitamin D, increased FGF-23

2- Which of the following is false for the progression of chronic kidney disease (CKD)?
A) The rate of progression of CKD is influenced by age.
B) Female gender has a slower rate of progression compared with male
OC) Nephrotic range proteinuria is associated with decreasing rate of CKD progression
D) Tight control of glucose in diabetes mellitus can potentially slow the rate of progression of
CKD.
E) Hyperuricemia may lead to progression of CKD.

3- Which one of the following is not a risk factor for contrast related nephropathy ?
OA) diabetes insipidus
B) A history of chronic kidney disease
C) dehydration
D) High volume radiocontrast agent
E) Intra-arterial application of contrast agent

4- Which one of the following is wrong for acute tubular necrosis (ATN)?
A) ATN is the most common cause of acute kidney failure in hospitalized patients
B) ATN occurs due to sepsis, prolonged prerenal azotemia and toxic agents
C) dysmorphic erythrocytes and erythrocyte casts are classical signs of ATN
D) Proteinuria usually below the nephrotic range in ATN 1

OE) Fractional sodium excretion is usually < 1.

5- Which one of the following causes AL amyloidosis ?


A) FMF (familial mediterranean fever)
B) Rheumatoid arthritis
C) Ulcerative colitis
OD) Multiple Myeloma
E) tuberculosis
6- Which one of the following is an unexpected manifestation for SLE nephritis?
OA) Hypotension
B) hematuria
C) proteinuria
D) hyperkalemia
E) Cellular casts

7- 62 years old man who is ongoing hemodialysis 3 times a week for 10 years, was admitted
to the emergency department. Analysis of the arterial blood gave the following values:
PCO2 16 mm Hg, HCO3-: 4 mmol /l and pH 7.08. What is the underlying acid-base disorder?
A) Uncompensated metabolic acidosis
OB) Compensated metabolic acidosis
C) Uncompensated metabolic alkalosis
D) Compensated respiratory acidosis
E) Compensated metabolic alkalosis

8- Which one of the following is not a treatment option for autosomal dominant polycystic
kidney disease?
OA) Vasopressin agonists
B) V2 receptor antagonists
C) sirolimus
D) everolimus
E) Somatostatin analogues

9- Which of the following is not a component of ISTH scoring system for DIC?
A) Prothrombin time
B) Fibrin degradation tests
C) Platelet count
OD) hemoglobin level
E) fibrinogen level

10- Which one is not an early complication of allogeneic stem cell transplantation?
A) Hemorrhagic cystitis
B) Acute graft versus host disease
C) Sinusoidal obstruction syndrome
OD) cataract
E) pneumonia

11- Which of the following is false regarding stem cell transplantation?


A) Mobilization is the process that provides the passage of stem cells from the bone marrow
to the peripheral blood.
B) The advantages of using peripheral stem cells include faster engraftment.
OC) Peripheral stem cell collection is performed under anesthesia
D) For autologous stem cell transplantation >2x10*6 CD34+ cells/kg of power is sufficient.
E) The most important criterion for donor selection is HLA compatibility.
12- Which of the following is true for polycythemia vera?
A) Erythropoietin level is increased.
B) BCR-ABL is positive.
C) Cytoreductive therapy is indicated for all patients.
OD) Erythrocyte sedimentation rate is decreasing.
E) Arterial oxygen saturation is low.

13- Which of the following is correct about Non- Hodgkin Lymphomas (NHL)?
A) The most common NHL seen in our country is follicular lymphomas.
B) Autoimmune diseases do not increase the risk of NHL.
C) Burkitt lymphoma is among the indolent lymphomas.
OD) Anaplastic Large cell lymphoma is among the T cell neoplasms.
E) t(14;18) often accompanies Mantle lymphoma.

14- Which of the following is wrong regarding Hodgkin's Lymphoma?


A) HL usually has supradiaphragmatic onset.
B) It is thought that EBV has a role in its etiology
OC) CD30 positivity is observed in all subtypes.
D) Nodular sclerosing HL is more common in women
E) Reed-Stenberg cells guide the diagnosis in histopathological samples

15- Which of the following is true for Antiphospholipid syndrome (APS)?


A) aPTT is prolonged and corrected with mixing test
B) Patients have only venous thrombosis risk
C) It is an inherited thrombophilia risk
OD) APS may be primary or secondary
E) Thrombocytosis is seen

16- Which of the following is not true for immune thrombocytopenia (ITP)?
A) D-dimer level is normal.
OB) Prothrombin time is prolonged
C) First line therapy is glucocorticoids.
D) There is no need for bone marrow biopsy during diagnosis.
E) IVIG can be used in emergent situations.

17- Which blood is the least used in today's transfusion practice?


A) Erythrocyte solution
B) Platelet solution
OC) whole blood
D) Fresh frozen plasma
E) Cryoprecipitate
18- The use of thrombocyte suspension for prophylaxis in thrombocytopenic patients is
generally done when platelets fall below 10,000. However in some special cases this
threshold level is not taken into account. Which of the following is not one of these special
cases?
A) Thrombotic thrombocytopenic purpura (TTP)
B) Immune thrombocytopenia (ITP)
C) Type II heparin-associated thrombocytopenia (HIT)
D) HELLP syndrome

OE) Acute leukemia patient receiving induction therapy

19- Which of the following is not a feature of megaloblastic anemias?


A) Prominent macroovalocytes in the peripheral smear
OB) Reticulocytosis
C) Hypersegmentation of neutrophils
D) Increased levels of homocysteine
E) Increased levels of lactate dehydrogenase

20- A 70-year-old patient who presents…. Complaints of weakness, fatigue, night….. and weight
loss is diagnosed with chronic …. Lymphocytic leukemia. Lymphadenopathy….
Hepato-splenomegaly are not detected in the physical examination of the patient. In the
hemogram , Hgb : 14 g/dL, leukocytes: 250x10 9 /L(250.000) and platelets are measured as
136x10 9 /L(136.000). What is the stage of this patient according to the Rai staging system?
A) Rai Stage 0
B) Rai Stage I
C) Rai Stage II
D) Rai Stage III
OE) Rai Stage IV

21- Which of the following matching of metabolic bone disease and defective mechanism is
incorrect?
OA) Hyperparathyroidism – Low bone turnover
B) Osteoporosis – Decreased bone density
C) Osteopetrosis – Increased bone density
D) Osteogenesis imperfecta – Decreased bone density
E) Rickets – Mineralization defect

22- Which disease is frequently involved in the etiology of a patient presenting with acute renal
failure?
A) Acute myeloblastic leukemia
B) Chronic lymphocytic leukemia
C) Waldenstrom macroglobulinemia
OD) multiple myeloma
E) Diffuse large B-cell lymphoma
23- Which of the following is not a cause of iron deficiency anemia?
A) hemorrhoids
B) pregnancy
C) Achlorhydria
D) celiac disease
OE) Ileal resection

24- Which of the following drugs is specifically used for the treatment of transfusion-dependent
low-risk myelodysplastic syndrome with ring sideroblasts ?
A) Lenalidomide
OB) luspatercept
C) Azacitidine
D) imatinib
E) blinatumomab

25- Which of the following is incorrect about antidiabetic agents?


A) Sulfonylureas may cause hypoglycemia and weight gain
B) The usage of sulfonylureas are contraindicated in pregnancy
OC) Insulin is contraindicated in hypoglycemic chose diabetic patients
D) Metformin is contraindicated in a patient with a history of lactic acidosis during prior
metformin therapy
E) SGLT-2 inhibitors increase glucose excretion by kidneys

26- Which of the following information is not correct about diabetes mellitus?
A) Beta cell functions are enough to prevent ketosis in initial years of LADA.
OB) Genetic penetrance is strong in type 1 diabetes patients.
C) MODY should be considered in patients with early onset diabetes without insulin resistance
that is treatable with sulfonylurea .
D) Insulin islet antibodies can be positive in diabetic patients without autoimmunity etiology.
E) In prediabetic patients the risk of retinopathy is increased.

27- Which of the following is not correct about the complications of diabetes mellitus?
A) Improvement and progression of diabetic nephropathy have been associated with glycemic
control.
B) Infections may precipitate ketoacidosis .
O C) NaHCO3, fluid and insulin should be given to diabetic patients with diabetic ketoacidosis
with a pH of 7.1.
D) The rising blood pressure increases the risk of retinopathy.
E) Water and sodium deficits should not be corrected rapidly in acute hyperglycemic
complications.
28- Which of the mutated genes is not associated with unfavorable prognosis in
myelodysplastic syndromes?
A) TP53
B) ASXL1
C) EZH2
OD) SF3B1
E) ETV6

29- Which of the following laboratory findings can be expected in a patient with parathyroid
cancer?
A) Increased calcium, increased phosphorus and increased parathyroid hormone
OB) Increased calcium, decreased phosphorus and increased parathyroid hormone
C) Decreased calcium, increased phosphorus and increased parathyroid hormone
D) Decreased calcium, increased phosphorus and decreased parathyroid hormone
E) Decreased calcium, decreased phosphorus and increased parathyroid hormone

30-Which of the following statement about denosumab is wrong?


A)It is a monoclonal antibody against RANKL (Receptor activator of nuclear factor KB ligand)
B)It inhibits osteoclast formation, function and life span
C)It can be used for the treatment of osteoporosis in postmenopausal women and men at
high risk of fracture
D) Osteonecrosis of jaw is a rare side effect
O
E) It is used orally once a week

31- Which can not be the etiology of hypertension of a patient whose aldosterone and renin
levels are both high?
A) malignant hypertension
B) pregnancy
C) Renin producing tumors
OD) Aldosterone producing adrenal adenoma
E) Renal artery stenosis

32- a 62 years old man with a history of coronary bypass graft three years ago presents with
severe myalgia and muscle weakness. His medications are metoprolol 1 x 150 mg,
valsartan 1x 160 mg, acetylsalicylic acid 1x300 mg and rosuvastatin 1x20 mg. Which of the
following tests should be measured first to identify the cause of myalgia?
A) Liver function tests
B) Thyrotrophin (TSH)
OC) Creatinine kinase
D) Cortisol
E) total bilirubin
33- Which is incorrect about adrenal incidentalomas ?
A) They are detected in radiologic examinations performed for adrenal gland unrelated
symptoms.
OB) Aldosterone and renin should be checked in all patients regardless of hypertension
presence.???
C) Malignancy rate is high for lesions larger than 4 cm in size.
D) Non-contrast computed tomography is the first choice for imaging.
E) Signal loss in outphase sequences in MRI is a benign finding showing the mass is lipid rich
adenoma.

34- A 54 years old woman with a diagnosis of type 2 diabetes and hypertension for 7 years
states that she wants to lose weight. Her height is 165 cm, weight is 92 kg and body mass
index is 33.7 kg/m 2 . She is using metformin 2x1000 mg/day and valsartan 160 mg/day.
She adheres to her diet and exercises regularly, but states that she cannot lose weight. Her
fasting blood glucose is 156 mg/dL and HbA1c is 8.5%. Biochemical examination and
thyroid function tests are normal. Considering the accompanying obesity, which of the
following is a good option for this patient?
A) Lifestyle changes and follow-up 3 months later
OB) Adding exenatide
C) Increasing the dose of metformin
D) Adding basal insulin
E) Adding a sulfonylurea

35- However, primary treatment is surgery in patients with Cushing's disease, medical treatment
can be used in some situations. Which one is wrong about medical therapy in patients with
Cushing's disease?
A) Medical therapy is used in patients without localized tumors in hypophysis.
B) Medical therapy is used in the period after radiotherapy, before initiation of effect of
radiotherapy.
C) Medical therapy can be used in the preoperative period in order to control undesired side
effects of cortisol and to decrease risk of surgery.
D) Medical therapy is a choice in patients who refuse surgery
OE) Metyrapone and ketoconazole are the medical treatment choices which exert their effects
primary on hypophysis via inhibition of ACTH secretion

36- Pregnancy is one of the physiological causes of hyperprolactinemia . Which of the following
is wrong about hyperprolactinemia associated with pregnancy?
A) Lactotroph hyperplasia occurs in pregnancy
B) The cause of hyperprolactinemia is the increase in serum estradiol concentrations during
pregnancy
OC) Serum prolactin concentration usually returns to normal in the pregnant women just after
delivery
D) In pregnant patients with prolactinoma , serum prolactin concentration is not useful in the
diagnosis follow up.
E) Serum prolactin increases throughout pregnancy and reaches a peak at delivery.
37- Which of the following cytotoxic drugs has a linear dose response curve?
A) vincristine
B) Etoposide
C) paclitaxel
GD) Ifosfamide
E) 5-fluorouracil

38- Which of the following drugs is a monoclonal antibody used in the treatment of cancer?
A) cyclophosphamide
B) Methylprednisolone
OC) Rituximab
D) Erlotinib
E) temsirolimus

39- Which can't be used for differential diagnosis of Cushing's syndrome?


A) High dose dexamethasone suppression test
B) CRH stimulation test
C) Serum potassium level
D) ACTH
OE) 24 hour urinary free cortisol

40- 2 days after the parenteral administration of cisplatin … mg chemotherapy is 59 year-old


male patient with metastatic lung …. cell cancer, the patient develops nausea and fatigue. …
Biochemical tests are performed and an increase in serum urea and creatinine levels is
detected. In this case, which of the following comments should be considered first?
A) Hepatototoxicity in prophylactic antiemetic treatments
OB) Cisplatin induced nephrotoxicity
C) Gemcitabine induced nephrotoxicity
D) Chemotherapy induced myelotoxicity
E) Peripheral sensory neuropathy due to cisplatin

41- Which one of the following is not true for oncologic emergencies?
A) Tumor lysis syndrome consists of hyperpotassemia , hyperphosphatemia , hypocalcemia,
hyperuricemia at various levels.
OB) T-wave flattening is the ECG findings of hyperkalemia .
C) In hypercalcemia , the treatment should be started with fluid replacement.
D) Headache, nausea and vomiting may occur in the course of intracranial pressure elevation
syndrome.
E) In malignant pericardial tamponade heart sounds may decrease.
42- Choose the appropriate match for cancer screening
A) Rectal cancer – High resolution rectal MRI
OB) Cervical cancer – Cervical smear
C) Ovarian cancer – Abdominal tomography
D) Breast cancer – Breast MRI imaging
E) Testicular tumor – Alpha fetoprotein and beta HCG

43- The hallmarks of cancer conceptualization is the heuristic tool for distilling the vast
complexity of cancer phenotypes and genotypes into a provisional set of underlying
principles. Which of the below is not one of the hallmarks of cancer?
A) Evading growth suppressors
B) Avoiding immune destruction
OC) Enabling replicative mortality
D) Tumor-promoting inflammation
E) Genome instability & mutation

44- Which of the following is true about the treatment approach to neutropenic fever?
A) The patient should always be treated as an outpatient to avoid hospital flora.
B) The most common agents causing neutropenic fever are viruses.
C) All chemotherapeutic drugs cause neutropenia with the same frequency.
OD) Presence of hypotension and COPD (Chronic obstructive pulmonary disease) are the worst
risk factors.
E) Antibiotic treatment should be started in 60 minutes after the blood culture results.

45- Which of the following is wrong about cancer etiology and cancer epidemiology?
A) Genetic factors constitute a small percent of the cancer causes
B) Screening with colonoscopy for colon cancer is an example for secondary prevention for
cancer
C) There is an association between high sodium content food consumption and gastric cancer
OD) The addition of vitamin preparations to the diet always creates a protective effect against
cancer
E) The most important changeable risk factor for cancer is cigarette smoking

46- Which of the following explanations about tumor markers is wrong?


A) It is used as a prognostic indicator of disease progression and to evaluate the success of
the treatment
OB) In the case of recurrence of cancer, marker decreases again???
C) Tumor marker values correlate with the effectiveness of treatment
D) Clinical staging of cancer and used to estimate tumor volume
E) Tumor marks can be classified as enzymes, hormones, oncofetal antigens, receptors
47- Which one of the following is not true for thyroiditis?
OA) In the subacute granulomatous thyroiditis treatment prednisolone is always used.
B) In the radiation thyroiditis thyroid gland is usually hard and painful with palpation.
C) Subacute granulomatous thyroiditis is painful, noninfectious and self-limited disease of the
thyroid gland
D) Subacute lymphocytic thyroiditis is the autoimmune inflammation of the thyroid gland
E) In the Riedel thyroiditis cytology can not be sufficient due to the fibrosis

48- Which one of the following is not true for thyroid cancers?
A) Papillary thyroid cancer is one of the differentiated thyroid cancers
B) External irradiation of the neck increases the risk for papillary thyroid cancer
C) Biological behaviors of papillary and follicular thyroid cancers are different
OD) Serum calcitonin level is used as a tumor marker in patients with differentiated thyroid
cancer
E) Medullary thyroid cancer may be a component of the multiple endocrine neoplasia (MEN)

49- Which one of the following is not true for hypothyroidism?


OA) Iodine deficiency is the most common cause of hypothyroidism in developed countries
B) The combination of a low serum free thyroxine (fT4) and elevated serum thyroid stimulating
hormone (TSH) levels are diagnostic for primary hypothyroidism.
C) Serum thyroid stimulating hormone (TSH) is used in the treatment follow-up of primary
hypothyroidism
D) Slowdown in movements and speaking, hoarseness are some of the signs of
hypothyroidism
E) People who use some drugs such as lithium, amiodarone , IFN-alpha are candidates for
hypothyroidism

50- Which of the following is not prognostic at the time of diagnosis in IgA nephropathy ?
A) Proteinuria level
B) High blood pressure
C) hyperuricemia
D) Presence of interstitial fibrosis in kidney biopsy
OE) Serum IgA level

51- Which one of the following is not true about hyperthyroidism or thyrotoxicosis?
A) The most common cause of thyrotoxicosis is Graves disease
B) The presence of thyroid stimulating antibody ( TSab ) is positively correlated with active
disease in diffuse toxic goiter
C) Radioactive iodine uptake (RAIU) is increased in Graves' disease
D) Thyroglobulin levels is in the diagnosis of thyrotoxicosis due to exogenous thyroid hormone
OE) Methimazole inhibits peripheral T4 to T3 conversion
52- Which one of the following is not true about nodular thyroid ……?
A) Thyroid nodules are the lesions which are different from the surrounding thyroid
parenchyma and can be visualized radiologically.
B) Most of the patients with thyroid nodules are euthyroid
C) Serum TSH level should be the initial laboratory study in the evaluation of a thyroid nodule
D) Thyroid fine needle aspiration biopsy is the most reliable test to distinguish benign from
malignant nodules

OE) Microcalcifications in a thyroid nodule is the characteristic feature of the benign thyroid
nodules

53- In patients with diabetic nephropathy BP should be controlled aggressively, usually


beginning with which of the following?
A) Diuretics
B) statins
C) Calcium channel blockers
OD) Angiotensin inhibition
E) alpha blockers

54- Which of the following statements about gout is false?


A) Gout usually progresses as an intermittent arthritis.
B) Target uric acid level in gout maintenance treatment is <6 mg/dL
OC) Inflammation by calcium pyrophosphate dihydrate crystals deposition in the joint fluid
causes gout
D) Steroids and/or colchicine and/or NSAIDs are used to treat acute gout
E) Patients with gout with a history of nephrolithiasis should be given uric acid-lowering
treatment

55- Proteinuria and hematuria were found in the urine analysis of a patient in whom Anti-GBM
antibodies were detected in the serum. Which of the following is the most likely diagnosis?
A) minimal change disease
OB) Rapidly developing glomerulonephritis
C) Interstitial nephritis
D) Focal glomerulosclerosis
E) IgA nephropathy

56- What maintains the difference in concentration between intracellular fluid compartment
and extracellular fluid compartment?
A) Osmotic pressure
B) Free movement of urea
OC) Sodium-potassium pump
D) Intracellular proteins
E) Cell membrane bicarbonate conductance pathways
57- Which of the following is false regarding spondyloarthritis ?
OA) It has close relationship with HLAB51
B) The axial skeleton is frequently affected.
C) Peripheral arthritis may accompany the disease.
D) Mucocutaneous findings are among the disease-specific changes
E) Nonsteroidal anti-inflammatory drugs are one of the main drugs

58- Which of the following drug group-mechanisms of action pairings is incorrect?


A) Rituximab – Anti CD20
OB) Adalimumab – IL-17 blockade
C) Anakinra – IL 1 blockage
D) Etanercept – TNF alpha blockade
E) Tocilizumab – IL 6 blockade

59- Which of the following is not an extraglandular manifestation of Sjogren's syndrome?


A) Distal tubular renal acidosis
OB) xerostomia
C) arthritis
D) Lymphocytic interstitial pneumonia
E) vasculitis

60- 36-year-old female patient, who has been under follow-up for 6 years with the diagnosis of
Seropositive Rheumatoid Arthritis, has swelling and tenderness in her wrists, proximal
interphalangeal joints for 4 months, and the treatment of the patient is methotrexate 15
mg/week, hydroxychloroquine 200 mg 2x1, salazopyrin 500 mg tb 3x2 and The ESR : 56,
CRP:40, DAS 28 score of the patient is calculated as 6.7. Which one should be used in the
next step in the treatment of the patient with a previous pulmonary embolism?
OA) etanercept
B) Upadacitinip
C) cyclophosphamide
D) azathioprine
E) leflunomide

61- Which of the following is false regarding familial mediterranean fever (FMF) disease?
A) It is one of the autoinflammatory syndromes
B) It is genetically inherited
C) Occurs in patients over 40 years of age
D) Recurrent fever, abdominal pain and musculoskeletal findings are the most common
symptoms .
E) Mutations in the MEFV gene are responsible for diabetes
62- A 49-year-old woman was diagnosed with type 2 diabetes 9 years ago and now she
presents with polyuria and polydipsia. One year ago, microalbuminuria was detected on an
annual urine screen, on a spot urine sample. Now her blood pressure is 140/90 mmHg and
her diabetes has been under fair control with a most recent hemoglobin A1c of 8.4%, eGFR:
65 mL/min/1.73m2. Urine dipstick shows both glycosuria and proteinuria. A renal biopsy is
taken. Which of the following is most likely to be found in the glomeruli?
A) IgA and C3 deposition
OB) Mesangial expansion, nodular sclerosis
C) IgM and C4 deposition
D) T-lymphocytes infiltrating the interstitium
E) IgG and C4 deposition

63- Which of the following is not a feature of cancer cells?


A) Escape from apoptosis
B) metastasis
C) Angiogenesis
OD) genetic stability
E) Loss of contact inhibition

64- Which of the following is the most common benign non epithelial tumor of esophagus?
OA) leiomyoma
B) Granular cell tumor
C) Fibrovascular polyp
D) hemangioma
E) lipoma

65- A 35-year-old male presents with sudden onset unilateral hearing loss in the left ear as well
as tinnitus. He has no preceding symptoms such as sinus congestion or nasal crusting, no
fevers or sore throat. He does note a history of keratitis of his eyes 2 months ago that was
treated with topical steroids by his ophthalmologist and has since been resolved. He has no
headache, vision symptoms or vertigo. He is seen by ENT who performs audiology
demonstrating a large decrease in both high frequency and low frequency hearing. MRI of
the brain is performed to evaluate for structural abnormalities that could explain the hearing
loss, but does demonstrate mild enhancement in the inner ear consistent with labyrinthitis.
Because of the sudden onset and potential autoimmune cause, he is given prednisone 60
mg, which rapidly resolves the hearing loss to baseline. He is sent to rheumatology for
evaluation of autoimmune inner ear disease. Which of the following studies are reasonable
to order for this patient?
A) Abdominal USG
B) Carotid MRI
OC) CT angiography aorta
D) Cerebral angiogram
E) cranial biopsy
66- Which of the following is correct about Adult onset Still Disease?
A) Still's rash develops independently of fever
B) Fever is constant throughout the day
C) Rheumatoid factor is generally positive.
OD) Ferritin value is generally increased by 5 times normal.
E) Pancytopenia is common

67- A 64-year-old female presents with progressive shortness of breath and hemoptysis after
multiple months' history of progressive sinus congestion and nasal crusting. In the
emergency room, findings are as follows: Pulse oxygenation measures 85% despite high
flow nasal cannula; White blood cell (WBC): 15K/ μL (normal: 3.7-11.0 K/ μL ); platelets are
511 K/ μ L (normal: 150-400 K/ μ L); creatinine is 3.6 mg/dL (normal 0.58-0.96), and
urinalysis with 2+ blood. Physical exam demonstrates fine crackles in diffuse lung fields,
Chest X-ray demonstrates bilateral patchy infiltrates. Bronchoscopy confirms diffuse
alveolar hemorrhage; her bronchoalveolar lavage is negative for infectious workup,
including tuberculosis. Biopsy of kidney is pursued. Test results are positive for P-ANCA,
MPO. Which of the following renal immunohistochemistry is most consistent with the
suspected diagnosis?
A) Linear IgG staining
B) Positive IgG, M and C1q/C3
C) Positive IgA
OD) Minimal immunofluorescence
E) Positive IgG

68- Which of the following is a motility disorder with esophagogastric junction disorder?
A) Absence of contractility
B) Distal esophageal spasm
C) Hypercontractile esophagus
GD) achalasia
E) Ineffective esophageal motility

69- A 55 year old male patient presents with belt-like abdominal pain radiating to the back and
….. increases in severity while leaning forward. Amylase and lipase levels are increased.
ALT-AST levels are about two times the upper limit of normal. Which of the evaluation and
treatment approaches is not appropriate for this patient?
OA) Restriction of oral intake for at least 1 week to decrease the severity of pancreatitis
B) Aggressive fluid resuscitation in the first 24 hours
C) MR cholangiography for evaluation regarding choledocholithiasis
D) History taking about drug intake
E) Meperidine administration for pain palliation
70- A 35-year-old female patient, who has been followed up with the diagnosis of systemic
lupus erythematosus for two years, presents to the emergency department with complaints
of sudden onset of shortness of breath, tachypnea, and cough. It is learned from his history
that he used steroids and cyclophosphamide for nephritis, and used azathioprine 150
mg/day, prednisolone 15 mg/day and hydroxychloroquine 400 mg/day as maintenance
treatment. The lung diffusion capacity of the patient with parenchymal infiltrates on chest
X-ray has increased. Anti-dsDNA is 3 times higher and hemoglobin: 7 g/dL. The most likely
diagnosis for this patient?
A) lupus pneumonia
&B) Diffuse alveolar hemorrhage
C) Pulmonary arterial hypertension
D) Pneumocystis jiroveci infection
E) tuberculosis

71- Which of the following statements about joint pathology is correct?


A) Ankle arthritis that persists for 5 weeks is called chronic monoarthritis
B) Axial involvement; is an asymmetrical involvement of the knee and ankle joints
C) Arthralgia; it is defined by signs of pain, swelling, and heat increase in the joint
D) Pain increases with movement in inflammatory joint pathologies
OE) Arthritis in a total of 5 joints is defined as polyarthritis

72- Which of the following is not a typical clinical finding of Behçet's disease?
A) oral aphthae
&B) myositis
C) genital aphthae
D) uveitis
E) Deep vein thrombosis

73- Which of the following is not a relative risk factor for the development of malignancy in
pancreatic cystic neoplasms?
OA) Pancreatic duct diameter greater than 10 mm
B) Thickened cyst wall
C) Recurrent acute pancreatitis
D) Increased CA 19-9 level
E) Cyst size greater than 3cm

74- Which of the following is not one of the peripherally acting μ opioid antagonists?
A) naloxegol
B) Alvimopan
:C) lubiprostone
D) Methylnaltrexone
E) Naldemedine
75- Which of the following is true for gastrointestinal polyposis syndromes?
I. Peutz-Jeghers syndrome is one of the hamartomatous polyposis syndromes. It is
autosomal dominant.
II. Turcot syndrome is a variant of Familial Adenomatous polyposis (FAP) and may be
associated with primary tumors of the central nervous system ( medulloblastoma ,
glioblastoma multiforme )
III. Colon cancer develops at an earlier age in Attenue Familial Adenomatous polyposis (FAP)
compared to classical FAP
IV. Classical Familial Adenomatous polyposis (FAP), adenomas begin to form at the age of
10-12, therefore annual sigmoidoscopy should be started at this age.
O A) I, II, IV is true
B) Only III is correct
C) All are true
D) Only I and II are correct
E) Only II and IV are correct

76- Which of the following is wrong regarding the extraintestinal manifestations of


inflammatory bowel disease (IBD)?
A) Extraintestinal manifestations can be diagnosed before, at the same time and after the
diagnosis of IBD
B) Erythema nodosum lesions are painful red-purple nodules whose severity parallels the
course of IBD
OC) Ankylosing spondylitis undergoes …. With IBD treatment
D) Urgent treatment of uveitis is important to prevent permanent vision loss
E) Risk of cholangiocellular carcinoma is increased in primary sclerosing cholangitis.

77- The patient, who was diagnosed with ulcerative colitis 2 years ago and was using oral
mesalazine, applied to the emergency department with the complaints of severe bloody
diarrhea and abdominal pain. Which is the most correct approach?
O A) Treatment should be planned after evaluating the patient's hydration and nutritional status
and excluding infectious causes such as CMV, C. Difficile, and amebiasis.
B) If a toxic megacolon is suspected, emergency colonoscopy should be planned.
C) Azathioprine should be added to his treatment immediately. If oral intake is insufficient, IV
cyclosporine should be started.
D) After evaluating the involvement of the disease with MR enterography or capsule
endoscopy, a treatment plan should be made.
E) Oral feeding should be cut immediately and surgery should be planned.

78- It would be correct to say which of the following is one of the aims of treatment in
Budd-Chiari Syndrome?
A) Anticoagulation to prevent the spread of the clot
B) Thrombolysis , angioplasty/ stenting to restore function of thrombosed vein
C) transjugular intrahepatic portosystemic shunt (TIPS) application surgical shunt to
decompress the congested liver
D) Portal hypertension therapy, liver transplant to prevent and control complications
O E) Treatment should be aimed at increasing cardiac output and systemic oxygenation
79- Which of the following is false about carcinoid syndrome?
A) Serotonin and other vasoactive amines are responsible.
OB) The most common symptom is chronic secretory diarrhea
C) Skin flushing is most common in the face, head and neck region
D) Consumption of tyramine-containing food provokes flushing symptoms
E) It is.common in gastric neuroendocrine tumor

80- A 60 year old with known chronic liver disease due to HBV presents to the emergency
department with confusion. Fingertip blood sugar is 150 and physical examination reveals
flapping tremor. Which of the following is the least probable condition causing this clinical
picture?
A) Spontaneous bacterial peritonitis
B) Variceal bleeding
C) Hypokalemia
OD) diarrhea
E) Overuse of diuretics

81- Which of the following is wrong regarding Hepatitis B infection?


A) Marker for window period is anti HBc IgM
B) Hbe Ag (+), HBV DNA very high and transaminases are normal in immunotolerant phase
OC) Treatment target is loss of cccDNA
D) Inactive carriers should also be screened for hepatocellular carcinoma
E) systemicIf HBV DNA is above 20000 IU/ml during pregnancy antiviral treatment should be
initiated in the third trimester to prevent vertical transmission

82- Which of the following acute liver disease-treatment pairs is wrong?


OA) Wilson's disease – Deferoxamine treat men
B) Acetaminophen toxicity – N-acetyl cysteine infusion
C) Mushroom poisoning – Oral active charcoal, IV penicillin G
D) Herpes simplex virus – acyclovir
E) Acute fatty liver of pregnancy – Termination of pregnancy

83- A 28 week pregnant female patient presents with pruritus that becomes worse at night.
Laboratory results are ALT: 50, AST: 55, OGT: 250, bilirubin total: 1.8. Which of the following
is not expected for this patient?
A) Even serum acids >10 μmol/L
B) Family history of cholestasis during pregnancy
C) Recovery immediately starts after delivery
OD) Hypoglycemia
E) Recurrence in other pregnancies
84- Which of the following is wrong for the pathophysiology of portal hypertension and
cirrhosis?
OA) Increased intrahepatic nitric oxide is due to endothelial dysfunction
B) Collaterals at distal esophagus and proximal stomach develop in portal hypertension
C) Capillarization of sinusoids is due to deposition of fibrous tissue at the space of Disse .
D) Nonselective beta blockers decrease portal pressure by constriction of splanchnic arteriolar
smooth muscle cells
E) Activation of hepatic stellate cells stimulate extracellular matrix synthesis in the liver

85- Which of the following is wrong for alcohol related liver disease?
A) Abstinence is the most important prognostic factor
B) Steroid treatment is recommended in suitable patients with acute alcoholic hepatitis
C) Some genetic factors may increase the risk of disease
OD) Increased liver glutathione levels are important in the pathogenesis
E) Differentiation of nonalcoholic fatty liver and alcohol related fatty liver is difficult on
pathology slides

86- In which of the following diseases serum ascites albumin gradient is <1.1 gr /dL?
A) Cirrhosis
B) Budd-Chiari syndrome
OC) Ovarian cancer
D) Constrictive pericarditis
E) Right heart failure

87- Which of the following is wrong for hepatocellular cancer (HCC)?


A) Washout sign is typical at dynamic imaging
B) Cirrhosis is an important risk factor
C) While making treatment decisions both the stage of HCC and the degree of liver
dysfunction should be considered
D) Radiofrequency ablation can be used in treatment
OE) HCC is the most frequent malignant liver tumor

88- A 45 year old female patient presents with fatigue and pruritus that increases in severity at
night. Laboratory results show increased ALP-GGT levels. Liver biopsy reveals granulomas
in portal areas. Which of the following is true for this patient?
A) MR cholangiography shows chain of lakes appearance
Oo
B) BUT positivity is detected AMA
C) Serum IgG levels are increased
D) Coexistence with inflammatory bowel disease is frequent
E) Response to steroids is good
89- A 57-year-old woman was admitted to the emergency room with a 10-day history of
epigastric pain and fever. Physical examination revealed jaundice and right upper quadrant
tenderness with superficial palpation. Vital signs were normal. Laboratory studies; total
bilirubin 7.0 mg/dL, direct bilirubin 4.8 mg/dL. Alkaline phosphatase 680 U/L (25-100 U/L)
and GGT 121 U/L (10-50 U/L), ALT 145 U/L, AST 92U/L. Albumin and INR-PTT were normal.
Ultrasonography of the abdomen showed biliary duct dilatation and hyperechogenic even
duct stones. Which of the following is the most appropriate next step in the management of
this patient?
A) Upper Gastrointestinal Endoscopy
B) Computed Tomography
C) Biochemical (viral and metabolic) studies for liver parenchymal disease
OD) Endoscopic retrograde cholangiopancreatography
E) Endoscopic ultrasonography

90- Which of the following statements is not true about biochemical, functional tests and
metabolic disease of the liver?
A) Aminotransferases (ALT, AST) and alkaline phosphatase (ALP) do not measure function of
the liver
B) Serum albumin and prolonged prothrombin time and INR show liver functions.
OC) The first step in the evaluation of mildly …. Serum aminotransferase (ALT, AST) …..
ultrasonographic examination
D) Wilson's disease is an autosomal-recessive disease with a defective gene ATP7B on
chromosome 13.
E) Phlebotomy and deferoxamine is used in Hemochromatosis treatment

91- Which of the following information about peptic ulcer, functional dyspepsia and alarm
features is not true?
A) Ulcer in the gastrointestinal tract can be defined as a 5 mm or larger break in the lining of
the mucosa and submucosa. An erosion is a break less than 5 mm.
B) The main risk factors of peptic ulcer disease are Helicobacter pylori infection and
non-steroidal anti-inflammatory drug use
C) Jaundice and palpable abdominal mass are alarm features that should prompt an upper
endoscopy or other tests.
D) Diagnostic criteria of functional dyspepsia include bothersome postprandial fullness, early
satiation, epigastric pain and epigastric burning.
OE) a 38-years-old female patient with epigastric pain for 8 months should have an upper
gastrointestinal endoscopy immediately.
92- a 69-year-old female patient was diagnosed with hypertension in the outpatient clinic. After
evaluating the patient, it is decided that drug therapy should be started in addition to
lifestyle changes. Which of the following drugs is not the first choice for this patient?
A) Angiotensin converting enzyme inhibitors
B) Angiotensin receptor blockers
OC) alpha blockers
D) Calcium channel blockers
E) Diuretics
93- Which one of the following is wrong regarding delirium?
A) The major predisposing risk factor for delirium is preexisting cognitive impairment.
B) Three forms of delirium have been recognized; hyperactive, hypoactive, and mixed types.
Most common type of delirium is hypoactive type.
C) One of the medications most frequently associated with delirium is drugs with
anticholinergic effects.
OD) Main treatment of all delirium types is anti psychotic medications.???
E) The cardinal features of delirium are acute onset and fluctuating course.

94- Which of the following is among the inappropriate/over prescribed medications in the older
population?
A) oral antidiabetics
OB) Anticholinergic agents
C) statins
D) Antihypertensives
E) Iron supplements

95- What is the most likely dementia diagnosis in an elderly patient presenting with personality
changes as a predominant symptom?
A) Vascular dementia
B) Alzheimer's disease
C) parkinson dementia
OD) frontotemporal dementia
E) Lewy body dementia

96- Which of the following is incorrect?


A) It is important to consider the patient's remaining life expectancy and cognitive status when
deciding which preventative health measures to offer.
OB) In the scans, the target is determined according to the chronological age.
C) Low-dose thoracic CT is recommended for lung cancer screening in high-risk patients up to
80 years of age.
D) Annual influenza, pneumococcal and herpes vaccination is recommended for older people.
E) For colon cancer screening, decisions about screening for the first time after age 75 must
be made in the context of patient health and life expectancy.

97- Which of the following is not one of the Basic activities of daily living (KATZ)?
A) Transfer
B) toileting
C) Nutrition
OD) Organizing medications
E) wearing
98- Which of the following is incorrect?
A) Intrinsic factors are more and more important than extrinsic factors in the cause of falls in
older people.
B) Past history of falls is the most important risk factor.
C) One of the causes of urinary incontinence in the elderly is the increase of involuntary
detrusor muscle contractions with aging.
O D) The most common form of urinary incontinence in the elderly is urge incontinence. It's
characterized by involuntary leakage of small amounts of urine.
E) It is pathological that more than 200 ml urine remains in the bladder after voiding.

99- Which of the following is incorrect?


A) The amount of calcium that individuals over the age of 70 should take daily is about
1000-1200 mg. The amount of Vitamin D is 800 U, the amount of protein is 1-1.2 g/kg, the
amount of energy is 1800 kcal/day for man and 1400 kcal/day for woman.
B) Strict dietary restrictions are not appropriate in older people. Meals should be frequent and
in small quantities.
C) Depression is an important cause of malnutrition in older people.
D) Malnutrition can cause pressure ulcers. It can also cause pressure ulcers not to heal.
O E) Muscle loss in older patients is only due to aging.

E2
1- Which one is a TNF-a blocker?
O
A) infliximab
B) Rituximab
C) Tocilizumab
D) Abatacept
E) tofacitinib
2- 70 year old patient. Neck swelling for 1 Year. Lymph Node enlargement in cervical and
axillary regions. What is the first step in diagnosis?
OA) Excision
B) Check Hemogram
C) Fine needle aspiration
D) Bone marrow biopsy
OE) Trucut biopsy
3- Diabetic in nephropathy preference The first antihypertensive drug What is he was asking .
A) ACE
4- Which of the following is wrong?
A) Vaccination is an example of secondary prevention
5- Which of the following does lead to normal anion gap metabolic acidosis?
A) Salicylate intoxication
B) Diabetic Ketoacidosis
②C) Renal Tubular acidosis
D) Ethanol intoxication
E) Paracetamol poisoning
6- Which of the following plasmapheresis is the most suitable for TTP patients?
A) dextrose
B) saline
OC) Fresh frozen plasma

7- Which diabetic nephropathy early Is it a finding ?


A) microalbuminuria

8- a 55 year old man with known alcohol related liver disease presents with jaundice and
impaired mentation. In physical examination he has jaundice, disorientation and flapping
tremor. INR 2.5, ALT 250, AST 300, bilirubin 26, creatinine 1.7. Which following option is
false?
A) Search for possible infection
OB) Transfusion of fresh frozen plasma to decrease INR below 1.5
C) Steroid if maddrey calculation >32
D) Serial measurement of blood glucose
E) Prophylactic intubation if encephalopathy gets worse

9- Which one of the following is not true for thyroiditis?


OA) In the subacute granulomatous thyroiditis treatment prednisolone is always used.
B) In the radiation thyroiditis thyroid gland is usually hard and painful with palpation.
C) Subacute granulomatous thyroiditis is painful, noninfectious and self-limited disease of the
thyroid gland
D) Subacute lymphocytic thyroiditis is the autoimmune inflammation of the thyroid gland
E) In the Riedel thyroiditis cytology can not be sufficient due to the fibrosis

10- Massive splenomegaly Reason Isn't it ?


A) CML
B) Kala azar
OC) Sickle cellular anemia
D) gaucher
E) hairy cell leukemia

11- 45- year-old patient fatigue And breath with its narrowness is coming . Pale in view
petechiae And external flesh hypertrophy is detected . Hemoglobin 7, white sphere 250
10*9, platelets 24 10*9. for diagnosis: The first test you want, why ?
A) usg
OB) Peripheral spreading
C) complete blood count
D) Thorax CT angiography
E) D-dimer
12- Orlistat contraindicated the situation?

⑧A)
B)
Pregnancy
UC
C) hyperlipidemia
13- Which is not included in the differential diagnosis in a patient with hypocalcemia and
hypoparathyroid level?
A) Vitamin D deficiency
OB) Autoimmune polyglandular syndrome type 1
C) Vit d dependent rickets type 1
D) Chronic kidney disease
E) Pseudohypoparathyroidism

14- Most common cause of primary budd chiari


A) Myeloproliferative disease
B) Malignant tumor invasion
C) PLD
D) Blunt trauma
OE) Liver abscess +inflammation

15- Which of the following is not one of the definite causes of gastric cancer?
OA)
B)
statin
Billroth II
C) helicobacter pylori
D) Chronic atrophic gastritis
E) Adenomatous gastric polyps

16- Which of the following is not a treatment option in achalasia?


A) Heller myotomy
B) POEM
C) balloon dilation
D) botulinum toxin
OE) sclerotherapy

17- myelofibrosis for is wrong


A) Clonal stem cell disorder
B)
C)
The most common finding anemia
Only treatment hematopoietic stem cell transplant ?
D) JAK-2 is available to everyone positive
E) Hairy cell leukemia myelofibrosis does

18- Which of the following is wrong regarding Celiac disease?


OA) Screening is done with antigliadin A.
B) There is usually coexistence with other autoimmune disorders.
C) Villous atrophy, crypt hyperplasia and intraepithelial lymphocytes are seen in biopsy.
D) There might be recurrent abortion and osteoporosis in the patient's history.
E) Extraintestinal manifestation can be dermatitis herpetiformis .
19- 50 year old female patient with metastatic breast cancer (with multiple metastases to lung,
liver, bone and soft tissues) was given 3 cycles of cyclophosphamide + doxorubicin
combined chemotherapy, in clinical and radiological response evaluation, more than 50%
reduction in tumoral lesions was detected compared to pre treatment , how should this test
results defined?
A) complete response
OB) Partial response
C) Minimal response
D) stable disease
E) Progressive disease

20- Which of the following is false for the progression of chronic kidney disease (CKD)?
A) The rate of progression of CKD is influenced by age.
B) Female gender has a slower rate of progression compared with male
OC) Nephrotic range proteinuria is associated with decreasing rate of CKD progression
D) Tight control of glucose in diabetes mellitus can potentially slow the rate of progression of
CKD
E) Hyperuricemia may lead to progression of CKD

21- Which is not a predisposing condition to hepatic encephalopathy?


OA) hyperkalemia
B) Infections
C) GI bleeding
D) constipation
E) Diuretic overuse

22- Which of the following is a cause of microcytic anemia?


OA) lead poisoning
B) acute bleeding
C) Hemolytic anemia
D) Aplastic anemia
E) hypothyroidism

23- A patient is going to have a lumbar puncture, what is the amount of thrombocyte
suspension that should be transfused ( I can't remember the question exactly But like this
One thing )
A) 80000
B) 60000
OC) 50000
D) 40000
E) 30000
24- Which of the following is not true in (..) pathogenesis of portal hypertension?
OA) Increase in intrahepatic nitric oxide
B) Intestinal (...) causing endotoxemia AMD increased TNF-a level
C) Activation of hepatic stellate cells.
D) An increase in portal hypertension
E) Increase in intrahepatic resistance

25- Which of the following is correct about Non-Hodgkin Lymphomas (NHL)?


A) The most common NHL seen in our country is follicular lymphomas.
B) Autoimmune diseases do not increase the risk of NHL.
C) Burkitt lymphoma is among the indolent lymphomas.
OD) Anaplastic Large cell lymphoma is among the T cell neoplasms.
E) t(14;18) often accompanies Mantle lymphoma.

26- a 27 year old female with ulcerative colitis. Present with desire for pregnancy. Which is not
true?
A) IBD activity at the time of conception very important regarding pregnancy outcome
B) Anti TNF drugs should be stopped during pregnancy due to teratogenic risks
OC) 5_ASA are safe during pregnancy
D) Disease activity can be monitored by fecal calprotectin
E) Fertility rate are low in active Crohn patient and UC patient with IPAA

27-Which of the following statements is wrong about acute lymphoblastic leukemia (ALL)?
A)The most common ALL immunophenotype in childhood is B-ALL.
B)The most common ALL immunophenotype in adults is B-cell.
C)In B-ALL, a WBC count >30×10^9/L in the peripheral blood at the time of diagnosis is a
negative risk factor.
D) Being 36 years old is a negative risk factor.
O
E) Hyperdiploidy is a negative risk factor, especially in childhood ALL.

28- Which of the following is wrong for NAFLD?


A) Multiple hits hypothesis involving first increasing resistance and obesity later inflammation,
oxidative stress, iron overload apoptosis and adipokine dysfunction is currently accepted.
B) For the diagnosis lobular inflammation, steatosis, ballooning must be present.
C) Key pathogenic event is activation of the stellate cells
D) Lifestyle changing like diet and weight loss are fundamental in
OE) Metformin is beneficial to decrease liver inflammation.
29- Which of the following is not true for thrombophilia tests?
A) During acute thrombosis we cannot measure natural anticoagulants because of
consumption
B) Heparin decreases antithrombin level
C) Genetic tests can always be done
D) LA may be false positive during warfarin therapy
OE) Warfarine decreases AT level
T1
1) A 23-year-old male patient applies to the clinic due to erectile dysfunction. There is a decrease
in facial hair growth and gynecomastia. Additionally, during palpation, the testicles are palpated as
small and hard. What is he sick with?
A) Kallman syndrome
B) Klinefelter syndrome**
C) Androgenic deficiency
I don't remember the other options

2) Which of the following is incorrect regarding hyperfunction of the anterior pituitary gland?
A) Measurement of macroprolactin level with asymptomatic prolactin elevation
B) Performing dexamethasone suppression test for differential diagnosis in corticotroph cell
hyperfunction
C) The first treatment for GH-secreting adenoma is medical therapy**
D) Pituitary stalk incision causes hyperprolactinemia
E) primary ovarian failure leads to hypergonadotropic hypogonadism

3) Which of the following causes of thyrotoxicosis has increased radioactive iodine uptake?
A) subacute thyroiditis
B) silent thyroiditis
C) amiodarone type 1
All ↓ RAI uptake
D) amiodarone type 2
E) iatrogenic thyrotoxicosis

4) Which of the following is incorrect regarding thyroid diseases?


A-Iodine deficiency is the most common cause of endemic and sporadic goiter.
B-The most common cause of primary hypothyroidism in iodine sufficient areas is Hashimoto's
disease.
C-Lithium, iodine and amiodarone use causes hypothyroidism
O
D-Serum calcitonin level is used as a tumor marker in follicular thyroid carcinoma.
E-Papillary thyroid carcinoma develops from thyroid follicular cells

5) A 47-year-old male patient applies to the urology clinic with decreased sexual desire and
infertility. Which of the following may be seen in a patient diagnosed with hypogonadotropic
hypogonadism?
O
A) Adenoma in the pituitary gland
B) FSH-LH secreting adenoma
C) Prolactin-secreting adenoma
D) Contrast-enhanced pituitary MRI should be performed.
E) Other anterior pituitary function tests should be performed.
6) Which of the following findings comes first in a patient who applies to the outpatient clinic with
complaints of drinking too much water and urinating nearly 5 liters per day?
Suggests Nephrotic Diabetes Insipidus?
A) The patient has a history of head trauma
B) Decrease in the amount of urine during periods when the patient is asleep
C) Low plasma ADH level
O
D) The patient is using lithium
E) When ADH is given externally to the patient, urine osmolality increases.

7) Which of the following is not true for pituitary diseases?


A) The first treatment option in patients with acromegaly is medical. **
B) The most common cause of acromegaly is GH-secreting macroadenomas.
C) The most commonly used test to evaluate GH reserve is the insulin tolerance test.
D) Insulin tolerance test is used to evaluate the corticotroph cell reserve in the pituitary.
E) 1 mg dexamethasone or 2 mg dexamethasone for 2 days suppression tests can be used to
screen for Cushing's syndrome.

8) Which of the following is not a risk factor for osteoporosis?


A) Hypogonadism *
B) Hypothyroidism
C) Rheumatoid arthritis
D) Malabsorption
E) Primary hyperparathyroidism

9) Which of the following test results would be expected in a patient with parathyroid adenoma?
A) total ca:10.3 albumin:3 pth:85**
B) total ca:11.2 albumin:5.5 pth:90
C) total ca:8.5 albumin:4 pth:120
D) total ca:7.5 albumin:4.2 pth:16
E) total ca:12.5 albumin4.1 pth:18

10) Fasting glucose is measured from the capillary of a 55-year-old obese patient. It is 137 in the
first test and 140 in the second. What is appropriate to say about this patient?

O
A) There is prediabetes, oggt should be done
B) venous fasting glucose should be measured

O
C) glucose should be checked in the urine
D) Type 2 diabetes is diagnosed
1
E) there is impaired glucose tolerance
11) Which is wrong about thyroiditis?
A-In subacute thyroiditis, symptoms occur following viral infection.
B-Sedimentation and CRP increase in subacute thyroiditis
C-RAI decreases in the hyperthyroidism phase in subacute thioiditis
O
D- RAI increases in the hyperthyroidism phase in silent thyroiditis
E-Postpartum thyroiditis is associated with autoimmunity

12) Which of the following is not expected regarding the clinical findings of hypothyroidism?
A) gastric emptying time increases
B) breathing slows down and becomes superficial
C) pulse pressure increases**
D) cardiac stroke volume decreases
E) diastolic hypertension

13) Which is not true for thyroid cancers?


A) Biological behaviors of papillary and follicular thyroid cancers are different
B) Serum thyroglobulin level is used as a tumor marker in patients with medullary thyroid cancer**
C) Medullary thyroid cancer may secrete calcitonin and CEA
D) External radiation to the neck increases the risk of papillary thyroid cancer
E) Papillary thyroid cancer is one of the differentiated thyroid cancers.

14) Which is incorrect regarding DKA and HHD?


A) While the primary problem in DKA is insulin deficiency, in HHD it is dehydration.
B) In HHD, arterial pH is greater than 7.3.
C) DKA bicarbonate treatment is not started when arterial pH is above 7.0.
O
D) In both cases, insulin treatment can be started when the K level is below 3.5 mEq/L.
E) The anion gap is increased in DKA.

15) Which of the following is incorrect about insulin and oral antidiabetics used in the treatment of
diabetes mellitus?
A) Thiazolidinediones (glitazone) cause fluid retention.
B) Weight gain may occur after insulin treatment.
C) Biguanides increase insulin sensitivity.
D) Sodium glucose co-transporter 2 inhibitors cause genitourinary infection.
E) DPP-4 inhibitors act by inhibiting the alpha glucosidase enzyme in the intestinal brush border.
(reply?)
16) Which of the following is not a sign of acute pancreatitis?
A) Most patients may have leukocytosis between 10000-15000.
B) There may be mild hypoglycemia initially**
C) Initially, hematocrit increases
D) ALT and AST, liver function tests, are generally increased.
E) Prerenal azotemia and increased creatinine levels may occur.

17) Which diet does not affect?


O
A) Breast ca
B) Colon ca
C) Stomach ca

19)In which of the following situations


Is antiviral therapy not indicated?
A) Child A cirrhosis, HBV DNA positive patient

O
B) Patient with HbsAg positive, Anti HBe positive, HBV DNA 100 IU/ml, AST ALT normal, liver
biopsy normal
C) Anti HBc Ig positive patient, planned for rituximab treatment
D) Patient with HbsAg positive, HbeAg positive, AST, ALT 2 times higher, HBV DNA 500000 IU/ml
E) Patient with HbsAg positive, Anti Hbe positive, AST, ALT 1.5 times higher, HBV DNA 30000
IU/ml, 2/6 fibrosis in liver biopsy

20) Which one is not used in secondary prophylaxis in esophageal varicose veins?
A) propranolol
B) Carvedilol
C) Band
D) Sclerotherapy
E) Somatostatin analogues**

22) Which pancreatic cyst neoplasm has a honeycomb appearance?


A) Serous cyst neoplasm***
B) Mucynous cyst neoplasm
C) Main channel IPMN
D) Branched IPMN
E) Solid pseudopolyp neoplasm

25) It was a question about Familial Adenomatous Polyposis. I don't remember the details of the
options, the most common being adenomatous polyp and
There were prevalence, genetic transmission, in which decade the risk increases, etc.
26) Which of the principles of approach to upper gastrointestinal system bleeding is correctly
stated?
A) Early (<12 hours) upper GI endoscopy should be performed on every patient following
hemodynamic resuscitation.
B) Nasogastric or orogastric lavage/aspiration should be performed routinely.
C) Hemodynamic stabilization and hemodynamic monitoring are indispensable for GI bleeding
control.**
D) Second-look endoscopy is a mandatory part of the approach to nonvariceal upper GI bleeding.
E) Helicobacter pylori eradication is required for all patients.

28) For which is central necrosis diagnostic in CT and MRI?


A) hepatic adenoma
B) hemangioma
C) focal nodular hyperplasia**
D) nodular regenerative hyperplasia
E) hepatocellular carcinoma

29) Which of the following is not due to primary biliary cholangitis cholestasis?
A) Biliary stenosis
B) Cholangitis
C) Choledac cyst**
D) Cholelithiasis
E) Cholangiocarcinoma

30) A 29-year-old, 32-week pregnant patient applies to the emergency room with complaints of
fatigue, nausea and vomiting that have been going on for 2 weeks. Chronic
There is no sign of liver disease, pulse is 100/minute, blood pressure is 160/94 mm Hg. There is
tenderness and peripheral edema in the right upper quadrant. Laboratory results: 10 days
hemoglobin 11 g/dL, platelet count 68 × 109/L (150–400), INR 1.7 (upper limit 1.2), schistocytes
present in peripheral smear, bilirubin 3.5 mg/dL, ALT 180 U/L , AST 260 U/L, Serum LDH 720 U/L.
Which of the following is the most likely diagnosis?
A) HELLP syndrome
B) Acute fatty liver of pregnancy**
C) Intrahepatic cholestasis of pregnancy
D) Immune thrombocytopenic purpura
E) Acute viral hepatitis
31) Which is not true for Crohn's disease?
A) Non-caseating granulomas are seen.
O
B) The rectum is definitely involved.
C) Stricture and fistula may be seen.
D) Smoking is a negative prognostic factor.
E) The risk of colon cancer has increased.

33) A 35-year-old female patient comes to the hospital with epigastric pain that has been going on
for 10 days. The patient's vital values are normal. There is jaundice but no itching. On physical
examination, there is tenderness on superficial palpation in the right upper quadrant. As a result of
Abdominal Ultrasonography, bile duct dilatation and heteroexogenic bile duct stones are
visualized. The patient's laboratory values are ALP: 872U/L, GGT:130U/L, AST:92U/L, ALT:145U/L:
Total bilirubin:8.7mg/dL, Indirect Bilirubin:4.3mg/dL. What is the next action needed for this
patient?
D) Endoscopic Ultrasonography
E) Endoscopic Retrograde
Cholangiopancreatography**

34) Which of the following statements about liver imaging, biochemical and function tests is NOT
true?
A) Abdominal Ultrasonography (USG) is the most appropriate imaging method for evaluation of
the liver.
B) Decreased serum albumin, prolonged prothrombin time (PT) indicate chronic liver parenchymal
disease (Cirrhosis).
C) Once elevated serum aminotransferase (ALT/AST) is detected, further evaluations should be
made.**
D) Careful history taking, physical examination and routine laboratory tests enable a definitive
diagnosis to be made in 85% of patients with jaundice.
E) Serum alkaline phosphatase (ALP) and gammaglutamyl transpeptidase (GGT) are
disproportionately increased compared to ALT and AST in obstructive jaundice

39) What is the most likely diagnosis of the patient whose predominant symptoms are
hallucinations, parkinsonism, and neuroleptic sensitivity whose symptoms fluctuate?
A) alzheimer's disease
B) frontotemporal dementia
O
C) dementia with lewy bodies
D) huntington dementia
E) vascular dementia
41) Which of the following is among the typical side effects of anticholinergic drugs used in the
treatment of urinary incontinence?
A) Hyperkalemia
B) Headache
C) Rash
D) Diarrhea
E) Confusion**

42) Which of the following is the main part of the confusion assessment method?
Isn't it one of its components? Please choose one:
A) Change in state of consciousness
b) Disorganized thought
C) Lack of attention
D) Acute change in mental status and the fluctuation of this change
E) Apathy*

43) Which of the following is the wrong option regarding pressure sores?
A) Friction is one of the main mechanical factors in the formation of pressure sores.
B) The first step in prevention is to identify the patient at risk of deterioration of skin integrity.
C) It is recommended to change position at least every two hours to prevent its formation.
D) The average calorie need of patients with pressure sores is 30-35 kcal/kg.
E) Protein intake of patients with pressure sores should not exceed 1.0 g/kg/day. **

44) An 83-year-old male patient with mild dementia and stage 3 chronic kidney disease is
hospitalized due to pyelonephritis. He stated that he was surprised and confused in the middle of
a conversation two days after his admission.
is being done. His family says he was sleepy when they visited and now doesn't recognize his son.
Which of the following would you do for this patient who tries to climb into bed several times at
night and removes the intravenous line?
A) Keeping the television on to distract him.
B) Having a caregiver with you. **
C) Tie his arms and restrict his movement.
D) Give quetiapine and calm down.
E) Observe
45) Which is incorrect about Hodking lymphoma?
A) CD30 is found in all subtypes**
B) It mostly originates from the supradiaphragmatic lymph node
C) Ebv is thought to be its etiology.
D) The more common type in women is nodular sclerezonation.
E) Histopathologically, Reed strenberg cells are used in diagnosis and prognosis (?).

46) Which of the following is incorrect information about stem cell transplantation?
a) Mobilization is to ensure cell migration from the bone marrow to the peripheral blood.
c) Peripheral stem cell collection is performed under general anesthesia**
d) Stem cells express CD34 protein on their surface.
e) The most important criterion in choosing a donor is HLA compatibility.

48) A 22-year-old female patient presents to the emergency department with paleness, petechiae
in her legs, and splenomegaly. In the examination, his hemogram showed hemoglobin 8, 110x10⁹
white blood cells, and 12x10⁹ platelets. In his peripheral smear, there are immature cells with
narrow cytoplasm, visible nuclei, dense chromatin and immature cells. Cytochemical examination
revealed MPO negative and Cd10-19-22 positive. What is the diagnosis?
A)AML M3
B)AML M4
C) BALL**
D) TALL
E) CLL

49) A 45-year-old male patient is admitted to the hospital due to weakness and shortness of
breath. On physical examination, the patient appears pale and petechial rashes are detected on his
legs. Gingival hypertrophy is detected. Hb:7, white blood cell:25.10^9, platelet:24.10^9.Which is the
first test to be requested for diagnosis?
A) Abdominal USG
B)Peripheral smear**
C)Full urinalysis
D)Thorax CT angiography
E)D-dimer

50) Which of the following is not true for Polycythemia Vera?


A) It is a clonal cell disease.
B) Erythropoietin level is low.
C) The most common side effect is thromboembolic complications.
O
D) Cytoreductive treatment should be applied to all patients.
E) 95-100 percent of the patient has JAK-2 mutation
51) Which of the following is true about Warfarin?
A) It may cause thrombocytopenia.
B) It is used only parenterally.
C) It may cause skin necrosis. *
D) It is a cofactor of antithrombin
E) Etiology is followed by aPTZ.

53) Which reaction can be prevented by reducing leukocyte count?


A) Acute hemolytic reaction
B) Alloimmunization
C) Hemosiderosis
D) Allergic reaction
E) Graft versus host disease**

54) Which of the following is the most common acute complication of hemodialysis?
A) nausea-vomiting
B) headache

O
C) hypotension

55) Which of the following is not true about Von Willebrand disease?
A) It is the most common hereditary bleeding diathesis.
B) Both primary and secondary hemostasis are affected.
C) Factor 5 level is low.
D) Bleeding time is prolonged.
O
E) Platelet aggregation does not occur with ristosetib.

56) Which of the following is not an indication for parenteral iron therapy?
A) Pregnant woman in the 3rd trimester
B) those who cannot tolerate oral iron therapy
O
C) people with hematochezia
D) those with heart failure
E) those with malabsorption
57) LDH level increased significantly in the patient with pancytopenia. Which condition should be
considered first in this patient?
A) Aplastic Anemia
B) Pure red cell aplasia (Pure erythroid aplasia)
C) Iron deficiency anemia
D) Primary Myelofibrosis

O
E) Megaloblastic Anemia

58) Which of the following should be done in a patient whose aPTT=120 seconds during routine
screening?
A) 5ml/kg FFP transfusion

O
B) 10ml/kg FFP transfusion
C) Transfusion of 10 units of cryoprecipitate
D) 10 mg vitamin K infusion
E) There is no need to do anything for this patient.

59) There is no evidence of bleeding in the patient whose platelets are 20x10^9 L. If this patient
has which underlying disease, would you transfuse platelet suspension?
A) immune thrombocytopenia
O
B) MDS
C) heparin-associated type 2 thrombocytopenia
D) posttransfusion purpura
E) HELP syndrome

60) Which of the following is not an expected finding for multiple myeloma?
A) Increased sedimentation
O
B) Total protein increase
C) Roll formation
D) Increase in alkaline phosphatase
E) Creatine increase

61) Which can be diagnosed most easily by hemogram and peripheral smear?
A) ALL-L2
B) AML M0-M1
C)MDS
D) Aplastic anemia

O
E) CLL
62) Which of the following is not true about renal findings of systemic diseases?
A) Hypertensive nephropathy is the second most common cause of end-stage renal failure.
D) The definition of TTP was given - it was said that there were kidney symptoms. (I think this was
the answer, there were no kidney symptoms) **

63) What is the first choice antihypertensive for asymptomatic hypertension in chronic kidney
disease patients under close follow-up?
A) Ca channel blocker
B)ACE inhibitor ARB*
C) Beta blocker
D) Alpha blocker
E) Potassium-sparing diuretic

65) A fifty-four-year-old male patient applies to the emergency department with complaints of
nausea and vomiting. In the physical examination of the patient, the pulse is 44/minute and
arrhythmic, and the blood pressure is 90/60 mmHg. Laboratory examinations revealed urea 240
mg/dI, creatinine 12 mg/dI, sodium 128 mEq/L and potassium 7.2 mEq/L. To prevent cardiac
arrest in this patient
Which of the following should be applied first for this purpose?
A) Emergency hemodialysis
O
B) intravenous calcium gluconate
C) insulin + dextrose
D) Oral potassium retaining resin
E) Sodium bicarbonate

67) A 27-year-old woman is 30 weeks pregnant and applies for routine follow-up. Blood pressure is
150/100 mmHg. It is learned that he was previously normotensive. Urine analysis shows density
1020 SG and +1 proteinuria. Serum uric acid level is 8. Platelets and liver function tests are
normal. There is 1.1 grams of protein in a 24-hour urine analysis. Antihypertensive treatment is
planned for the patient. What is a first-line agent?
A) Captopril
B) Furosemide
C) MgSO4 (IV)
O
D) Methyldopa
E) Hydrochlorothiazide

68) A 64-year-old male patient comes to the hospital with swelling in his hands, face and legs.
Blood pressure is normal, hg: 9, ht: 22%, sAlb: 2, kre: 0.6, urea nitrogen: 24, cholesterol: 280. Kidney
bx: IgG, complement accumulated in the subepithelial region, diagnosis?
A) IgA nephropathy

O
B) Membraneous nephropathy
69) Which of the following is not prognostic in terms of diagnosis?
A) high blood pressure
B) hyperuricemia
C) proteinuria
D) There was something about interstitial nephritis in the kidney biopsy, I can't remember exactly.
E) Serum Iga

70) 20-year-old female patient. The patient has edema and xanthelasma. Steroid use feels good.
Pathology?
A) crescent
B) deletion of podocyte foot processes**

71) Which of the general approach principles to upper gastrointestinal system bleeding is correctly
stated?
A) Following hemodynamic resuscitation, early (<12 hours) upper GI endoscopy should be
performed in every patient.
B) Nasogastric or orogastric aspiration/lavage should be performed routinely.

O
C) Hemodynamic stabilization and hemodynamic monitoring are indispensable for gastrointestinal
bleeding control.
D) Second-look endoscopy is a mandatory part of the approach to nonvariceal upper GI bleeding.
E) Helicobacter pylori eradication is necessary in all patients.

72) What is the main mechanism responsible for the pathogenesis of preeclampsia?
A) Uteroplacental insufficiency**
B) Proteinuria
C) Decreased angiotensin II receptor activity
D) Increase in antiangiogenic factors in circulation
E) Decrease in glomerular filtration rate

74) Which of the following is not a feature of autosomal dominant polycystic kidney disease?
A) Female/Male ratio is 1.
O
B) The most common extrarenal finding is intracranial aneurysms.
C) Among concomitant heart valve diseases, MVP is the most common.
D) In individuals over 60 years of age, having 4 or more cysts in each kidney is diagnostic.
E) PKD1 and PKD2 are identified genetic disorders.
75) Which of the following is not an indication for biopsy in acute renal failure?
A) If acute renal failure is unclear
B) If acute renal failure shows systemic findings

O
C) Uncontrolled diabetic patient with 200 albuminuria in 24 hours urine
D) Development of acute renal failure after kidney transplantation
E) The patient who was diagnosed with ATN and did not improve even after 6 weeks

76) A 61-year-old male patient has been complaining of cough, hemoptasis and chest pain for 3
weeks. ECOG 1 and lung auscultation show no pathology findings other than respiratory rhonchi
on the left. Left lung hilar There is an 8x8 mass in the area, an 8x10 necrotic mass in the upper
zone extending to the upper lobe, and left hilar and mediastinal conglomerate and metastatic
lesions. A metastatic focus is detected in the 3rd vertebra. Complete blood count, biochemistry,
biomarker There are no pathological findings. How should this patient be approached?
A) Systemic chemotherapy
B) Targeted drug
C) Immunotherapy
D) Pet

O
E) Biopsy for histopathological diagnosis from the most appropriate location**

77)Which of the following parameters is mandatory in patients who are planned to have systemic
chemotherapy containing cisplatin as antineoplastic treatment?
a. Leukocytes above 10,000/mm3
b. Platelets above 250,000/mm3
c. Serum transaminases are normal
D. Blood hemoglobin level above 13 grams/dL

O
to. Creatinine clearance higher than 50 mL/minute

81) Which of the following is not one of the factors associated with carcinogenesis and cancer
defined by Hanahan et al?
A) Sustained proliferative signaling capacity
B) Disruption of regulation of cell metabolism
C) Development of resistance to APAOTOAZA

O
D) geom stabilization and mutation capacity
E) tumor-associated inflammation
82) Which of the following is incorrect?
A) Giant cell arthritis involves small vasculitis
B) Behcet's can involve vasculitis of any size.

83) Which of the following cancer screenings is incorrect?


A) Colon cancer - colonoscopy
B) rectal cancer - colonoscopy
C) breast cancer- mammography
D) lung cancer - low dose Bt
E) ovarian cancer - pelvic USG + CEA 125**

84) In which disease is diet not recommended for treatment?


A) Column Ca
B) Stomach Ca
C) Pancreatic Ca
D) Malignant Melanoma*
E) Breast Ca

85) A 55-year-old man presents with fever and arthralgia symptoms. Hematuria, erythrocyte slips,
and protein (+++) are detected in the urine. There is a segmental creset appearance. No IG or
Complement deposition. Creatine in 3 days
It increased from 1.5 to 2.9. Which of the following would be useful to look for in this patient?
A) Anti-GBM
B) Cryoglobulin
C) ANCA**
D) MAIN
E) C3-C4
86) A 65-year-old woman with rheumatoid arthritis presents with polyarthritis involving her hands,
wrists, and feet. He is currently on triple therapy with methotrexate 20mg Sc once a week,
hydroxychloroquine 400mg daily and sulfasalazine 1g bid, and prednisone 20mg daily. History of
moderate to severe COPD There is a history of bowel perforation and 2 shingles, one involving the
eye, in the background of diverticulitis. On physical examination, he has eight tender and swollen
joints. What is the next best treatment option?
A) Rituximab
B) TNF alpha blocker

O
C) Baricitinib
D) Tocalizumab
E) Tofacitinib

88) Which of the following is not seen in the polymyositis clinic?


A) Interstitial lung disease
B) Neuropathic findings
C) It is proximal and symmetrical. Neck muscles, trunk muscles and respiratory muscles may be
involved.
D) myocarditis
E) Dysphagia

89) Which of the following is not a symptom of adult still disease?


A) Mobile, slightly sensitive lymphadenopathy, often in the cervical region
B) Fever
C) Splenomegaly
D) Salmon-colored rashes that appear on the trunk and proximal upper extremities, usually with
fever.

O
E) Leukopenia

91) Medication that requires routine eye examination due to retinal deposition?
A) HQ(Antimalarial)*
B) 5-ASA
C) Azathioprine
D) Anti-TNF
E) MTX
93) What is the next treatment option for axial spondyloarthritis unresponsive to NSAID treatment?
A) Rituximab
B) Tocilizumab
C) Azathioprine
D) Colchicine

O
E) Anti-TNF agent

95) Anion gap is one of the causes of normal metabolic acidosis?


A) Iron intoxication
B) Diabetic ketoacidosis
C) Renal tubular acidosis**
D) Lactic acidosis
E) Salicylate intoxication

96) Which one is not seen in the course of Sjögrem syndrome?


A) Dry eye
B) Sclerodactyly*
C) Parotid gland enlargement
D) Lymphoproliferative malignancy
E) Peripheral neuropathy
99) Which of the following genes is associated with PeutzJeghers syndrome?
A) APC
B) STK11 **
C) BRCA
D) Rb
E) VHL
100) Which of the following turns from proto-oncogene to oncogene by amplification?
A) BCR
B) ABL
C) RB
D) TP5
T2
1) A 23-year-old male patient applies to the clinic due to masculine dysfunction. There is increased
facial hair growth and gynecomastia. Additionally, during palpation, the testicles are palpated as
small and hard. What do you think?
A) kallman syndrome
B)klinifelter syndrome **
C) androgenic deficiency
D) turner syndrome
E) germinoma

2) A 41-year-old woman is referred to the endocrine clinic due to a gradual onset of headache and
high blood pressure. There is no significant disease history. The examination blood pressure is
measured as 175/110 mmHg. Laboratory and radiology findings: Na: 144 mmol/L, (135-145), K:
4.4 mmol/L (3.5-5.5), Urea: 7.0 mg/dL (5-9), Aldosterone 18ng/dL, Plasma Renin Activity (PRA) 0.6
ng/ml/h. Abdomen CT: 1.7 cm mass in the right adrenal gland. Which of the following is the most
appropriate next step for this patient?

O
A) Verification Test
B) Genetic Testing
C) Ultrasound guided biopsy
D) Surgery
E) Starting a beta blocker

3) In which of the following causes of thyrotoxicosis is RAI uptake increased?


A)Subacute thyroiditis
B) Graves' Disease*
C)Type 2 Amiodarone Thyrotoxicosis
D)silent thyroiditis
E)struma ovarii
4) Which of the following is an opioid receptor antagonist and dopamine and noradrenaline
reuptake inhibitor used in the treatment of obesity?

O
A) Orlistat
B)naltrexone/bupropion
C)Locaserin
D) Liraglutide
E)Phentermine/topiromat
5) A 45-year-old female patient applies to the endocrine clinic with complaints of enlargement of
the nose, hands and feet, intense sweating, thickening of the skin, coarsening, snoring and joint
pain. Which cannot be said?
A) Glucose metabolism may also be impaired
B) There is a GH-secreting adenoma

O
C) Insulin tolerance test should be performed for diagnosis
D) Contrast-enhanced pituitary MRI should be performed
E) Other anterior pituitary hormones should also be checked.

6) Which of the following findings particularly suggests "Dipsogenic Diabetes Insipidus" in a


patient who applies to the outpatient clinic with complaints of drinking too much water and
urinating nearly 5 liters per day?
A) History of head trauma
B) Decrease in the amount of urine while the patient is asleep**
C) Low plasma ADH
D) The patient uses lithium
E) Urine osmolality increases when ADH is given externally

7) Which of the following statements is incorrect for hypofunctioning conditions of the anterior
pituitary gland?
A) Corticotroph cells are vital
B) There is no need for treatment in prolactin deficiency
C) Growth hormone deficiency in adult patients may lead to dyslipidemia and central obesity.
D) In cases of gonodotropic hormone deficiency, fertility is mostly achieved by assisted
reproductive methods.
E) Cutting the pituitary stalk causes a decrease in all anterior pituitary hormones**

8) In which of the two situations represented by areas A and C, where serum calcium and
parathormone values coincide, are given correctly? (There was a table in the question, area A
showed high pth and low serum calcium, and area c showed high pth and high calcium.)
A)A familial hypercalciuric hypercalcemia c secondary hyperparathyroidism
B) a primary hyperparathyroidism c d vitamin intoxication
.
o
C) Disc hyperparathyroidism for A, primary hyperparathyroidism for c.
D) a d vit deficiency c hypercalcemia due to malignancy
E) a tertiary hyperparathyroidism c hypercalcemia due to sarcoidosis
9-) Which of the following is not a risk factor for Gestational Diabetes Mellitus?
A-)Obesity
B-) Polycystic Ovary syndrome
O
C-) <40 years old**
D-) Glycosuria
E-) Antipsychotic drug use

10) Which of the following is a sign or finding of chronic hypocalcemia?


A. Tetany

O
B. Extrapyramidal findings
C. Epilepsy
D. Laryngospasm
E. Hypotension

11) Which of the following is incorrect regarding the acute complications of diabetes?
A) In diabetic ketoacidosis, the main cause is insulin deficiency, and in hyperosmolar
hyperglycemic state, it is dehydration.
B) Anion gap is increased in diabetic ketosidosis.
C) Ketonuria is not expected in hyperosmolar hyperglycemic state.
D) In diabetic ketoacidosis, the total fluid deficit is greater than in the hyperosmolar hyperglycemic
state.*****
E) Brain edema is a complication seen after the treatment of diabetic ketoacidosis and
hyperosmolar hyperglycemic state.

12) Which of the following is incorrect?


A) In secondary hypothyroidism, the dysfunction is in the pituitary.
B) Lithium can cause hypothyroidism
C) Hypothyroidism can cause bone fractures
D) Primary hypothyroidism is monitored with serum free t4**
E) Hypothyroidism can cause galactorrhea
14) What is the oral antidiabetic that is not used in congestive heart failure that causes fluid
retention and edema?
A) Sulfonylurea
B) Metformin
C) thiazolidinediones**
D) Alpha glucosidase inhibitors
E) DPP-4 inhibitors

15) Which of the following is not an indication for treatment with insulin?
A) Type 1 DM
B) Hyperglycemic Emergencies
C) Severe Liver Failure
D) Gestational Diabetes that cannot be controlled with diet
E) Excessive weight gain**

16) Which of the following is the most common cause of Acute Pancreatitis?
A) common bile duct stones**
B) high triglyceride
C) alcohol
D) trauma
E) hypercalcemia

17) Which test is the most sensitive in the early stage of chronic pancreatitis?
A) Fecal elastase
B) Serum trypsin test
C) C-13 labeled triglyceride breathing test
D) Secretin/cerulein test **
E) Fecal chymotrypsin measurement
18) Which of the following is not seen in Behçet's Disease with gastrointestinal system
involvement?
A) Usually involves the ileocecal region
B) Colonoscopy reveals deep intestinal ulcers
C) Bleeding and perforation may occur due to ulcer
D) Intestinal stenosis and perianal disease are frequently observed**
E) It may also involve the esophagus or small intestines

19) Which of the following is the most common complication of cirrhosis?


A) acid******
B)esophageal bleeding
C)hepatorenal syndrome
D)hepatopulmonary syndrome
E) I don't remember this option

21) Which of the following is not used in chronic HCV infection?


A. sofosbuvir
B.ledipasvir
C.pibrentasvir
D.entecavir**
E.glecaprevir

22) Which of the following is suggested in irritable bowel syndrome?


Isn't it one of those situations?
A. Food sensitivity
B. Abnormal 5 hydroxy tryptamine metabolism
C. Mucosal inflammation
D. Motility change
E. Visceral hyposensitivity**
23) Which of the following is not one of the drugs that reduce lower esophageal sphincter (LES)
pressure?
a. beta blocker
b. theophylline
c. calcium channel blocker
D. Metoclopramide*
to. Diazepam

24) Which of the following is the most common benign non-epithelial tumor of the esophagus?
A) Leimyom***
B) Lipoma
C) Hamartoma
D) Hemangioma
E) Fibrovascular Polyp

25) The patient, who is being followed up with the diagnosis of extensive ulcerative colitis, applies
to the emergency clinic with the complaint of fever and abdominal pain while receiving
azathioprine and mesalazine treatment. Physical examination reveals hypotension, tachycardia,
abdominal tenderness, distension, and decreased bowel sounds. Rebound is not detected. What is
your most likely diagnosis for this patient? What would be your approach?
a. The likely diagnosis is colon perforation. Colonoscopy should be performed to confirm the
diagnosis.
b. The possible diagnosis is Toxic megacolon. To confirm this diagnosis, it would be good to have
the patient take an urgent CT scan or a direct standing abdominal radiograph to see that the
transverse colon diameter is more than 6cm.**
c. The possible diagnosis is Toxic megacolon. There is no need for additional examination for
diagnosis. Physical examination is sufficient. It is necessary to undergo urgent surgery.
D. The possible diagnosis is Toxic megacolon. To confirm the diagnosis, the patient should have
an urgent colonoscopy.
to. The possible diagnosis is Toxic megacolon. After the diagnosis is confirmed by USG, diarrhea
should be reduced with antidiarrheal drugs.
26-Choose the correct option according to the information given below.
1-Peutz-Jeghers is a hamartomatous polyp and shows transition to OD.
2-Turcot syndrome is the subtype of FAP associated with CNS primary tumors (medullablastoma,
etc.).
3-Attenuated FAP occurs earlier in age than classical FAP.
4-Since the age at which polyps appear in classic FAP is 10-12, annual sigmoidoscopy should be
performed.
A) 1,2 and 4 are correct.**
B) All are correct.
C) 1 and 3 are correct.
D) 1 and 2 are correct.
E) 2 and 4 are correct.

27) Which of the principles of approach to upper gastrointestinal system bleeding is correctly
stated?
A) Early (<12 hours) upper GI endoscopy following hemodynamic resuscitation should be
performed on every patient.
B) Nasogastric or orogastric lavage/aspiration should be performed routinely.
C) Hemodynamic stabilization and hemodynamic monitoring are indispensable for GI bleeding
control.**
D) Second-look endoscopy is a mandatory part of the approach to nonvariceal upper GI bleeding.
E) Helicobacter pylori eradication is required for all patients.

28) Which of the following is incorrect about hepatic adenomas?


A) It is sensitive to hormones
B) It grows during pregnancy.
C) There is the potential for hepatocellular carcinoma development
D) There is a risk of spontaneous bleeding
E) It does not require surgical removal**
29) Which of the following is the most common symptom seen in carcinoid syndrome?
A) secretory diarrhea**
B) flushing on the skin
C) wheezing
D) asthma
E) headache

30) Which of the following is not true about liver diseases seen during pregnancy?
A. The most common cause of jaundice during pregnancy is viral hepatitis.
B. Intrahepatic cholestasis of pregnancy presents with itching in the palms of the hands and soles
of the feet, which increases at night.
C. Family history of acute fatty liver of pregnancy is usually positive.**
D.The treatment for intrahepatic cholestasis of pregnancy is urcedeoxycholic acid.
E. The prognosis for hyperemesis gravidarum is generally good for both mother and baby.

31. A 30-year-old male patient presents with abdominal pain and bloodless diarrhea for the last 3
months. Which is incorrect regarding the patient whose colonoscopy revealed deep ulcers in the
terminal ileum?
a) Inflammation is limited to the mucosal layer.**
b)Western diet increases the risk of contracting the disease.
c) Intestinal tuberculosis should be considered in the differential diagnosis.
d) The presence of perianal disease is generally considered a negative prognostic factor.
e) Ustekinumab, which acts by blocking the common p40 ligand of IL12 and IL23, can be used in
treatment.

32.Which of the following treatment approaches has no place in nonalcoholic fatty liver disease?
A) Calorie restriction-regular exercise
B) Recommendation of coffee consumption
C) Pioglitazones**
D)GLP-1 analogues
E)Metformin
33) Which of the following information about functional (non-ulcer) dyspepsia, peptic ulcer and
alarm markers is not correct?
A) Evaluation of the patient with abdominal pain and dyspepsia should begin with history taking
and physical examination.
B) The main risk factors of peptic ulcer are H.pylori infection and NSAID (analgesic) use.
C) Dysphagia, vomiting, iron deficiency anemia are alarm indicators.
D) Diagnostic criteria for dyspepsia due to peptic ulcer include postprandial bloating, early satiety,
epigastric pain and epigastric burning.**
E) Upper gastrointestinal endoscopy should be performed on a 75-year-old male patient with
epigastric pain, bloating and burning for 2 months.

34)Which of the following is incorrect about the liver?


A) In case of mild elevation, serum aminotransferase level measurement should be repeated.
B)ALP, AST, ALT do not measure liver functions.
C)Hemachromatosis is an autosomal recessive disease with an ATP7B gene defect on
chromosome 13.**
D) Balooning hepatocytes and Mallory equivalent bodies are seen in NAFLD biopsy.
E) In Wilson treatment, penicillamine, trientine and zinc are used for maintenance.

35) A 35-year-old female patient comes to the hospital with epigastric pain that has been going on
for 10 days. The patient's vital values are normal. There is jaundice but no itching. On physical
examination, there is tenderness on superficial palpation in the right upper quadrant. As a result of
Abdominal Ultrasonography, bile duct dilatation and heteroexogenic bile duct stones are
visualized. The patient's laboratory values are ALP: 872U/L, GGT:130U/L, AST:92U/L, ALT:145U/L:
Total bilirubin:8.7mg/dL, Indirect Bilirubin:4.3mg/dL. What is the next action needed for this
patient?
A) Hepatitis tests
B) Abdominal ultrasound
C) CT
D) Endoscopic Ultrasonography**
E) Endoscopic Retrograde Cholangiopancreatography
37) A 69-year-old female patient is diagnosed with hypertension at the outpatient clinic. Which one
is not given first?
A) Ace inhibitor
B)Arb
C)B blocker **
D)Loopdiuretic
E) Ca canal blocker

38) Which is not a side effect of anticholinergics?


A. Visual impairment
B.Hypersalivation
C.Urinary retention**
D. Constipation
E. Somnolence, lethargy, cognitive impairment

39) What is the most likely diagnosis of dementia in an elderly patient presenting with personality
change as the dominant symptom?
A) Vascular dementia
B) Alzheimer's disease
C) Parkinson's dementia
D) Frontotemporal dementia**
E) Dementia with Lewy bodies

40) What is the stage of the pressure ulcer when the ulcer with full thickness tissue loss covers
the base wound bed with crust (colors) and/or eschar (tan, brown, black)?
A) Stage 1
B) Stage 2
C) Stage 3
D) Stage 4
E) Unstageable**
41) Which of the following is one of the changes we expect during the normal aging process?
a. Disease symptoms such as pain or fever are less obvious**
b. Decrease in body fat percentage
c. Increased vital capacity
D. Decreased insulin resistance
to. Increased frequency of autoimmune diseases

43) Which of the following drugs is one of the drug groups that are frequently prescribed
inappropriately or unnecessarily in the elderly?
A) Oral Antidiabetic
B) Anticholinergic Agents**
C) Statins
D) Antihypertensives
E) Iron Supplements

44) (An elderly person had a story of fainting.) Which of the following is incorrect about falls in the
elderly?
A) the cause of falls is multifactorial
B) I couldn't remember :(
C) extrinsic factors are among the most modifiable factors**
D) The most common causes of falls include cognitive impairment, a history of falling in the past,
and advanced age.
E) A decrease in the number of falls is detected with vitamin D supplementation.

45) Which is incorrect about Hodgkin lymphoma?


A. CD30 is found in all subtypes.**
B. Mostly supradiaphragmatic lymph
It originates from the node.
C. EBV is thought to be the etiology.
D. The more common type in women is nodular sclerosing.
E. Histopathologically, Reed Sternberg cells are used in diagnosis and prognosis.
46) Which of the following is incorrect information about stem cell transplantation?
a) Mobilization is to ensure cell migration from the bone marrow to the peripheral blood.
c) Peripheral stem cell collection is performed under general anesthesia**
d) Stem cells express CD34 protein on their surface.
e) The most important criterion in choosing a donor is HLA compatibility.

47-) A 22-year-old female patient presents to the emergency department with paleness, petechiae
in her legs, and splenomegaly. In the examination, his hemogram showed hemoglobin 8, 110x109
white blood cells, and 12x109 platelets. In his peripheral smear, there are immature cells with
narrow cytoplasm, visible nuclei, dense chroma, and immature cells. Cytochemical examination
reveals mpo nega3f Cd10-19-22 positive. What is the diagnosis?
A)AML M3
B)AML M4
C) BALL**
D) TALL
E) CLL

48) Which is wrong about apharesis?


A) adsorption is an apharesis technique in which bioactive membrane is used.
B) Apharesis is ordered from the inside to the outside, starting from the erythrocytes.**
C) Hypocalcemia may be observed due to citrate toxicity.
D) Gcsf is used in stem cell mobilization.
E)granulocyte aphasia is used in the presence of severe infection.

49) A 45-year-old patient comes with weakness and shortness of breath. During the examination,
pale-looking petechial rashes are detected. Gum hypertrophy is detected. Hemoglobin 7 white
blood cell 250 .10'9 platelets 24 10'9 What is the first test we will request for diagnosis in this
patient?
A) ultrasonography
B)peripheral smear**
C) full blood examination
D)thorax CT angiography
E)d-dimer
50) Which one is not one of the hematological remission criteria in CML?
A) spleen not palpable
B)precursor cell not visible
C) leukocyte count <10,000
D) platelet count <450,000
E) LDH level being normal**

51. Which of the following is not true about Warfarin (Coumadin)?


A) It is a vitamin K antagonist.
B) It is started together with heparin.
C) Can be used parenterally only.***
D) There may be hereditary resistance to Warfarin.
E) Its effectiveness is monitored by INR

52)Which of the following is incorrect for Factor V leiden mutation?


A) is a risk factor for hereditary thrombophilia
B) is the most common hereditary thrombophilia risk factor
C) causes protein C resistance
D)causes arterial thrombosis**
E)heterozygous factor V Leiden mutation is a weak risk factor for thrombosis

53) Which of the following is not true about Von Willebrand disease?
A) It is the most common hereditary bleeding diathesis.
B) Both primary and secondary hemostasis are affected.
C) Factor 5 level is low.**
D) Bleeding time is prolonged.
E) Platelet aggregation does not occur with ristosetib.
54) Which of the following is incorrect for ITP?
A) PT time is normal
B) aPTT time is normal
C) D-Dimer level increased
D) Biochemical analyzes are generally normal
E) Bone marrow examination is required for diagnosis **

55) Which of the following is a complication of immune transfusion?


A) Fe loading
B)Bacterial contamination

O
C)Febrile non-hemolytic transfusion reaction
D) Transfusion transmitted infections
E)Volume loading

56) In which unit do clinics request blood components?


A) Blood center
B) Transfusion center**
C) Regional blood center
D) Blood donor center
E) Hemovigilance

57) Chemotherapy was started for a patient with leukemia and thrombocytopenia developed.
Below which value should we give this patient platelet suspension for prophylaxis?
A) 50 000
B) 30 000
C) 20 000
D) 10 000**
E) 5 000
58) LDH level increased significantly in the patient with pancytopenia. Which condition should be
considered first in this patient?
B) Autoimmune hemolytic anemia
C) Iron deficiency anemia

O
D) Primary Myelofibrosis
E) Pernicious anemia*

59) Which of the following is not considered in a 60-year-old patient with pancytopenia and
splenomegaly?
A) Hairy cell leukemia
B) Aplastic anemia**
C) Acute Leukemia
D) ITP
E) Cirrhosis

60) In which stage of CLL is drug treatment not given?


A) Stage IV
B) Stage III
C) Binnet C staging
D) Leukocyte count increases from 45000 to 90000 in 3 months
E) Hemolytic Anemia**

61) Highly effective medications have been given for multiple myeloma. Which is not correct?
A) Ibrutinib**
B) Daratumumab
C) Lenalidomide
D) Bertezomib
E) Dexamethasone
62) Which of the following is incorrect?
a) Hypertensive nephropathy is the 2nd most common cause of end-stage renal failure.
b) Benign nephrolithiasis rarely results in renal failure.
c) Giant cell arthritis is small vessel vasculitis.**
d) In TTP; Maha is accompanied by thrombocytopenia, fever, neurological symptoms and renal
failure.
e) Wegener's ANCA is positive.

63) Which of the following is the MOST common acute complication in a patient on a chronic
hemodialysis program?
a) Muscle cramps
b) Waist and face pain
c) Nausea-vomiting
d) Hypotension**
e) Chief agrist

64) Which of the following is not one of the findings of hypokalemia?


A. Arrhythmias
B. Worsening hypertension
C. Rhabdomyolysis

O
D. Insulin tolerance
E. Metabolic alkalosis

65) Which of the following is not true about erythropoietin?


A) Erythropoitine increases blood pressure
B) It is a treatment option for CKD-related anemia.
C) Does not increase the risk of thrombosis*
D) It extends the lifespan of red blood cells
E) It is synthesized in the renal cortex
68) A patient notices blood in his urine within a few days of an upper respiratory tract infection or
gastroenteritis. Considering the possible diagnosis, what is the likely pathological finding?
a. Marked IgA accumulation with mesangial proliferation**
b. Storage of immunoglobulins and complement across the basement membrane
c. loss of podocytes
D. Complement storage
E. Focal segmental glomerulosclerosis

70) Which of the following is the method of removing sodium by hemodialysis?


What is the mechanism?
A) Convection and diffusion
B) Diffusion**
C) Osmosis
D) Convection
E) Ultrafiltration

71) Which of the following provides intra-extracellular cation concentration in dialysis?


A) Extracellular osmotic pressure
B) free circulation of urea
C) Sodium potassium pump
·
D) Intracellular protein
E) Bicarbonate permeability of the cell membrane

72) Which is incorrect about fluid and electrolyte metabolism?


A) 50% of body weight in women and 60% in men is liquid.
B) Extracellular fluid: interstitial fluid and plasma fluid
C) Plasma osmalite is 275-290
D) The main anion of the extracellular fluid is phosphorus esters**
E) The main cation of the extracellular fluid is sodium
74) Which is incorrect for Autosomal Dominant Polycystic Kidney Disease (ADPKBD)?
A) Decreased renal blood flow
B) Nephrolithiasis

O
C) Hypotension
D) Glomerular sclerosis
E) Apoptosis in tubular epithelial cells

75) Contrast-enhanced MRI aniography will be performed on a type 2 diabetes mellitus patient.
Serum kidney functions are normal. Which is the most effective to prevent the development of
contrast nephropathy?
A.iv. Ringer's lactate
Biv. Sodium bicarbonate
Civ. Adequate hydration with 0.9% NaCl**
D.iv. Ringer's lactate + iv. Sodium bicarbonate
E.iv. Dextrose + adequate hydration

77) A 61-year-old male patient has been complaining of cough, hemoptysis and chest pain for 3
weeks. ECOG 1 and lung auscultation show no pathology findings other than respiratory rhonchi
on the left. Left lung hilarThere is an 8x8 mass in the area, an 8x10 necrotic mass in the upper
zone extending to the upper lobe, and left hilar and mediastinal conglomerate and metastasis
lesions. A metastatic focus is detected in the 3rd vertebra. There are no pathological findings in
complete blood count, biochemistry, and biomarkers. How should this patient be approached?
A)Systemic chemotherapy
B)Targeted drug
C)Immunotherapy
D) Pet
E) biopsy for diagnosis from the most appropriate histopathological site**

78) Which of the following is a B cell-directed anti-CD20 inhibitor and is used in the treatment of
both RA and SLE?
A) Tocilizumab
B) Rituximab**
C) Secukinumab
D) Tofacitinib
E) Certolizumab
79) Which of the following hormonal-targeted agents is not used alone for premenopausal breast
cancer?
A)Fulvestran
B)tamoxifen
C)toremifene
D) Letrozole**
E)megestrol acetate

80) Which of the following is not true for oncological emergencies?


A) The most common causes of VCS are lymphoma and lung cancer.
B) Facial edema, pleatorrhea, and shortness of breath are symptoms of VCSS.
C) Treatment of hypercalcemia is primarily loop diuretics.*****
D) Headache, nausea and vomiting are symptoms of CIBAS.
E) TLS causes hypocalcemia hyperkalemia hyperphosphatemia hyperkalemia.

81) Which of the following is not true regarding neutropenic fever treatment approaches?
A) The most common agents that cause neutropenic fever are viruses.**
B) Massc scoring, hypotension and COPD are negative risk factors
C) rectal touch is contraindicated for neutropenic patients
D) Antibiotic treatment should be started for a patient with neutropenic fever within 60 minutes at
the latest after the necessary cultures are taken.
E) The type of cancer is a factor that affects the development of neutropenic fever.

83) Which of the following molecules is not one of the endogenous angiogenesis inhibitors?
a. FGF
b. Angiopoietin-2
c. PSA
D. Interleukin-12******
to. Interferon-α
84) Which of the following is the correct option used for cancer screenings?
A) Colon cancer – Colon radiography
B) Prostate cancer – Prostate ultrasonography
C) Cervical cancer - PAP Smear**
D) Breast cancer - Breast MRI
E) Ovarian cancer – Ultrasonography

85) In which one does diet play the least role?


A-) Column Ca
B-) Stomach Ca
C-) Pancreatic Ca
D-) Malignant Melanoma**
E-) Breast Ca

86) A 65-year-old woman with rheumatoid arthritis presents with polyarthritis involving her hands,
wrists, and feet. He is currently on triple therapy with methotrexate 120mg Sc per week,
hydroxychloroquine 400mg per day and sulfasalazine 1g bid and prednisone 20mg per day. He has
a history of moderate to severe COPD, bowel perforation on the background of diverticulitis, and a
history of 2 shingles, one of which involves the eye. On physical examination, he has eight tender
and swollen joints. What is the next best treatment option?
A) Rituximab**
B) TNF alpha blocker
C) Baricitinib
D) Tocalizumab
E) Tofacitinib
88) A 51-year-old male patient applied with complaints of shortness of breath that increased with
exertion and a moderate level of fresh blood producing cough for 2 days. During the previous 3
weeks, he had felt very tired and had noticed migratory joint and muscle pain. In the anamnesis, it
was learned that he saw his family doctor several times last year due to recurrent nasal
congestion and received treatment with the diagnosis of allergic sinusitis. He had no significant
illness in his past. On examination, his temperature was 37.9 °C, blood pressure was 160/100
mmHg, and O2 saturation was 89% in air. Bronchial breath sounds were heard in the middle and
lower zones of both lung fields. Neurological examination revealed conductive hearing loss in his
left ear. Hb 9.4 g/dL; WCC 13.2x10^9/mL; neutrophils 9.1x10^9/mL; platelets 623x10^9/L; CRP 160
mg/LUrin microscopy; Red cell casts were present. What is the most likely diagnosis?
A)Microscopic polyangiitis
B)Good pasture syndrome
C)Systemic Lupus Erythematosus
D)Rheumatoid Vasculitis
E)Granulamatous Polyangiitis*

89) 61-year-old woman, with a history of Reynound phenomenon and ischemic ulcer in the foot for
8 years. He smokes 15 branches of cigarettes a day. On examination, there is evidence of spindle
fingers with distal pulp atrophy and coldness in the fingers and toes. ENA+ for anticentromere
antibody. 72% reduction in DLCO on Ac function test. Chest CT and Echo are normal. What is your
diagnosis?
A) Limited Scleroderma **
B) Diffuse Scleroderma
C) SLE
D) Morphrea

90) Which of the following idiopathic inflammatory myositis autoantibodies is myositis specific
autoantibody?
A) anti-PM-SCL
B) anti-U1RNP
C) anti-Jo-1**
D) anti-Ro/SSA
E) anti-ku
91. Which drug is generally not used in the treatment of acute gout attacks?
A. Febuxostat**
B. Non-steroidal anti-inflammatory drugs
C. Corticosteroids
D. Colchicine
E. IL1 antagonists

92) Which of the following is not a risk factor for gout?


A. Alcohol use

O
B. Fertility period female gender
C. Diuretics
D. Chronic kidney disease
E. Obesity

94) Which is true about arthritis?


A) Arthritis lasting more than 6 months is chronic arthritis.
B) If there is involvement of 5 regions, it is oligoarthritis
C) Gout and pseudogout cause intermittent arthritis*

95) In which of the following does anterior uveitis not occur?


A) Gout**
B) Oligoarticular JIA
C)Behçet
D)
E)Ankylosing Spondylitis

96) Which is not one of the ASAS criteria for axial spondyloarthropathy?
a-myositis*
b-uveitis
c-HLA-B27
d-Psoriasis
e-family history for spA
97) Which one is an IL-6 inhibitor?
A) Infliximab
B) Rituximab
C) Tocilizumab**
D) Tofacitinib
E) Secukinumab

98) Which of the following is not an autoinflammatory disease?


A FMF
B carpal tunnel syndrome**
C PFAPA Syndrome
D TNF receptor-associated periodic fever syndrome
E Cryoprinopathies

99) What is the minimum age for screening for genetically transmitted cancer syndromes?
A) 20
B) 10
C) 18**
D) 30
E) 25

100-) Which of the following does not occur as a result of chromosome breaks in neoplastic cells?
a. deletion
b. translocation
c. duplication
D. ring chromosome
to. Monosomy**
Makeup
1- Which of the following statements is not true about the complications of diabetes?

O
A) In the hyperosmolar hyperglycemic nonketotic state, water and sodium deficiencies should not
be corrected quickly.
B) Bicarbonate replacement is not routine in diabetic ketoacidosis.
C) Retinopathy and nephropathy complications often develop together in diabetic patients.
D) The probability of thrombotic stroke is high in diabetic patients.
E) An unconscious hypoglycemic patient should be given sugar water or fruit juice quickly.

2- A male patient who has been diagnosed with Type 1 diabetes mellitus since the age of 23
applies to the emergency department with complaints of fever, weakness, cough, diffuse
abdominal pain and vomiting. Fever and cough started 2 days ago, the patient could not eat or
drink water. The patient was receiving intensive insulin therapy at home, including insulin glargine
28 U at night and a short-acting insulin analogue before each meal. On examination, he was
tachypneic, his temperature was 39°C, his pulse was 105/min, his respiratory rate was 26/min, and
his blood pressure was 90/70 mmHg. Mucous membranes were dry, skin turgor was decreased,
and rales were heard in the lower right chest. His consciousness was slightly confused. According
to rapid hematological and biochemical tests, hematocrit 48%, hemoglobin 14.3 g/dl, white blood
cells 18,000/microliter, glucose 450 mg/dl, urea 60 mg/dl, creatinine 1.4 mg/dl, Na 152 mEq/L, K
5.3 mEq. /L and Cl were 110 mmol/L. Arterial pH 7.5, PO2 95 mmHg, PCO2 28 mmHg, bicarbonate
9 mEq/L and O2 saturation 98% were detected. Ketone levels were found to be high in urine and
serum. Glucose in urine was 600 mg/dl and urine density was high. Which of the following is not
appropriate for the treatment of this patient?
A) Saline and insulin infusion should be started quickly.
B) 20 mEq potassium should be added to each 1 L of saline.
C) Arterial pH and electrolyte levels should be measured every 2 hours.

O
D) If infection is thought to occur, antibiotic treatment should be started without waiting for blood
culture results.
E) There is no need for bicarbonate replacement in this patient.
3- Drug treatment is started for a 48-year-old patient diagnosed with type 2 diabetes mellitus. 2
months after starting the treatment, it was determined that the patient's fasting blood sugar was
normal, but his urinary blood sugar was high. In addition, the patient reported that he had lost
weight and his blood pressure was the same as before.He says that even though he takes
antihypertensive medications, he is now at normal levels. Drugs used in this patient's treatment.
Which one is most likely to provide these effects?
A) Metformin
B) Acarbose

O
C) Empagliflozin
D) Glyburide
E) Insulin aspart

4- Which of the following is incorrect for Diabetes Mellitus?


A) The risk of retinopathy increases in prediabetic patients.
B) The diagnosis of MODY should come to mind in a diabetic patient who starts at an early age,
does not have insulin resistance and can be treated with sulphonulurea.
C) Insulin islet antibody may also be positive in patients without a diagnosis of type 1 DM.
D) Gemetic penetrance is low in patients with type 2 DM.

O
E) In patients with LADA, the development of ketoacidosis is prevented since beta cell functions
continue initially.

5- Trousseau's sign is a physical examination finding of which clinical condition?


A) Cushing's syndrome
B) Hyperphosphatemia
C) Thyrotoxicosis

O
D) Hypocalcemia
E) Hyponatremia

6- Which of the following is the most common cause of osteomalacia?


A) Advanced age
B) Increased parathyroid hormone release
O
C) Vitamin D deficiency
D) Osteogenesis imperfecta
E) Hypoparathyroidism
7- What is the main apoprotein of chylomicrons?
A) Apo A-1
B) Apo C-11
C) Apo B-100
D) ApoE

O
E) Apo B-48

8- Which of the following endocrinological diseases is not a cause of obesity?


A) Polycystic ovary syndrome
B) Adrenal insufficiency
C) Insulinoma

O
D) Growth hormone deficiency
E) Hypothyroid

9- Respiratory problems for various reasons in acromegaly patients


visible. Which of the following is not a cause for respiratory problems in patients with acromegaly?
A) Macroglossia

OB) Pulmonary fibrosis


C) Soft tissue hypertrophy in the larynx and pharynx
D) Deformation occurring in the mandible
E) Obstructive sleep apnea syndrome

10- Which of the following is incorrect regarding the syndrome of inappropriate ADH?
A) Patients often present with hypnatremia.
B) Sodium excretion in urine has increased.

O
C) Fractionated uric acid excretion decreased.
D) Hyponatremia improves with fluid restriction.
E) Serum urea level decreased.
11- It is useful to use stimulation tests in the diagnosis of anterior pituitary hormone deficiencies.
Which of the pairings of anterior pituitary hormone deficiency and the stimulation test used is
incorrect?
A) Growth hormone deficiency - insulin hypoglycemia test

O
B) TSH deficiency - insulin hypoglycemia test
C) Growth hormone deficiency -glucagon stimulation test
D) ACTH deficiency - short ACTH stimulation test
E) ACTH deficiency - metyrapone test

12- Clinical findings in patients with hyperprolactinemia vary depending on whether the patient is a
premenopausal, postmenopausal woman or man. Which of the following is not a finding that may
be seen in the presence of hyperprolactinemia in premenopausal female patients?
A) Infertility
B) Oligomenorrhea
C) Galactorrhea

OD) Osteoporosis
E) Hirsutism

13- Which of the following is not an indication for Cushing's screening?


A) Osteoporosis and bone fractures at a young age
B) Presence of specific symptoms such as proximal muscle weakness and pink-purple striae

O
C) Being diagnosed with diabetes before the age of 40
D) Failure to grow taller despite gaining weight in childhood
E) Presence of adrenal incidenteloma

14- Which of the following is true for primary adrenal insufficiency?


A) In developed countries, the most common cause is tuberculosis.
B) Lymphocytosis and eosinophilia are expected in the peripheral blood smear.
C) The most common electrolyte disorder is hypokalemia.
D) Hydrocortisone treatment alone is sufficient; the addition of mineralocorticoids is not
necessary.

OE) Zona fasciculata and reticularis are affected, glomerulosa functions are preserved.
15- Which of the following is not true for primary hyperaldosteronism?
A) Patients with blood pressure > 140/90 mmHg despite triple anti-hypertensive therapy should be
screened.
B) Only patients whose blood pressure can be controlled with quadruple antihypertensive should
be screened.
C) Hypertensive patients with spontaneous or diuretic-induced hypokalemia should be screened.

G
D) Patients with diabetes and hypertension should be screened.
E) Patients with adrenal incidentiloma and hypertension should be screened.

16- Which of the following is incorrect during treatment in a patient diagnosed with gluten
enteropathy?
A) Pneumococcal vaccine is recommended for patients due to hyposplenism.
B) Histological recovery usually occurs within 6-24 months.
C) After diagnosis, patients are recommended a lactose-free diet for 3 months. !
D) Diet compliance monitoring of patients is done according to endoscopic findings.
E) In many patients, symptoms improve in the first 2 weeks with diet compliance.

17- Which of the following is not a symptom of acute pancreatitis?

O
A) Hypercalcemia
B) Increase in hematocrit value at baseline
C) 4-6 fold increases in ALT and AST values
D) Mild hyperglycemia initially
E) Leukocyte value between 10000 - 25000 in most patients

18- Which of the symptoms that causes a patient with chronic pancreatitis to be admitted to the
hospital is most common?
A) Steatorrhea
B) Nausea-vomiting
C) Creator
D) Weakness and weakness

O
E) Abdominal pain
19- Which of the following is the most common complication of cirrhosis?

G
A) Acid
B) Spontaneous bacterial peritonitis
C) Varicose bleeding
D) Hepatorenal syndrome
E) Hepatopulmonary syndrome

20- Which of the following is not important in determining the risk for esophageal variceal
bleeding?

O
A) Location of varicose veins
B) Size of the heir
C) Accompanied by gastric varicose veins
D) Appearance of the heir
E) Advanced liver disease

21- Which of the following does not play a role in the etiology of acute liver failure?

O
A) Chronic hepatitis B
B) Paracetamol
C) Autoimmune hepatitis
D) Wilson's disease
E) Mushrooms
22- Which of the following is a motility disorder with esophagogastric junction disorder?
A) Lack of contractility
B) Distal esophageal spasm
C) Hypercontractile esophagus

O
D) Achalasia
E) Ineffective esophageal motility
23- Which of the following is not one of the peripherally acting u-opioid antagonists?
A) Naloxegol
B) Alvimopan

O
C) Lubiprostone
D) Methylnaltrexone
E) Naldemedine
24- Which of the following is not one of the relative risk factors for the development of malignancy
in pancreatic cystic neoplasia?
O
A) Pancreatic duct diameter larger than 10 mm
B) Thickened cyst wall
C) Recurrent acute pancreatitis
D) Increased CA 19-9 level
E) Cyst size larger than 3 cm

25- Which of the following is the most common benign non-epithelial tumor of the esophagus?

O
A) Leiomyoma
B) Granular cell tumor
C) Fibrovascular polyp
D) Hemangioma
E) Lipoma

26- Which of the following is the definition of Budd-Chiari syndrome?


A) It is the expansion of the hepatic sinusoidal lumen without blockage or infiltration by abnormal
cells or materials.
B) Thrombus formation in the portal vein and/or its right or left branches.

OC)to the
It is defined as obstruction of the hepatic venous flow anywhere from the small hepatic venules
entrance of the inferior vena cava to the right atrium (regardless of the mechanism of the
obstruction).
D) It is a condition characterized by congestion as a result of the destruction of endothelial cells in
the sinusoids, especially in the central part of the hepatic lobule, and focal obstruction of the
sinusoid lumen.
E) Hepatic venous flow is obstructed due to cardiac or pericardial diseases or sinusoidal
obstruction syndrome.
27- Which of the general approach principles to upper gastrointestinal system bleeding is correctly
stated?
A) Nasogastric or orogastric aspiration/lavage is routine practice.
B) Second-look endoscopy is a mandatory part of the approach to non-varicose upper GI bleeding.
C) Helicobacter pylori eradication is necessary in all patients.
D) Early (<12 hours) upper GI endoscopy following hemodynamic resuscitation should be
performed in every patient.

O
E) Hemodynamic stabilization and hemodynamic monitoring are indispensable for gastrointestinal
bleeding control.
28- Which of the following is not true for Familial Adenomatous polyposis (classical FAP)?
A) Lifetime risk of duodenal cancer is 4-12%.
B) Adenomas begin to form at the age of 10-12, the risk of cancer in the 5th decade of life is 90%,
the average colon age is 39.
C) Autosomal dominant inheritance. Mutation in the gene located at 5q21-q22 (APC gene) (tumor
suppressor gene)

O
D) Having 3 or more relatives diagnosed with colorectal or endometrial cancer or small intestine or
ureter or renal pelvis cancer. One of them must be a first degree relative of the other two for
diagnosis.
E) It is the most common adenomatous polyposis syndrome, its estimated prevalence is
1/5000-7500.

29- Which of the following is not included in the diagnosis of autoimmune hepatitis?

O
A) 20-fold increase in liver enzymes
B) There is no etiological agent known to cause hepatitis
C) Presence of autoantibodies such as ANA, ASMA or LKM1
D) High serum gammaglobulin level
E) Presence of at least periportal hepatitis in histological examination

30- Which of the following liver cysts has the highest malignant potential?
A) Simple cyst
B) Hydatid cyst
C) Polycystic liver disease
D) Type 1 choledochal cyst

O
E) Biliary cystadenoma

31- Which of the following benign liver lesions is related to hormones?

OHepatic adenoma
A)
B) Focal nodular hyperplasia
C) Caroli disease
D) Hemangioma
E) Nodular Regenerative Hyperplasia
32- A 30-year-old male smoker presents with right lower quadrant pain and non-bloody diarrhea
attacks. Laboratory results: hemoglobin 10.5 g/dL, MCV 78, CRP 35 (upper limit 5), albumin 3.2
g/dL. Stool culture and Clostridium difficile tests are negative. Which of the following is not an
expected finding in this patient?
A) Cobblestone view during colonoscopy
B) Longitudinal deep ulcers running parallel to the colonoscope
C) Protection of the rectum
D) Narrowing of the ileocecal valve

O
E) View of lead pipe in the column

33- Which of the following is incorrect for alcoholic liver disease?

O
A) After diagnosis, liver transplantation should be recommended to all decompensated patients.
B) Glucocorticoid treatment is indicated for patients with discriminant function >32.
C) Abstinence is the most important prognostic factor in alcoholic hepatitis.
D) The presence of malnutrition in patients should be investigated and corrected.
E) Hepatomegaly is detected in most patients.

A 34-28 week pregnant patient comes with complaints of itching that is intense at night. No
fatigue or abdominal pain. Laboratory results: ALT 50, AST 55 GGT 250 bilirubin total 1.8. Which of
the following is not expected in this disease?
A) Serum bile acids >10 micromol/L
B) Family history of cholestasis during pregnancy
C) Immediate improvement in the postpartum period

O
D) Hypoglycemia
E) Recurrence in other pregnancies

35- A 33-year-old man presents to the gastroenterology outpatient clinic with chronic, ongoing
epigastric pain that has been present for 8 months. The pain is dull, accompanied by swelling, and
improves with food intake. Nausea or vomiting He has no weight loss and does not use NSAIs.
Which of the following is the most appropriate next step in the management of this patient?
A) Colonoscopy

O
B) Esophagogastroduedonoscopy
C) Abdominal Computed Tomography
D) Checking proton pump inhibitor (PPI) treatment by giving it for 6 weeks
E) Abdominal Ultrasonography
A 36-67-year-old female patient presents with complaints of jaundice and itching in the eyes,
which started 2 weeks ago. On physical examination, jaundice appears on the skin. Laboratory
values: ALT 158 U/L AST 102 U/L Alkaline phosphatase 479 U/L (25-100) GGT 182 U/L (10-50)
Total bilirubin 4.9 mg/dL Direct bilirubin 2.1 mg/dL Albumin and INR- PTT is normal.
What is the next most appropriate step in diagnosis?
A) Endoscopic ultrasonography (EUS)
B) Liver biopsy

O
C) Endoscopic retrograde cholangiopancreatography (ERCP)
D) Computed Tomography (CT)
E) Abdominal Ultrasonography (USG)

37- Which of the following statements about biochemical, function tests and metabolic diseases
of the liver is not true?
A) The first step in evaluating a slightly elevated serum aminotransferase level is to perform repeat
tests to confirm the persistence of the elevated value.
B) Low serum albumin and prolonged prothrombin time (PT) indicate chronic liver parenchymal
disease (cirrhosis).
C) D-penicillamine, trientine and zinc are used for maintenance in the treatment of patients with
hemochromatosis.
D) Wilson disease is an autosomal recessive disease with the defective gene ATP7B on
chromosome 13.

O
E) Macrovesicular fat accumulation and Mallory equivalent bodies in liver biopsy indicate
nonalcoholic fatty liver disease (NASH).

38- Which of the following is wrong?


A) Depression is an important cause of malnutrition in older people.
B) Malnutrition can cause pressure ulcers. It can also cause pressure ulcers to not heal.
C) Muscle loss (sarcopenia) in elderly patients is associated with both aging and diseases.

O
D) Strict dietary restrictions are appropriate for elderly patients. Meals should be frequent and
small in quantity.
E) The daily amount of calcium that individuals over 70 years of age should take is approximately
1000-1200 mg, the amount of vitamin D is 800 IU, the amount of protein is 1-1.2 g/kg, the amount
of energy is 1800 kcal/day for men and 1400 kcal/day for women.
39- A 73-year-old female patient applies to the geriatric clinic with complaints of high blood
pressure and sugar. It was determined that he had a history of diabetes mellitus for 15 years, and
in the complication screening, he was being checked for neuropathy and retinopathy. In the tests
performed within the scope of comprehensive geriatric evaluation, Katz basic activities of daily
living scale: 6/6 Lawton-brody instrumental activities of daily living 8/8 Mini-nutritional
assessment: 12/14 Geriatric depression scoring 7/15 and Mini-Mental test score 30/30. . Upon
system interrogation, it is determined that the patient is suffering from severe diabetic neuropathy
pain and has urinary incontinence. It states that the incontinence complaint occurs in small
amounts with coughing and sneezing. Which option gives the correct options for the case given
above? 1. A full history and physical examination should be performed on the patient, first the
causes of temporary incontinence should be excluded. 2. The patient's incontinence type is urge
type incontinence. 3. The patient was found to be in a depressed mood in the depression
screening performed with the GDS score. 4. The first treatment to be given to the patient will be
medication administration. 5. In treatment, behavioral methods should be tried first. Kegel exercise
and regulating fluid intake are among them. 6. The most appropriate drug treatment option for the
patient is duloxetine. 7. The most appropriate drug treatment option for the patient is darifenacin.
A) 2, 3, 4, 6
B) 1, 2, 3, 5
C) 1, 3, 5, 7

O
D) 1, 3, 5, 6
E) 1, 2, 3, 5, 7

40- Which of the following is wrong?


A) Individuals aged 65 and over should receive the pneumococcal vaccine at least once in their
lifetime.
B) It is important to consider the patient's remaining life expectancy and cognitive status when
deciding what preventive health measures to offer.

O
C) Vaccines are an example of secondary protection.
D) The condition being screened for must be serious and prevalent in the population being tested.
E) Lead-time bias is when the person spends more time as a patient, but the course of the disease
does not fundamentally change.

41- Which of the following is not one of the basic daily life activities?
A) Transfer
B) Going to the toilet
C) Nutrition

O
D) Regulating your medications
E) Dressing
42- Aging is associated with multiple changes in the gastrointestinal system. What is the most
clinically important change in the gastrointestinal tract with aging?

O
A) Decrease in intestinal motility
B) Decrease in pancreatic secretions
C) A decrease in the area of absorption of the small intestine
D) Increased prevalence of atrophic gastritis
E) Increased prevalence of Helicobacter pylori

43- Which of the following is incorrect regarding the pathogenesis of hypertension in the elderly?
A) Sodium sensitivity increases.
B) Isolated systolic hypertension is greater than systolic plus diastolic hypertension.
C) Arterial stiffness increases.
O
D) Vascular remodeling and fibrosis increase arterial elasticity.
E) Endothelial dysfunction increases.

44- Which of the following is not routinely requested in the initial evaluation of an elderly person in
delirium?
A) Complete blood count
B) Biochemistry (metabolic panel)
C) Random blood sugar

O
D) Lumbar puncture
E) Urine analysis

45- What is the most likely diagnosis of dementia in a patient whose dominant symptoms are
hallucinations, parkinsonism, fluctuating symptoms, and neurolepric sensitivity?
A) Alzheimer's disease
B) Frontotemporal dementia

O
C) Dementia with Lewy bodies
D) Parkinson's dementia
E) Vascular dementia
46- Which of the following is not an expected finding for SLE nephritis?

O
A) Hypotension
B) Hematuria
C) Proteinuria
D) Hyperkalemia
E) Cellular casts

47- Which of the following causes AL Amyloidosis?


A) FMF (Familial Mediterranean Fever)
B) Rheumatoid arthritis
C) Ulcerative colitis

O
D) Multiple Myeloma
E) Tuberculosis

48- Which of the following is not true for acute tubular necrosis (ATN)?
A) ATN is the most common cause of acute renal failure in hospitalized patients.
B) Sepsis occurs due to prolonged prerenal azotemia and toxic agents.
C) Dysmorphic erythrocytes and erythrocyte casts are classic findings of ATN.
D) In ATN, protrinuria is usually below the nephrotic level.

GE) Fractionated sodium excretion is usually <1%.


49- Which of the following anion gap occurs due to normal metabolic acidosis?
A) Lactic acidosis
B) Diabetic ketoacidosis
C) Uremic acidosis
D) Salicylate intoxication

O
E) Renal tubular acidosis **
50- Which of the following is incorrect for chronic kidney disease (CKD) progression?
A) The rate of progression increases with age.
B) The rate of progression in women is slower than in men.

O
C) Nephrotic level proteinuria reduces CKD progression.
D) Tight blood sugar control in diabetes mellitus is effective in reducing CKD progression.
E) Hyperuricemia is a factor in CKD progression.
51- Which of the following is most appropriate for the clinic of renal osteodystrophy due to chronic
kidney disease?
A) Phosphate excretion increased, parathyroid hormone level was low, 1,25 OH D votamine
decreased, FGF-23 increased
B) Phosphate excretion decreased, parathyroid hormone level was low, 1,25 OH vitamin D
increased, FGF-23 decreased

O
C) Phosphate excretion decreased, parathyroid hormone increased and 1,25 OH vitamin D
decreased, FGF-23 increased
D) Phosphate excretion is decreased, parathyroid hormone level is low, 1, 25 OH vitamin D is
decreased, FGF-23 is decreased
E) Phosphate excretion increased, parathyroid hormone level increased, 1, 25 OH vitamin D
decreased, FGF-23 increased

52- Which of the following is not a risk factor for the development of contrast-associated
nephropathy?
A) Congestive Heart Failure
B) Chronic kidney disease
C) Hypovolemia

O
D) Hypernatremia
E) Diabetes Mellitus

53- A 22-year-old female patient visits the emergency department due to nosebleeds and fatigue.
On physical examination, the patient appears pale, with petechial rashes on his legs and
splenomegaly. Hemoglobin is 8 g/dl and white blood cell is 110×109/L. In the peripheral smear,
immature cells with narrow cytoplasm and dense chromatin with visible nucleoli were observed.
Myeloperoxidase is negative in cytochemical staining. In flow cytometry, these cells are
determined to be CD10, CD19, CD22 positive. Which diagnosis would you first consider in this
patient?
A) AML M4
B) AML M3

O
C) B-ALL
D) T-ALL
E) CLL
54- In the immunophenotypic evaluation of blasts obtained from the bone marrow of an acute
myeloid leukemia patient with Down syndrome, it was reported that there was a CD61 marker on
the surface of the blast cells. Which of the following is this patient's most likely FAB type?
A) AML M1
B) AML M2
C) AML M4
D) AML M5

O
E) AML M7

55- Which of the following statements about acute lymphoblastic leukemia (ALL) is incorrect?
A) The most common ALL immunophenotype in childhood is B-cell.
B) The most common ALL immunophenotype in adults is B-cell.
C) In B-cell ALL, a white blood cell count >30×10⁹/L in the peripheral blood at the time of diagnosis
is a negative risk factor.
D) Being 36 years old is a negative risk factor.

O
E) Hyperdiploidy is a negative risk factor, especially in childhood ALL.

56- Which of the following is true for Antiphospholipid Antibody Syndrome?


A) aPTT is long and corrects with correction testing.
B) Only venous thrombosis is observed in patients.
C) Thrombocytosis is observed.
D) Hereditary thrombophilia is a risk factor.

O
E) Thrombocytopenia is observed.

57- Which of the following is true for von Willebrand (vWB) Disease?
A) Only primary hemostasis is affected.
B) Bleeding Time is normal.
C) Desmopressin is used in all types of vVB disease.
O
D) It is the most common hereditary bleeding diathesis.
E) PFA 100 test is normal.
58- Which of the following is not true for Polycythemia Vera (PV)?
A) The most important cause of morbidity and mortality in these patients is thromboembolic
complications.
B) Most patients have JAK-2 mutation.

O
C) Blood oxygen pressure is low.
D) Erythrocyte sedimentation rate decreased.
E) Serum erythropoietin level is low.

59- There are many clinical and laboratory findings in multiple myeloma patients. Select the one
that is not among the expected findings in this patient group.
A) Acute kidney failure
B) Hypercalcemia
C) Mechanical and inflammatory type pain
O
D) Osteosclerotic bone lesion
E) Neuropathy

60- A 70 year old male patient presents with swelling in his neck that has been present for 6
months. On physical examination, multiple lymphadenopathy measuring 4 cm in size is detected.
In the patient's hemogram, Hb is measured as 13 g/dl, platelets are 200,000/mm³ and leukocytes
are 150,000/mm³. How should the diagnostic procedure proceed for this patient?

O
A) Performing a trucat biopsy from the lymph node after performing a peripheral smear
B) Excisional removal of one of the lymph nodes after performing a peripheral smear
C) Performing cytogenetic examination after performing a peripheral smear
D) Performing flowcytometric examination after performing a peripheral smear
E) Performing fine needle aspiration from the lymph node after performing a peripheral smear

61- The patient using warfaarin (coumadin) presents with gastrointestinal system bleeding (the
patient has melena). In the hemogram, Hb is measured as 11 g/dl, platelets are 150,000/mm³ and
INR is 10. A decision is made to transfuse the patient. Which blood component should be given to
this patient?
A) AB0 Rh compatible erythrocyte suspension
G
B) AB0 Rh compatible fresh frozen plasma
C) AB0 Rh compatible platelet suspension
D) Cryoprecipitate
E) AB0 Rh compatible whole blood
62- Which of the following is not one of the treatment approaches used in the treatment of
aplastic anemia?
A) Antithymocyte globulin
B) Allogeneic stem cell transplantation

O
C) Azacitidine
D) Cyclosporine A
E) Eltrombopag

63- A 52-year-old female patient with a history of Hashimoto's thyroiditis and receiving thyroid
hormone treatment consults her family physician with complaints of increasing weakness, fatigue
and paleness for several months. In the complete blood count, it is seen that the leukocyte count
is 3200/uL, the neutrophil count is 1100/uL, Hb is 6 g/dl, hematocrit is ?%, MCV is 124 fl, and the
platelet count is 96×10⁶/uL. According to the biochemical evaluation of the patient, lactate
dehydrogenase 1200/u/l (N: 120-246), total bilirubin 3.2 mg/dl (N 0.3-1.2), indirect bilirubin 2.6
mg/dl (N: <0). ,9) and liver and kidney function tests are within normal limits. The patient's
measured reticulocyte count is 1%. Which is wrong in approaching this patient?
A) The patient's corrected reticulocyte level is 0.45%.
B) Pernicious anemia should be investigated in the patient.
C) The patient's cobalamin (B12) and folic acid levels should be checked.

O
D) The patient should be started on oral cobalamin treatment.
E) The patient's homocysteine level is high.

64- Which of the following is not a drug used in myelodysplastic syndrome?


A) Luspatercept
B) Azacitidine
C) Decitabine
D) Lenalidomide

O
E) Imatinib
65- A 35-year-old female patient with no other health problems is referred to the outpatient clinic
due to anemia. In complete blood count, Hb is 8 g/dl, mean erythrocyte volume (OEH) is 70 fl,
mean erythrocyte hemoglobin (OEHb) is 24 pg, red blood cell (RBC) count is 3.6×10⁶/uL, white
blood cell (WBC) count is 7500/uL. (leukocyte formula is normal) and platelet count is 438×10⁶/uL.
It is determined that the patient's serum ferritin is 8 mg/dl, serum iron is 27 ug/dl and CRP is <0.5
mg/dl. What is the most appropriate next step in the evaluation of this patient?
A) Hemoglobin electrophoresis should be requested to exclude beta thalassemia carrier state.
B) Oral iron (Fe+2) treatment should be started for iron deficiency anemia.
O
C) Chronic disease should be investigated.
D) A bone marrow biopsy should be performed to exclude sideroblastic anemia.
E) 2 units of erythrocyte suspension should be transfused.

66- Which of the following types of cancer is not associated with BRCA1/2 genes?
A) Breast cancer
B) Pancreatic cancer

O
C) Thyroid cancer **
D) Prostate cancer
E) Ovarian cancer

67- Which of the following is not true about nodular thyroid disease?
A) Thyroid nodules are lesions that are different from the normal thyroid parenchyma around them
and can be seen radiologically.
B) Most patients with thyroid nodules are euthyroid.
C) The first laboratory test in the evaluation of thyroid nodule is serum TSH level.
D) Thyroid fine needle aspiration biopsy is the most reliable method in differentiating benign and
malignant nodules.

O
E) The presence of microcalcification in the thyroid nodule is a characteristic feature of benign
thyroid nodules.

68- Which of the following is not true about thyroiditis?

O
A) Prednisolone is always used in subacute granulomatous thyroiditis.
B) In radiation thyroiditis, the thyroid gland is usually hard and painful on palpation.
C) Subacute granulomatous thyroiditis is a painful, non-infectious and self-limiting disease of the
thyroid gland.
D) Subacute lymphocytic thyroiditis is an autoimmune, painless inflammation of the thyroid gland.
E) In Riedel's thyroiditis, cytology may not be sufficient due to fibrosis.
69- Which of the following is not true about hypothyroidism?
A) Low serum thyroxine (fT4) and high serum thyroid-stimulating hormone (TSH) levels are
diagnostic of primary hypothyroidism.
B) TSH level is used to monitor primary hypothyroidism treatment.
C) Slowness in movements and speech and hoarseness are some of the symptoms of
hypothyroidism.
O
D) Iodine deficiency is the most common cause of hypothyroidism in developed countries.
E) People who use certain drugs such as lithium, amiodarone, and IFN-alpha are at risk for the
development of hypothyroidism.

70- Which of the following is not true about hyperthyroidism/thyrotoxicosis?


A) The most common cause of thyrotoxicosis is Graves' disease.
B) The presence of thyroid stimulating antibody (TSab) positively correlates with active disease in
diffuse toxic goiter.
C) Radioactive iodine uptake (RAIU) is increased in Graves' disease.
D) Thyroglobulin level is useful in the diagnosis of thyrotoxicosis due to exogenous thyroid
hormone.

O
E) Methimazole inhibits peripheral T4-T3 conversion.

71- Which of the following is not associated with an increased risk of hypoertension during
pregnancy?

O
A) Fourth pregnancy
B) Hydrops fetalis
C) Multiple pregnancy
D) Obesity
E) Chronic kidney disease

72- Proteinuria and hematuria were detected in the urinalysis of a patient with anti-GBM antibodies
detected in his serum. Which of the following is the most likely diagnosis?
A) Minimal change disease

O
B) Rapidly developing glomerulonephritis
C) Interstitial nephritis
D) Focal segmental glomerulosclerosis
E) IgA nephropathy
73- Which of the following is not a clinical symptom of hypokalemia?
A) Arrhythmias

O
B) Resistant hypertension
C) Rhabdomyolysis
D) Insulin increase
E) Metabolic alkalosis

74- What is the main mechanism responsible for the pathogenesis of preeclampsia?
O
A) Uteroplacental insufficiency
B) Decrease in circulating antiangiogenic factors
C) Proteinuria
D) Decrease in glomerular filtration rate
E) Reduced angiotensin II receptor activity

75- Which of the following statements about renal replacement therapy is correct?
A) A person with eGFR less than 5 ml/min/1.73m² should be treated with dialysis.
O
B) Mortality is less with kidney transplantation than dialysis.
C) Mortality is less in hemodialysis than in peritoneal dialysis.
D) The minimum hemodialysis frequency is three times a week.
E) Peritoneal dialysis is performed 5 days a week.

76- Which of the following statements about fluid and electrolyte metabolism is not true?
A) Approximately 50% of body weight in women and 60% in men is water.
B) The extracellular fluid compartment consists of plasma fluid and interstitial fluid volume.
C) Normal plasma osmolality is between 275-290 mosmol/kg.
O
D) Phosphate esters are the main anions of the extracellular fluid compartment.
E) Sodium, the main cation in the extracellular fluid compartment
77- Which of the following cancer drugs is a monoclonal antibody type?

A) Imatinib

B) Interferon alpha

C) Gemcitabine

O
D) Trastuzumab

E) Bortezomide

78- Which of the following cytotoxic agents can cause morbidity and mortality by causing
pulmonary fibrosis?

A) Vincristine

O
B) Bleomycin

C) Doxorubicin

D) 5 fluorouracil

E) Cisplatin

79- A 56-year-old female patient with locally advanced home left breast cancer underwent left
modified radical mastectomy surgery following 4 cycles of systemic combined chemotherapy
before curative surgery. For what purpose was the chemotherapy given to the patient before the
surgery?

A) Palliative chemotherapy

B) Adjuvant chemotherapy

O
C) Neoadjuvant chemotherapy

D) Maintenance chemotherapy

E) Consolidation chemotherapy
80- Which of the following is wrong?

A) The microorganisms most frequently responsible for the development of neutropenic fever are
bacteria.

B) Malignancies that commonly cause hypercalcemia include breast cancer and multiple
myeloma.
O
C) Peaking of the P wave is one of the typical findings of hyperkalaemia seen on the ECG.

D) For spinal cord compressions, 16 mg/day dexamethasone should be started.

E) The most common symptoms of IIBAS (increased intracranial pressure syndrome) are
headache and nausea.

81- Which of the following is wrong?

O
A) HTLV-8 virus can cause Kaposi's sarcoma.

&
B) Exposure to nitroso compounds in pesticides can lead to the development of cancer.

O
C) Today, the main source of radiation to which human beings are exposed is the procedures
performed for examination and treatment purposes.

D) Screening the community with colonoscopy for early diagnosis of colon cancer is an example
of primary cancer prevention.

E) Smoking is one of the leading modifiable risk factors that cause cancer.

82- Which of the following is wrong?

A) In superior vena cava syndrome, edema on the face and arms, pleatorrhea and hoarseness may
be observed.

B) Loss of sensation in the extremities, loss of strength and sphincter dysfunction are the
symptoms of spinal cord compression.

O
C) Sharpening of the T wave and shortening of the QT interval are ECG findings of hyperkalaemia.

D) In hypercalcemia, dry skin, constipation and changes in consciousness may occur.

E) Hyperuricemia, hypocalcemia, hyperphosphatemia and hyperkalemia are the findings of tumor


lysis syndrome.
83- A 35-year-old male patient was given chemotherapy for lymphoma 4 days ago and the patient's
temperature measured at home was found to be 39°C. Which of the following is not correct in the
next treatment of a patient whose neutrophil count is 450/mm³?

A) The patient's anamnesis should be taken and a complete physical examination should be
performed.

B) Rectal examination should be avoided during physical examination.

C) It is necessary to determine the risk status according to MASCC scoring.

D) If the patient is at high risk, intravenous antibiotic treatment should be given.

·
E) Antibiotic treatment should be started immediately after the patient's cultures are completed.

84- The hallmarks of cancer conceptualization are a heuristic tool for connecting the vast
complexity of cancer phenotypes and genotypes to a tentative set of fundamental principles.
Which of the following is not one of the main characteristics of cancer?

A) Genome instability and mutation

B) Immortalization of proliferative replication

C) Avoiding immune destruction

O
D) Inhibition of the development of resistance to cell death

E) Inflammation induced by tumor

85- Find the wrong option from the explanations about tumor markers?

A) Tumor markers can be classified as Enzymes, Hormones, Oncofetal antigens, Receptors.

B) It is used for clinical staging of cancer and estimating tumor volume.

C) It is used as a prognostic indicator of disease progression and to evaluate treatment success.

D) In case of cancer recurrence, the marker increases again.

O
E) Tumor marker values are not related to the effectiveness of treatment.
86- In order to prevent a non-hemolytic febrile reaction, it is not necessary to perform leukocyte
reduction in which of the following components?

A) Whole blood

B) Platelet suspension from HLA compatible donor

O
C) Fresh frozen plasma

D) Platelet suspension from random donor

E) Erythrocyte suspension

87- Which of the following is not a cause of massive splenomegaly?

A) Chronic myeloid leukemia

B) Kala Azar

C) Gaucher disease

D) Thalassemia major

O
E) Sickle Cell anemia

88- Which of the following is the cardioprotective agent used against the cardiotoxicity of
anthracyclines?

O
A) Dextrazoxane

B) Glutamine

C) Thiamine

D) Dextrose

E) Ethyol

89- Which of the following is not one of the findings of Churg Strauss syndrome?

A) Vasculitic neuropathy

B) Nasal polyposis

O
C) Congestive heart failure

D) Eosinophilia

E) Arthritis multilans
90- Which of the following does not cause secondary Raynaud's phenomenon?

A) Dermatomyositis

B) Cryoglobulinemia

C) Systemic sclerosis

D) Mixed connective tissue disease

O
E) Hyperthyroidism

91- A 28-year-old man goes to the emergency room after stumbling down the stairs at his home.
He states that he has not been feeling well for the last 2 weeks, with subfebrile fever, chills,
weakness, abdominal pain, and an unintentional loss of 5 kilos. He also described testicular pain.
On physical examination, there was mild diffuse tenderness to abdominal palpation. He has foot
drop on his right foot and has bilateral pretibial edema. Notable for livedo reticularis in the lower
extremities. Conventional angiography revealed microaneurysms in the ontrahepatic and intrarenal
arteries. Fusiform aneurysms and occlusive lesions were also seen throughout the superior
mesenteric artery distribution, including the hepatic and splenic branches. Which of the following
is the most likely diagnosis?

A) Temporal arteritis

B) Takayasu arteritis

O
C) Polyarteritis nodosa

D) Cryoglobulinemic vasculitis

E) Only related

92- 35-year-old female patient, who has been followed up for 6 years with the diagnosis of
seropositive rheumatoid arthritis, has swelling and tenderness in the proximal interphalangeal
joints of her wrists that has been going on for 4 months, and her treatment is methotrexate 15
mg/week, hydroxychloroquine 200 mg 2*1, salazopryne 500 mg tablet 3*2. and the patient using
deltacortil 1×7.5 mg/day, sedim:56 crp:40, DAS 28 score is calculated as 6.7. Which one should be
used in the next stage of treatment of the patient, who has been diagnosed with Lymphoma and
has been observed in remission for 3 years?

A) Infliximab

B) Golimumab

C) Tofacitinib

O
D) Rituximab

E) Etanercept
93- Which of the following is incorrect about Sjögren's syndrome?

A) It is 9 times more common in women than in men.

B) The immune response, including the innate and adaptive immune system, plays a central role in
pathogenesis by causing autoimmunity and chronic inflammation.

C) Unilateral, usually bilateral, parotid swelling is observed in 60% of patients with primary
Sjögren's.

O
D) Grades 2, 3 and 4 on minor salivary gland biopsy strongly support the diagnosis of Sjögren's
syndrome.

E) Distal renal tubular acidosis is one of the extraglandular findings.

94- Which one has no place in the treatment of acute gout?

O
A) Allopurinol

B) NSAID

C) Corticosteroids

D) Colchicine

E) IL-1 antagonists

95- Which of the clinical and laboratory findings matches between Adult Still's Disease (ESD) and
Macrophage activation syndrome (MAS) is correct?

A) Fever is continuous in ESH.

B) Central nervous system involvement is more common in ESR.

C
C) Pancytopenia is seen in MAS.

D) Hypertriglyceridemia is seen in ESR.

E) Arthritis is more prominent in MAS.

96- Which of the following is a sign of inflammatory arthritis?

A) Pain decreases at rest.

B) There is no loss of appetite.

O
C) Morning stiffness is > 1 hour.

D) There is no fatigue.

E) Acute phase values are normal.


97- Which of the following drugs used in the treatment of rheumatoid arthritis shows its effect by
blocking TNF-alpha?

A) Rituximab

B) Toeilizumab

O
C) Certolizumab

D) Abatacept

E) Tofacitinib

98- Which of the following diseases is not an autoinflammatory disease?

O
A) Idiopathic inflammatory myositis

B) Familial Mediterranean fever

C) Hyper Ig D syndrome

D) TNF receptor-related periodic syndrome (TRAPS)

E) Cryoprinopathies

99- Which of the following is not one of the clinical findings of antiphospholipid syndrome?

A) Venous thrombosis

O
B) Sacroilid

C) Recurrent first trimester miscarriages

D) Recurrent pregnancy-related morbidities (preeclampsia, eclampsia or premature birth)

E) Arterial thromboses

100-Which of the following is not one of the mucocutaneous lesions of Behçet's disease?

A) Oral aphthous

B) Genital aphtha

C) Papulopustular eruptions

D) Erythema nodosum

OE) Malar rash


2023/2024 INTERNAL MEDICINE THEORETICAL

1- In patients with diabetic nephropathy , blood pressure should generally be controlled


aggressively, starting with which of the following?

A) Diuretic

B) Statin

C) Ca channel blocker

O
D) Angiotensin inhibition

E) Beta blocker

2- Which intrinsic factor is related to falls in the elderly ?

A) Cognitive disorders

B) Vitamin D deficiency

C) History of falling in the past

D) Heart rate and rhythm disorders

O
E) Unsuitable grounds

3- Which of the following cytotoxic agents has a linear dose response curve?

O
A) vincristine

B) etoposide

C) paclitaxel

D) ifosfamide

E) 5 FU

4- Which is inevitable as a result of viral hepatitis?

A)chronic hepatitis

O
B)cirrhosis

C) hemangioma

D) hepatocellules ca

E) cholangiocellular ca
5- What is the purpose of requesting protein electrophoresis in a patient with suspected multiple
myeloma?

O
A) To understand whether monoclonal globulin is present or not

B) Determining the plasma cell increase rate

C) Determine whether the patient is in remission or not

D) Determining which drug should be given.

E) Determining the type of multiple myeloma

6- Which of the following is most likely a symptom of idiopathic inflammatory myositis ?

A) Having a family history of muscle disease

O
B) Subacute or chronic symmetrical proximal muscle weakness

C) Neuropathy

D) Weakness of facial or eye muscles

E) Periodic muscle weakness related to activity or hunger

7- When a patient has an URTI or gastroenteritis, he notices blood in his urine within a few days.
Given the possible diagnosis, what is the likely pathology finding?

O
A) Marked IgA accumulation with mesangial proliferation

B) Complement storage

C) Podocyte loss

D) Storage of immunoglobulin and complexes along the basement membrane

E) Focal segmental glomerulosclerosis

8- Which of the following is not expected to be seen in tuberculosis with gastrointestinal system
involvement ?

It is more common in those using steroids and anti- TNF .

B) Ileocecal involvement may be seen

C) Chronic abdominal pain, weight loss, fever and diarrhea may occur

O
D) Most of them have findings related to lung involvement.

E) Treated in the same way as lung involvement


9- Which of the following is not one of the findings of hypokalemia?

A) Arrhythmias

O
B) Worsening Hypertension

C) Rhabdomyolysis

D) Insulin Increase

E) Metabolic alkalosis

10- Posterior or panuveitis is a characteristic of which disease?

A) Systemic scleroderma

O
B) Behçet***

C) Gout

D) Sjogren's

E) Idiopathic inflammatory myositis

11- Which of the following is incorrect about hypertension in the elderly?

A) Sodium sensitivity increases.

C)... vascular elasticity increases due to reasons.* **

D) The arteries harden.

E) The function of the endothelium is impaired.

12- Which of the following drugs acts by inhibiting vascular endothelial growth factor receptor 2?

O
A. Ramucirumab

B. Trastuzumab

C. Bevacizumab

D. Aflibercept

E. Pazopanib

13- What is wrong with superior vena cava syndrome?

malignant causes such as mediastinal fibrosis may also be the cause.

B) The most common cause is lymphoma

C) Pallor and dry hair are seen on the face. 1


.
D) Salt restriction can be made (incomplete)

E)Radiotherapy and chemotherapy may be given


14- A 64-year-old female patient presents with progressive shortness of breath and hemoptysis
after a several-month history of progressive sinus congestion and nasal crusting. Findings in the
emergency department are as follows: Pulse oxygenation was 85% despite the high-flow nasal
cannula; White blood cell (WBC) count 15 K/ uL (normal: 3.7-11.0 K/ uL ); Platelets 511KuL
(normal: 150-400 K/ uL ); Creatinine 3.6 mg/ dL (normal: 0.58-0.96 mg/ dL ); TiT (++)
erythrocytes and casts are present. On physical examination, there are fine rales in widespread
lung areas . Chest x-ray showed bilateral patchy infiltrates . Diffuse alveolar bleeding on
bronchoscopy; bronchoalveolar lavage negative for infectious workup, including tuberculosis.
Test results are positive for P-ANCA, MPO. Which of the following renal immunohistochemistries
is most consistent with the suspected diagnosis?

A) Linear IgG staining positivity

B) Ig G,M and C1q/c3 positivity

C) Ig A positivity
%
O
D) Minimal immunofluorescent staining

E ) IgG positivity

15- Borrmann According to its classification , in which group are fungative lesions?

A) type 1

O
B) type 2

C)type3

D)type4

E) type5

16- Which of the following genes is associated with Peutz-Jeghers syndrome?

A) APC

B) =>
NGO11** STR1A
C) BRCA

D) Rb

E) VHL
17- Isn't it one of the findings of organ failure in acute pancreatitis?

A) Shock

B) Kidney failure

O-DIC
C) VERTICAL

D) Lung failure

E) GIS bleeding

18- Which of the following is incorrect information about tumor markers?

A) Tumor markers can be classified as enzymes, hormones, oncofetal antigens and receptors.

B) It is used for clinical staging of cancer and estimating tumor volume.

as a prognostic indicator of disease progression and to evaluate treatment success.

D) In case of cancer recurrence, the marker increases again.

O
E) Tumor marker values are not related to the effectiveness of treatment.

19-Which antihypertensive agent should be used first in a patient diagnosed with diabetic
nephropathy due to its nephroprotective effect?

A) Ca Channel Blocker

B) Furosemide

C) B Blocker

D) Johnnyvaptan

O
E) ARB***

20- Which treatment is used for FTL3 molecular structure in AML ?

A) Dasatinib

B) Imatinib

O
C) Glitenib

D) Panatinib

E) Nilotinib
21- The patient, who is being followed up with the diagnosis of extensive ulcerative colitis,
applies to the emergency clinic with the complaint of fever and abdominal pain while receiving
azathioprine and mesalazine treatment . Physical examination reveals hypotension, tachycardia,
abdominal tenderness, distension, and decreased bowel sounds. Rebound is not detected. What
is your most likely diagnosis for this patient? What would be your approach?

A) The probable diagnosis is colon perforation. Colonoscopy should be performed to confirm the
diagnosis.

O
B) Possible diagnosis toxic megacolon . To confirm this diagnosis, it would be good to have the
patient take an urgent CT scan or a standing direct abdominal radiograph to see that the
transverse colon diameter is more than 6cm.

C) Possible diagnosis toxic It is megacolon . There is no need for additional examination for
diagnosis. Physical examination is sufficient. It is necessary to undergo urgent surgery.

D) Possible diagnosis toxic megacolon . To confirm the diagnosis, the patient should have an
urgent colonoscopy.

E) Possible diagnosis toxic It is megacolon . After the diagnosis is confirmed with USG, diarrhea
should be reduced with antidiarrheal drugs.

22- Which of the following is NOT CORRECT about metabolic dysfunction in steatotic disease of
the liver ?

A) 2 or more metabolic risk factors criteria

B) Dysbiogenesis is involved in the pathophysiology

C) Lifestyle changes are important in treatment

D) Metformin shows beneficial results in histological researches

E) Noninvasive fibrosis markers ( ie . fib 4.... ) are in the algorithm

Answer :D

A 23-30 year old female patient comes to endocrinology with complaints of menstrual
irregularity, weight gain, pink- purple stretch marks on the skin, easy bruising and increased hair
growth. In the examinations, ACTH and cortisol levels are high. Which of the following is the
most likely situation?

A) Prolactinoma

O
B) Cushing's disease

C) Adrenal adenoma

D) Samatotroph adenoma

E) Iatrogenic cushing
has a defect in vitamin D 1 alpha hydroxylase ?

a)Hyperthyroidism

O
b)Renal failure

c) Hypogonadism

25- What is the main mechanism responsible for the pathogenesis of preeclampsia ?

O
A) Uteroplacental insufficiency ***

B) Proteinuria

C) Decreased angiotensin II receptor activity

D) Increase in antiangiogenic factors in circulation

E) Decrease in glomerular filtration rate

26- Which one of the following is wrong for acute tubular necrosis (ATN)?

A) ATN is the most common cause of acute kidney failure in hospitalization patients

B) ATN occurs due to sepsis, prolonged prerenal azotemia oath toxic agents

C) Dysmorphic erythrocyes oath erythtocte casts are classical signs of ATN

D) Proteinuria usually below the nephrotic range in ATN 1

O
E) Fractional sodium excretion is usually < 1.

27- Which one is not classical Hodgkin lymphoma?

A) Lymphocyte-rich type

B) Nodular sclerosing type

C) Mixed cellular type

D)-
Type lacking lymphocytes lymphocyte poor
OE) Lymphocyte predominant nodular type
28- Which of the following is not a cause of hereditary thrombophilia ?

A) Factor 5 Leiden mutation

B) Prothrombin gene mutation

O
C) factor 9 deficiency

D) Protein C deficiency

E) Protein S deficiency
29- Which of the following drugs is one of the drug groups that are frequently prescribed
inappropriately or unnecessarily in the elderly?

A) Oral antidiabetics

O
B) Sedatives

C) Statins

D) Antihypertensives

E) Iron supplements

is not one of the drugs used in acute esophageal variceal bleeding?

A) Carvedilol

B) Somatostatin Analogs

C) Serum Physiological

D) Antibiotic
?
O
E) Proton Pump Inhibitors

31- Which of the following is true regarding the etiology and epidemiology of cancer?

A) Colonoscopy to diagnose colon cancer is an example of primary prevention .

B) Genetic factors constitute a large percentage of cancer formation.

C) Prevalence is the number of new cases developing in the community at risk at a certain time.

OD) HHV-8 causes Kaposi's sarcoma.

E) The most important cancer modifiable risk factor is fiber in the diet.

32- Which of the following is true for familial pheochromocytoma ?

A) It occurs at a young age

B) Bilateral, multifocal located outside the adrenal

C) The frequency of recurrence is higher than the sporadic case.

D) The risk of malignancy is approximately 26-35%.

O
E) All
33- Which one is an IL-17 inhibitor?

A) Rituximab

O
B) Secukinumab

C) Tofacitinib

D) Tocilizumab

E) Infliximab

34- Which of the general approach principles to upper gastrointestinal system bleeding is
correctly stated?

A) Nasogastric or orogastric aspiration/ lavage is routine practice.

B) Second-look endoscopy is a mandatory part of the approach to non- varicose upper GI bleeding.

C) Helicobacter pylori Eradication is required in all patients.

D) Early (<12 hours) upper GI endoscopy following hemodynamic resuscitation should be


performed in every patient.

O
E) Hemodynamic stabilization and hemodynamics monitoring It is indispensable for
gastrointestinal bleeding control.

35- Which of the following is incorrect regarding diabetic ketoacidosis?

A) Primary anion gap acidosis is usually observed.

B) Plasma glucose level is above 300 mg/dl.

OC) Serum bicarbonate level is above 15 mEq /1.


D) Amylase and lipase levels may be elevated, not exceeding the upper normal limit by 2-3 times.

E) Blood pH is 7.30 or below 7.30

36- Which is incorrect for autoimmune hepatitis?

A) It is more common in women.

B) There is specific antibody positivity.

37- Which of the following is incorrect for chronic kidney disease (CKD) progression?

A) CKD progression rate increases with age.

B) The rate of progression in women is slower than in men.

O
C) Nephrotic level proteinuria reduces CKD progression.

D) Diabetes mellitus tight blood sugar control is effective in reducing CKD progression.

E) Hyperuricemia is a factor in CKD progression.


38- Which of the following is true for Polycythemia Vera?

A) JAK-2 mutation is usually negative.

B) Arterial oxygen saturation is low.

O
C) Erythrocyte Sedimentation Rate (ESR) is low.

D) Erythropoietin level increased.

E) BCR-ABL is positive.

39- Which is incorrect about TB intestinal involvement?

A) Causes iliocecal involvement

B) Treated like pulmonary tuberculosis

O
C) Lung involvement is seen in most

D) Causes abdominal pain and diarrhea

40- Which of the following is non Is it true about Hodgkin lymphoma?

A) The most common NHL in our country is follicular lymphoma.

B) It is not affected by autoimmune causes.

C)t(14,18) positivity is seen in mantle cell lymphoma.

D) It said something about T-cell lymphoblastic lymphoma.

41- Which of the following can be used to stimulate the abnormal viscerosematic reflex?

A)Peppermint oil

B) Microbiota modification

O
C) Pinaverium

D) Biofeedback

E) Oticeum ?
43- Which of the following is incorrect regarding the acute complications of diabetes?

A) The main cause of diabetic ketoacidosis is insulin deficiency in hyperosmolar hyperglycemic


state. is dehydration.

B) The anion gap is increased in diabetic ketosidosis .

C) Ketonuria is not expected in hyperosmolar hyperglycemic state .

O
D) The total fluid deficit in diabetic ketoacidosis is greater than in the hyperosmolar hyperglycemic
state.

E) Brain edema is a complication seen after the treatment of diabetic ketoacidosis and
hyperosmolar hyperglycemic state.

44- Which of the following is not a feature of autoimmune hepatitis?

A) It is more common in women.

B) Characteristic histological changes ( lymphoplasma interface hepatitis).

O
C ) ALP/GGT elevation.

D) Elevated IgG .

E) Presence of one or more characteristic autoantibodies .

45- is not one of the drugs used in acute esophageal variceal bleeding?

A) Carvedilol

B) Somatostatin Analogs

C) Serum Physiological

D) Antibiotic

OE) Proton Pump Inhibitors


46- Which is wrong for platelet suspension?

The options were so long that I couldn't keep them in mind, but they were related to transfusion
principles.

47- Euthyroidic thyroid of a patient with multinodular goiter Which is the suspicion of malignancy
on USG ?

A) Presence of halo

O
B) Hypoechogenicity

C) Pure cystic nodule

D) Low vascularity

E) Spongioform structure
48- which is a molecular target-directed treatment for FLT3+ acute myeloid leukemia?

A) Dosatinip

B) Imatinip

OC) Gliteritinip
D) Ponatinip

E) Nilotinip

49- Which of the following causes AL Amyloidosis?

A) FMF

B) Rheumatoid Arthritis

C) Ulcerative Colitis

O
D) Multiple Myeloma

E) Tuberculosis

50- Which of the following is wrong?

superior vena cava syndrome, edema on the face and arms, pleatorrhea and hoarseness may be
observed.

B) Loss of sensation in the extremities, loss of strength and sphincter dysfunction are the
symptoms of spinal cord compression.

O
C) Sharpening of the T wave and shortening of the QT interval are ECG findings of hyperkalaemia.

D) In hypercalcemia, dry skin, constipation and changes in consciousness may occur.

E) Hyperuricemia , hypocalcemia, hyperphosphatemia and hyperkalemia are the findings of tumor


lysis syndrome.

51- Which of the following is not an indication for uric acid treatment?

O
A) Uric acid level is 8mg/ dL

B) Tophi

C) Urate nephropathy

D) Radiographic findings of erosive gouty arthritis

E) History of nephrolithiasis
52 - Which of the following is not true for von willebrand disease ?

A) It is an autosomal inherited disease

B) Acquired von Willebrand disease can be seen in malignant oath autoimmune diseases

O
C) Bleeding time is normal****

D) The activated partial thromboplastin time ( aPTT ) may be long

E) Both primary and secondary hemostasis are affected

53- A 23-year-old male patient applies to the clinic due to masculine dysfunction. There is
increased facial hair growth and gynecomastia. Additionally, during palpation, the testicles are
palpated as small and hard . What do you think?

A) Kallman syndrome

O
B) Klinifelter syndrome

C) Androgenic deficiency

D)Turner syndrome

E) Germinoma

54- Which of the following is a laboratory finding seen in chronic kidney disease-high turnover
bone disease ?

A) FGF-23 decreases, klotho decreases, parathormone increases, phosphorus increases, calcitriol


decreases

B) FGF-23 increases, klotho decreases, parathormone increases, phosphorus increases, calcitriol


increases

C) FGF-23 increases, klotho increases, parathormone increases, phosphorus increases, calcitriol


increases

D) FGF-23 decreases, klotho increases, parathormone increases, phosphorus increases, calcitriol


increases

O
E) FGF-23 increases, klotho decreases, parathormone increases, phosphorus increases, calcitriol
decreases

55- Which of the following information about delirium is incorrect?

A) Acetylcholine may play a role in its pathophysiology

B) Wavy course and lack of attention are significant for the diagnosis of delirium.

C) Pain and emotional stress can lead to delirium

OD) Hyperactive type delirium is most commonly observed

E) Antipsychotics can be used in the symptomatic treatment of delirium.


A 56- to 61-year-old woman was referred by her general practitioner with an 8-year history of
Raynoud syndrome. He described several incidents where ischemic ulcers appeared on his feet.
She smoked 15 cigarettes a day and was using combined birth control pills. It was seen by the
registrar who noticed the fingers with distal pulp atrophy. His fingers and toes were cold and
there was a dark blue discoloration . Examinations were arranged and he was advised to quit
smoking. ENA was positive for anticentromere antibody. Lung function tests showed a 72%
reduction in DLCO . Chest CT scan and ECG were normal. What is the most likely diagnosis?

A) Diffuse scleoderma

O
B) Limited scleoderma

C) Lupus

D) Morphea

E) Sjogren's

57 - Which one of the following is not a risk factor for contrast related nephropathy ?

OA) Diabetes insipidus

B) A history of chronicle kidney disease

C) Dehydration

D) High volume radiocontrast agent

E) Intra-arterial application of contrast agent

58- Which of the following is the definition of Budd-Chiari syndrome?

A) It is the expansion of the hepatic sinusoidal lumen without blockage or infiltration by abnormal
cells or materials.

B) Thrombus formation in the portal vein and/or its right or left branches.

O
C) Hepatic venous flow from small hepatic venules It is defined as an obstruction of the inferior
vena cava anywhere from its entrance to the right atrium (regardless of the mechanism of the
obstruction).

D) It is a condition characterized by congestion as a result of the destruction of endothelial cells in


the sinusoids , especially in the central part of the hepatic lobule , and focal obstruction of the
sinusoid lumen .

E) Hepatic venous flow is obstructed due to cardiac or pericardial diseases or sinusoidal


obstruction syndrome.
59- Which of the following is not true about peptic ulcer, functional dyspepsia and alarm
markers?

anamnesis and physical examination are first performed.

B) The main risk factors of peptic ulcer are H.pylori and NSAIDs . !
C) The most common cause of dyspepsia in the Gastroenterology department is Functional
dyspepsia.

D) It includes the criteria of functional dyspepsia alarm features.

E) A 70-year-old male patient presents with epigastric pain and weight loss. Upper GI endoscopy
should be performed.

60- Which of the following cytotoxic agents causes pulmonary fibrosis?

A) Vincristine

O
B) Bleomycin

C) Doxorubicin

D) 5 FU

E) Cisplatin

61-76 year old female patient, there is a 1.1.2 compressive area in the sacral area. How should
the patient be approached?

A) The bed should be at a 30 degree angle and a wedge pillow should be used.

O B) Air mattress should be used

C) Its position should be changed every 2 hours

D) If there is a wound elsewhere, cream should be used.

62- The patient will be transfused with erythrocyte suspension. The antibody screening test, one
of the compatibility tests before transfusion, was found positive. Which procedure should be
done next?

A) Cross match should be made and then the patient should be transfused.

B) After the direct Coombs test is performed, the patient should be transfused.

C) The patient's ABO blood group should be checked again and then the patient should be
transfused.

O
D) Antibody identification should be made and the antibody type should be determined.

E) After the indirect Coombs test is performed, the patient should be transfused.
63- A 36-year-old female patient has been followed up with the diagnosis of Seropositive
Rheumatoid Arthritis for 6 years. Her wrists, proximal The treatment of the patient, who has
swelling and tenderness in the interphlangeal joints that has been going on for 4 months , is
methotrexate 15 mg / week, hydroxychloroquine 200 mg 2 * 1, salazopryne 500 mg tablet 3 * 2
and deltacortil 7.5 mg daily. The patient's sedim: 56 CRP: 40 DAS 28 score is 6.7. is being
calculated. The patient has had previous Which one should be used in the next stage of
treatment for a patient with pulmonary embolism?

A) Etanercept

O
?
B) Upadacitinip

C) Cyclophosphamide

D) Azathioprine

E) Lymflunomide

64- Behçetin Which of the following is incorrect regarding gastrointestinal system involvement?

A) Abdominal pain and diarrhea are the most common symptoms.

B) Deep ulcers are seen on colonoscopy

C) Ileocecal involvement is seen

O
D) Stricture and perianal disease are common

E) Serious bleeding and perforation may occur

65- Which one is not one of the Spondyloarthritis ASAS diagnostic criteria?

A) Good response to NSAIDs

B) Enthesitis

C) Dactylitis

O
D) Sikka findings

E) Family history
66- Which of the following statements about biochemical, function tests and metabolic diseases
of the liver is not true?

A) The first step in evaluating a mildly elevated serum aminotransferase level is to perform repeat
tests to confirm the persistence of the elevated value.

Low serum albumin and prolonged prothrombin time (PT) indicate chronic liver parenchymal
disease (cirrhosis).

C) D- penicillamine , trientine and zinc are used for maintenance in the treatment of patients with
hemochromatosis .

D) Wilson disease is an autosomal recessive disease with the defective gene ATP7B on
chromosome 13.

O
E) Macrovesicular fat accumulation and Mallory equivalent bodies in liver biopsy indicate
nonalcoholic fatty liver disease (NASH).

67- is a biomarker for Immune checkpoint inhibitors ?

A)EGFR mutation

B)EML4/ALK translocation

O
C)PD-L1 level

D)Met mutation

E) Ros mutation

68- Which of the following is not one of the parameters used in MASCC scoring in a patient
diagnosed with febrile neutropenia?

A) Performance status

B) Age

O
C) Presence of port catheter

D) Presence of hematological malignancy

E) Hypotension

69 - Which of the following is not a sign of sjogrens syndrome ?

A) Keratoconjunctivitis

B) Peripheral oath central neuronal system involvement

C) Reynauds phenomenon

O
D) Erosive arthritis

E) Bilateral parotid gland swelling


70- Which of the following is not an autoinflammatory disease?

O
A) Fibromyalgia

B) Cryoprinopathies

C) Familial Mediterranean fever

D)TNF receptor related periodic syndrome

E)PFAPA

71- Which is wrong in the complete pituitary incision in a 30-year-old female patient ?

A) Low IGF 1 level in the patient

O
B) Corticotroph hypofunction is vital

C) Galactorrhea is observed due to hyperprolactinemia

D) In its treatment, thyroid hormone is given first.

E) Cyclic estrogen and progesterone tablets are started for menstruation.

72- Which one inhibits VEGF receptor 2 ?

A) Ramicirumab

B) Trastuzumab

C) Bevacizumab

D) Aflibercept

O
E) Pazopanib

73- the hereditary disease characterized by a defect in the structure of Type IV collagen, the
basic component of the glomerular basement membrane ?

A) Alport Syndrome

74- Isn't severe plastic anemia a parameter?

A)Bone marrow cellularity <25%

O
B) PNH clone in FLAER test

C)Absolute reticulocyte <40,000

D)Absolute neutrophil<500

E) Platelets<25,000
75- Proteinuria and hematuria are found in urine analysis in a patient with serum Anti-GBM
antibodies detected . Which of the following is the most likely diagnosis?

A) Minimal change disease

O
B) Rapidly developing glomerulonephritis

C) Interstitial nephritis

D) Focal glomerulosclerosis

E) IgA nephropathy

76- Which of the following is incorrect for gastroesophageal reflux disease?

O
A) Endoscopy is required for diagnosis.

B) It may be related to dental erosions.

C) It may be related to asthma.

D) Obesity is a risk factor.

E) The esophagus may appear normal on endoscopy.

77- A 57-year-old woman was admitted to the emergency room with 10-day history of epigastric
pain oath fever _ physical examination revealed jaundice oath right upper quadrant tenderness
with superficial palpation _ vital signs were normal. Laboratory studies ; total bilirubin 7.0 mg/ dL
, direct bilirubin 4.8 mg/ dL . Alkaline phosphatase 680 U/L (25-100 U/L) and GGT 121 U/L (10
-50 U/L), ALT 145 U/L, AST 92U/L. Albumin and INR-PTT were normal. Ultrasonography of the
abdomen showed biliary duct dilatation oath hyperecogenic even duct stones _ Which of the
following is the most appropriate next step in the management of this patient ?

A) Upper gastrointestinal endoscopy

B) Computed Tomography

C) Biochemical (viral and metabolic studies _ for liver parenchymal disease

O
D) Endoscopic retrograde cholangiopancreatography

E) Endoscopic ultrasonography

Answer :D
78- Which of the following findings in a patient who applies to the outpatient clinic with the
complaint of drinking too much water and urinating nearly 5 liters a day is primarily considered
Nephrotic Diabetes? Does it suggest insipidus ?

A) The patient has a history of head trauma

B) Decrease in the amount of urine during periods when the patient is asleep

C) Low plasma ADH level

O
D) The patient is using lithium

E) When ADH is given externally to the patient, urine osmolality increases.

79- What do you do for the differential diagnosis between high corrected Ca value and high
parathormone ?

O
A)24-hour urine calcium

B) CaSR mutation

C)1,25(OH)2D

80- Which is not a cause of hereditary thrombophilia ?

A) Factor 5 leiden mutation

O
B) Prothrombin gene mutation

C) Protein c deficiency

D) Protein s deficiency

E) Factor 9 deficiency

81- Which of the following is not an indolent lymphoma?

A) Marginal zone Lymphoma

⑧B) Mantle cell lymphoma

C) Burkitt lymphoma

D) MALT LT type lymphoma

E) Follicular Lymphoma
83- Which of the following is true?

A) Bilirubin levels are very high in acetaminophen intoxication.

B) Serum tranaminases are usually normal in ischemic hepatitis damage.

C) In acute hepatitis B, the AST/ALT ratio is above 2.

D) For the diagnosis of acute hepatitis C, Ig alone and anti-HCV are sufficient.

OE) Anti- Hbc IgM is used in the diagnosis of Acute Hepatitis B.

84- Which of the following is wrong?

A) In pericardial tamponade, low voltage and filiform pulse are seen on the ECG.

B) Loss of sensation, strength and sphincter dysfunction in the extremities suggest spinal cord
compression.

Hypocalcemia, hyperuricemia , hyperkalemia and hyperphosphatemia are observed in tumor lysis


syndrome .

D) In hypercalcemia, dry skin, constipation and altered state of consciousness are observed.


E) Sharpening of the T wave and shortening of the QT interval are ECG findings of hyperkalemia.

85- Which of the following is the most common cause of endogenous Cushing's Syndrome?

A)Adrenal adenoma

B) Ectopic ACTH-secreting tumors

C) Pituitary adenoma

D)Adrenal carcinoma

E)None

86- Which of the following is incorrect regarding oral antidiabetic drugs?

A) DPP-4 inhibitors should not be preferred in patients with congestive heart failure.

B) Medullary GLP-1 agonists should not be preferred in patients with thyroid cancer.

O
C) Metformin should not be preferred in those with a history of lactic acidosis.

D) Weight loss may occur in those using SGLT-2 inhibitors.

E) Alpha glucosidosis inhibitors reduce carbohydrate absorption .


87- Which one does not cause hepatocellular cancer on the basis of cirrhosis?

OA) Hepatitis B
B) Hepatitis C

88- Which of the following is Is it wrong for willebrand ?

A) It is the most common hereditary disease

B) PFA-100 test has been extended

OC) Prothombin time is extended

D) Factor 8 deficiency

E) Earned Von Willebrand disease may occur

89- Which one is wrong about afaresis ?

A) Adsorption It is an afaresis technique in which bioactive membrane is used .

O
B) Apharesis is ordered from the inside to the outside, starting from the erythrocytes .

C) Hypocalcemia due to citrate toxicity may be observed.

D) In stem cell mobilization Gcsf is used .

E) Granulocyte Afaresis is used in the presence of severe infection.

90- Which of the following causes hepatocellular cancer without causing cirrhosis?

A) Alcohol.

B) Non- alcoholic fatty liver.

O
C) Hepatitis B virus.

D) Hepatitis C virus.

E) Wilson's disease.

91- Which of the following is incorrect for chronic lymphocyte leukemia?

A) Lymphadenopathy

B) Anemia

C) Direct Coombs Positivity

O
D) M Spike in protein electrophoresis

E) Bone Lytic Lesion (answer)


92- Which of the following is incorrect regarding acute infectious thyroiditis?

A) Infection may develop through hematology or neighborhood

O
B) It is a thyroid infection usually caused by viral agents.

C) Thyroid in physical examination gland is enlarged and there are signs of inflammation

D) Leukocytosis, CRP and sedimentation are increased in the laboratory

E) The area involved in thyroid scintigraphy is hypoechoic .

93- Which of the following is not hypoproloferactive anemia?

A) Chronic damage anemia

O
B) Hot antibody auto immune anemia

C) Iron deficiency anemia

D) Aplastic anemia

E) Megaloblastic anemia

94- Which does not cause hepatitis viruses?

A) Cholangiocellular carcinoma

B) Hepatocellular carcinoma

O
C) Hemangioma

D) Chronic hepatitis

E) Cirrhosis

95- A 56-year-old female patient with locally advanced home left breast cancer underwent left
modified radical mastectomy surgery following 4 cycles of systemic combined chemotherapy
before curative surgery. For what purpose was the chemotherapy given to the patient before the
surgery? (This was not the question, it was a similar question.)

A) Palliative chemotherapy

B) Adjuvant chemotherapy

O
C) Neoadjuvant chemotherapy

D) Maintenance chemotherapy

E) Consolidation chemotherapy
96- In which of the following causes of thyrotoxicosis is RAI uptake increased?

A) Subacute thyroiditis

B) Medicine 1 Amiodarone

O

C) Type 2 Amiodarone thyrotoxicosis

D) Silent thyroiditis

E) Struma ovarii

97- 67-year-old female patient, does not have any disease other than hypertension, uses
lercanipidine 10 mg for hypertension, 6/6 in Katz test, 8/8 in Lawton-Brody , 12/14 in mini
nutritional test, geriatric depression 3/15, mini mental test takes 30/30. Which of the following is
true for this patient?

?
I) Pneumococcal and influenza vaccination should be administered

II) Annual dexa is recommended even if bone mineral density does not decrease

III) its functionality and life expectancy are not good, it is not suitable for routine screening

IV) Colorectal cancer screening is recommended if it has never been done before

98- causes normal anion gap metabolic acidosis?

A) Lactic acidosis

B) Diabetic ketoacidosis

O
C) Uremic acidosis

d) Salicylate intoxication cation

e) Tuberculosis

99- Which of the following is not used in the treatment of Autosomal Dominant Polycystic Kidney
Disease?

A) Sirolimus

B) V2 receptor antagonists

C) Somatostatin analogues

D) Everolimus

pE) Vasopressin agonists are the answer


100- Which of the drugs used in the treatment of obesity prevents the hydrolysis of fat in the
small intestine by binding gastrointestinal lipase?

A) Pramlintide

O
B) Orlistat . this is the answer

C) Lorcaserin

D) Bupropion

E) Topiramate

DAHİLİYE T2 TEORİK SINAVI

1. Which of the following does not cause prerenal acute renal failure?

a) Vomiting

b) Hemorrhagic shock

c) Acute interstitial Nephritis

d) Septic Shock

e) Acute Pancreatitis

2. Which of the following statements about renal involvement in Henoch-Schönlein Purpura is


incorrect?

a) Nephrotic syndrome may be seen in 20-30% of cases

b) It most often occurs clinically as an asymptomatic urinary anomaly.

c) Renal involvement is usually temporary.

d) Diffuse mesangial IgM deposition is a typical finding.

e) Acute kidney injury may occur secondary to crescentic glomerulonephritis.


3. Which of the following accelerates the progression of renal failure in ADPKD (Autosomal
dominant polycystic kidney disease)?

a) Female gender

b) Increased HDL level

c) First hematuria attack before the age of 35

d) White race

e) Diagnosis of ADPKD after the age of 35

4. Which of the following is a laboratory finding seen in chronic kidney disease-high turnover bone
disease?

a) FGF-23 increases, klotho increases, parathormone increases, phosphorus increases, calcitriol


increases

b) FGF-23 decreases, klotho increases, parathormone increases, phosphorus increases, calcitriol


increases

c) FGF-23 increases, klotho decreases, parathormone increases, phosphorus increases, calcitriol


decreases

d) FGF-23 decreases, klotho decreases, parathormone increases, phosphorus increases, calcitriol


decreases

e) FGF-23 increases, klotho decreases, parathormone increases, phosphorus increases, calcitriol


increases

5. A 62-year-old female patient, who has been on hemodialysis 3 days a week for 10 years with the
diagnosis of chronic renal failure, is brought to the emergency room due to poor general condition.
In the arterial blood gas taken from the patient: PCO2: 16 mm Hg, HCO3: 4 mmol/l and pH 7.08.
What is the underlying acid-base disorder?

a) Uncompensated metabolic acidosis

b) Compensated metabolic acidosis

c) Uncompensated metabolic alkalosis

d) Compensated respiratory acidosis

e) Compensated metabolic alkalosis


6. Which of the following is not a contrast nephropathy (CN) risk factor?

a) Using high volume contrast is an important risk factor for CN.

b) Intravenous administration of contrast material is higher risk than intra-arterial administration.

c) Repeating the procedure within the first 72 hours is an important risk factor for CN.

d) Using contrast at high osmolar load is more risky in terms of CN than using contrast at
isosmolar load.

e) Intravascular volume deficiency is one of the most important risk factors for CN.

7. Which of the following is not one of the clinical and laboratory findings expected in chronic
kidney disease?

a) Malnutrition

b) Hypertension

c) Hyperkalemia

d) In insulin renal clearance

e) Erythropoietin deficiency

8. Which of the following is the mechanism of sodium removal by hemodialysis?

a) Diffusion

b) Convection

c) Convection and diffusion

d) Osmosis

e) Ultrafiltration

9. Which of the following is not a clinical sign of hypokalemia?

a) Arrhythmias

b) Resistant hypertension

c) Rhabdomyolysis

d) Insulin increase

e) Metabolic alkalosis
10. Which of the following does not have a high risk of gestational hypertension?

a) Fourth pregnancy

b) Multifetal pregnancy

c) The mother is obese

d) Maternity Age <18 years old

e) The mother has chronic kidney disease

11. A 65-year-old male patient presents with complaints of nausea and fatigue, significant swelling
in his ankle, and blood in his urine. On examination, he has bilateral pretibial edema and his blood
pressure is high. Laboratory evaluation: creatinine level in the last two weeks It shows he's
doubled in size and he's anemic. Anti-GBM antibodies are detected. Urinalysis shows proteinuria
and hematuria. What is the most likely diagnosis?

a) Rapidly Progressive Glomerulonephritis

b) Interstitial Nephritis

c) Minimal change disease

d) IgA nephropathy

e) Focal glomerulosclerosis

12. Whenever a patient has an upper respiratory tract infection or gastroenteritis, blood in his urine
is noticed within a few days. Which of the following is the most likely histopathological finding in
line with the possible diagnosis?

a)Mesangial proliferation with significant IgA accumulation

b) Accumulation of complement accumulation

c)Podocyte loss

d) Deposition of immunoglobulins and complement along the basement membrane

e)Focal segmental glomerulosclerosis


13. A 27-year-old woman is 30 weeks pregnant and applies for routine follow-up. Her blood
pressure is 150/100. It is learned that he was previously normotensive. Urine analysis: 1+
proteinuria with density 1.020 SG. Serum Uric acid level is 8. Platelets and liver function tests are
normal. There is 1.1 grams of protein in the 24-hour urine analysis. Antihypertensive treatment is
planned for the patient. What is a first-line agent?

a) Captopril

b) Furosemide

c) MgSO4 (IV)

d)Methyldopa

e) Hydrochlorothiazide

14. A 56-year-old female patient with locally advanced left breast cancer was treated with a 4-cycle
system and combined chemotherapy followed by left modified radical surgery before curative
surgery. mastectomy surgery was performed. For what purpose was the chemotherapy given to
the patient before the surgery?

a) Palliative chemotherapy

b) Adjuvant chemotherapy

c) Neoadjuvant chemotherapy

d) maintenance chemotherapy

e) Consolidation chemotherapy

15.Which of the following cytotoxic agents can cause morbidity and mortality by causing
pulmonary fibrosis?
a) bleomycin
b) vincristine
c) doxorubicin
d) 5-FU
e) cisplatin

16.Which of the following is incorrect for PARP inhibitors?


a) It inhibits the PARP enzyme, which is responsible for DNA single-strand break repair.
b) It is more effective in tumors without BRCA mutation than in tumors with it.
c) It is approved for use in tumors such as breast, ovarian, parkreas and prostate cancer.
d) Hematological side effects are frequently observed.
e) Olaparib, Niraparib, Veliparib, Talazoparib, Rucaparib are PARP enzyme inhibitors used today.
17. Which of the following is incorrect?
a) Exposure to pesticides and nitrous compounds may lead to the development of cancer,
b) HTLV-8 virus can cause Kaposi's sarcoma.
c) Screening with colonoscopy in the community for early diagnosis of colon cancer is a priority
for the prevention of primary cancer.
d) Today, the main source of radiation to which human beings are exposed is the procedures
performed for examination and treatment purposes.
e) Smoking is one of the modifiable risk factors that cause cancer.

18.Which of the following is true for oncological emergencies?


a) Sharpening of the T wave and shortening of the QT interval are ECG findings of hyperkalemia.
b) In tumor lysis syndrome, hyperkalemia, hypercalcemia, hyperphosphatemia and hyperuricemia
are seen at various levels.
c) High voltage findings may appear on the ECG in malignant pericardial tamponade.
d) Patients with increased intracranial pressure syndrome (IPR) often present with nausea,
vomiting and headache.
e) The most common cause of Vena cava superior syndrome (VCSS) is goiter.

19.When a 55-year-old male patient, who was receiving chemotherapy for lung cancer, presented
to the emergency room with confusion, his blood biochemistry values were Na 136 mEq/L, K 4.2
mEq/L, Ca 9.1 mg/dL, Urea 35 mg/Dl, creatinine 0. It was determined as .8 mg/dL and albumin as
2.2 g/dL. In addition, the patient's mouth and skin appeared dry.
Which of the following is not true for this patient?
a) In addition to confusion, headache and constipation may be accompanying symptoms.
b)Treatment should be started with diuretics simultaneously with hydration.
c) The most suitable liquid to be preferred for hydration is 0.9% NaCl.
d) The patient's corrected calcium value is 10.5 mg/dL
e)Zolendronic acid should be given for long-term control after hydration.

20.Which of the following does not induce angiogenesis?


A) Vitamin D
B) COX2
C) Nitric Oxide
D) Metalloproteinase
E) Incretin

21.55-year-old female patient applies for screening tests without any complaints. Which
organ-system screenings should not be performed?
a)Mammography
b) Colonoscopy
c)Sigmoidoscopy
d)Thorax and abdominal tomography
e)Cervical smear (pap smear)
22.Which of the following findings are correct for the definition of neutropenic fever. I. A single
temperature reading above 38.3 degrees. II. Detection of absolute neutrophil count below 500 III.
Presence of port catheter IV. Hospitalization history
a) I and II
b) II and III
c) III and IV
d) I and IV
e) None

23.The family physician refers a 46-year-old patient who has been complaining of bruising on his
fingers for the last three months to the rheumatology service. It is learned that the patient was
diagnosed with reflux a year ago and used omeprazole. On physical examination, swelling in the
soft tissue of both fingers and skin edema in the arms and legs are detected. Lab tests are normal,
1/640 ANA is positive. What is your possible diagnosis?
a) morphea
b) diffuse scleroderma
c) limited scleroderma
d) SLE
e)eosinophilic fasciitis

24.A thirty-year-old female patient has difficulty getting out of bed in the morning. He could only
relax after 1-2 hours. There was pain and swelling in the wrists and bilateral elbows. There is
swelling in the knees. Physical examination reveals a 1x1 cm mobile lesion under the skin on the
olecranon. Which of the following tests should be requested from this patient for diagnosis?
a) Complete blood count – ANA
b)Rheumatoid factor – MAIN
c) Rheumatoid factor – Anti DsDNA
d) Rheumatoid factor – Radiological examination
e)ANA – Anti Ds DNA

25.A 19-year-old male patient is being evaluated for shortness of breath, hemoptysis and fever for
five days. The patient was diagnosed with Class IV lupus nephritis 6 years ago and received
cyclophosphamide and pulse steroid treatment, and is currently taking 10 mg prednisolone and
hydroxychloroquine. On physical examination, blood pressure is 170/100 mmHg, oxygen
saturation is 88%, and diffuse coarse rales are heard in bilateral lung fields. And edema is detected
in bilateral legs. In laboratory tests, the glomerular filtration rate is 22ml/min, 24-hour Protein
excretion in urine is 2000 mg/day, c3 and c4 are low and anti-ds DNA is high. The patient's DLCO
test result is found to be high. Which of the following is the most likely diagnosis in this patient?
a)Pulmonary Embolism
b)Acute respiratory distress syndrome
c)Shrinking lung syndrome
d)Diffuse alveolar hemorrhage
e)Acute lupus pneumonia
26. In a 30-year-old patient presenting with weakness, muscle pain, and pain in the hands, the
blood pressure measured in the right arm is 140/80 and the blood pressure measured in the left
arm is 80/40. The pulses are detected as weak on the left side. What is the most likely diagnosis
in this patient?
a) aortic coarctation
b) takayasu arteritis
c)aortic dissection
d)giant cell arteritis
e)aortic arch syndrome

27.Which of the following is seen in adult Still's disease?


A) Thrombocytopenia
B) LAP is frequently seen.
C) Triglyceride level is high.
D) Fever is constantly high.
E) Glycosized ferritin level is high.

28.Which of the following is not an absolute indication for starting uric acid-lowering treatment in
gout?
a) Presence of urolithiasis
b) Having 2 or more gout attacks per year
c) Presence of tophus
d) Presence of erosion findings on direct radiography
e) Uric acid level is over 10 mg/dl

29. A sixty-year-old female patient applies to the outpatient clinic with complaints of dry mouth,
dry eyes and rash on the legs that have been going on for more than three months. On physical
examination, there is bilateral parotid swelling and purpuric rashes on both lower extremities.
What is the patient's possible diagnosis?
a) Rheumatoid arthritis
c) Behcet's Disease
b) Systemic Lupus erythematosus
d) Sjögren's Syndrome
e) Henosch schonlein purpura

30. Which of the following statements about the approach to arthritis is incorrect?
a) In inflammatory arthritis; synovial fluid is purulent, its viscosity is markedly reduced, and the
leukocyte count is around 100,000 (mostly polymorphonuclear leukocytes).
b) Arthrocentesis must be performed to distinguish septic arthritis in patients presenting with
monoarthritis.
c) If the synovial fluid obtained by arthrocentesis is hemorrhagic, trauma, villonodular hemophilia
or iatrogenic causes can be considered in the etiology.
d)Normal synovial fluid is clear, colorless, viscous and contains a small number of cells.
e) In septic arthritis; The most commonly affected joint in adults is the knee.
31.Which of the following is not one of the mucocutaneous lesions of Behçet's disease?
a) Oral aphthous
b) Genital aphtha
d) Erythema nodosum
c) Papulopustular eruptions
e) Heliotrope debris

32. Which of the following drugs used in the treatment of rheumatoid arthritis shows its effect by
blocking TNF-alpha?
a) Adalimumab
b) Rituximab
c) Tocilizumab
d) Abatacept
e) Tofacitinib

33. Which of the following is not one of the expected clinical findings of primary antiphospholipid
syndrome?
a) Venous thrombosis
d) Recurrent pregnancy-related morbidities (preeclampsia, eclampsia or premature birth)
c) Arthritis
d) Recurrent first trimester miscarriages
e) Arterial thromboses

34. Which of the following diseases is not an autoinflammatory disease?


a) Familial Mediterranean fever
b) Hyper IgD syndrome
c) Antisynthetase syndrome
d) TNF receptor-related periodic syndrome (TRAPS)
e) Cryoprinopathies

35. Which of the following describes the term "VUS", which can be frequently reported as a result
of testing in familial cancer syndromes?
a) Positive result
b) Uninformative negative result
c) Result with unknown meaning
d) Negative result
e) Strong positive result

36. Which of the following causes of thyrotoxicosis involves increased radioiodine uptake?
a) Subacute thyroiditis
b) Silent thyroiditis
c) Amiodarone-induced thyroiditis type 2
d) Graves' disease
e) Iotrogenic thyrotoxicosis
37. Which of the following drugs used in the treatment of obesity is effective as an opioid
antagonist and dopamine and adrenaline reabsorption inhibitor?
a) Orlistat
b)Phentermine/Topiramate
c)Naltrexone/Bupropion
d)Lorcaserin
e) Liraglutide

38.A 23-year-old male patient is referred to the endocrine clinic due to erectile dysfunction. It
describes decreased sexual desire. On physical examination, his height was 187 cm. He had
sparse facial hair and gynecomastia. On palpation, the testicles were small and hard. Which of the
following is the most likely diagnosis for this patient?
a) Kallman syndrome
b) Klinefelter syndrome
c) Complete androgen insensitivity syndrome
d) Craniophrangioma
e) 5 alpha reductase deficiency

39. A 41-year-old woman is referred to the endocrine clinic due to a gradual onset of headache and
high blood pressure. There is no significant disease history. On examination, his blood pressure is
measured as 175/110 mmHg. Laboratory and radiology findings: Na 144 mmol/L (135-145) K 4.4
mmol/L (3.5-5.5) Urea 7.0 mg/dL (5-9) Creatinine 90 μmol/L (60-115) Aldosterone 18 ng/dL
Plasma renin activity (PRA 0.6) ng/mL/h Abdomen CT: 1.7 cm adrenal mass in the right adrenal
gland Which of the following is the most appropriate next step for this patient?
a) Verification test
b) Genetic testing
c) Ultrasound-guided biopsy
d) Surgery (right adrenalectomy)
e) Starting a beta blocker

40.Which enzyme defect is the most common cause of Congenital Adrenal Hyperplasia?
a) 11 Beta Hydroxylase deficiency
b) 21 Hydroxylase deficiency
c) 17 Alpha Hydroxylase deficiency
d) 3 Beta- Hydroxysteroid Dehydrogenase deficiency
e) None

41. Which of the following is usually the first symptom and the most common symptom of
Cushing's Syndrome?
a) Abdominal Striae
b) Hirsutism
c) Centripedal Obesity
d) Renal stone
e) Hyperlipidemia
42. In which of the following situations is there a problem in the 25 hydroxylation phase of vitamin
D synthesis?
a) Vitamin D-dependent Type 2 rickets
b) Hypoparathyroidism
c) Use of anticonvulsants
d) Nephrotic syndrome
e) Renal failure

43. Which of the following features is considered to be in favor of Type 2 Diabetes Mellitus in
distinguishing between Type 1 and Type 2 diabetes?
a) Age <30
b) AntiGAD positivity
c) Heavy burden of diabetes in the family
d) Ketosis
e) Acute symptomatic onset

44. Which of the following shows the biochemical parameters of a Primary Hyperparathyroidism
patient with moderate hypercalcemia? (Reference range for Serum Calcium: 8.5-10.5 mg/dl,
Reference range for Serum Parathormone (PTH): 20-65 pg/ml)
a) Serum Calcium: 11 PTH: 85 Serum Albumin: 5 g/dl Fractionated Urinary Calcium Percentage:
1.5%
b) Serum Calcium: 11.9 PTH: 180 Serum Albumin: 3.4 g/dl Fractionated Urinary Calcium
Percentage: 2%
c) Serum Calcium: 11.5 PTH: 75 Serum Albumin: 3.9 Fractionated Urinary Calcium Percentage:
0.9%
d) Serum Calcium: 10.8 PTH: 100 Serum Albumin: 4 Fractionated Urinary Calcium Percentage: 2%
e) Serum Calcium: 12 PTH: 15 Serum Albumin: 3.9 Fractionated Urinary Calcium Percentage: 1.5%

45. Which of the changes in the osteoporosis development process is mainly a pathophysiological
change due to aging?
a) Decrease in the synthesis ability of osteoblasts
b) Decrease in serum estrogen level
c) Increase in IL-1, IL-2 and TNF levels
d) Increase in osteoclast activity
e) Increase in RANKL and RANK expression

46. Which of the following is incorrect regarding postpartum thyroiditis?


a) It is associated with autoimmunity.
b) In the hyperthyroid stage, free T3, free T4, thyroglobulin are increased, and TSH is suppressed.
c) Sedimentation is high.
d) Radioactive iodine uptake decreased in the hyperthyroidism phase.
e) The risk of postpartum thyroiditis is increased in female patients with anti-thyroid peroxidase
antibody (anti-TPO) positive
47. Regarding the clinical findings of hypothyroidism, which of the following is not expected in a
patient with hypothyroidism?
a) Dryness and paleness of the skin
b) Increase in pulse pressure
c) Diastolic hypertension
d) Increase in gallstone formation
e) Increased risk of fracture

48. Which of the following is incorrect regarding oral antidiabetic drugs?


a) Alpha glucosidase inhibitors reduce carbohydrate absorption
b) Metformin should not be preferred in those with a history of lactic acidosis.
c) Weight gain may be observed in those using SGLT-2 inhibitors
d) Thiazolidinediones should not be preferred in patients with congestive heart failure.
e) GLP-1 agonists should not be preferred in patients with medullary thyroid cancer

49.Which of the following is not a possible complication of Diabetic ketoacidosis and


Hyperosmolar hyperglycemic condition?
a) Fluid overload
b) Cerebral edema
c) Hyperkalemia
d) Pulmonary embolism
e) Hypoglycemia

50. Which of the following is not an indication for insulin treatment?


a) Type 1 diabetes mellitus
b) Severe hyperglycemic symptoms in patients with type 2 diabetes mellitus
c) Hyperglycemic emergencies
d) Severe liver failure
e) Excessive weight gain

51. Which of the following information is incorrect regarding thyroid diseases?


a) Iodine deficiency is the most common cause of endemic and sporadic goiter.
b) The most common cause of primary hypothyroidism in iodine-sufficient regions is Hashimoto's
thyroiditis.
c) Use of lithium, iodine and amiodarone may cause hypothyroidism
d) Medullary thyroid carcinoma develops from the parafollicular C cells of the thyroid
e) Serum calcitonin level is used as a tumor marker in papillary thyroid cancer.

52. Which of the following statements about Gestational Diabetes Insipidus is incorrect?
a) Frequent urination and dry mouth are the most obvious symptoms in pregnant women.
b) The growing uterus that puts pressure on the bladder aggravates the clinical picture.
c) Placental vasopressinase enzyme activity decreased.
d) It often occurs in the third trimester of pregnancy.
e) The clinical picture improves after birth.
53. A patient with a macroadenoma detected in the pituitary develops sudden vision loss,
hypotension and coma-shock symptoms. Which of the following could be the cause of this clinical
picture?
a) The tumor is malignant
b) Pituitary apoplexy
c) Invasion into the cavernous sinus
d) Development of inappropriate ADH syndrome
e) Infrasellar spread of the tumor

54. Which of the following is not an indication for surgical treatment at the time of initial diagnosis
in a patient with macroprolactinoma?
a) Fertility desire
b) Presence of CSF rhinorrhea
c) There is a risk of pituitary apoplexy
d) There is a risk of permanent blindness
e) Prolactin value is too high

55.What is the most common complaint of a patient diagnosed with chronic pancreatitis?
a) Abdominal pain
b) Jaundice
c) Fatty defecation
d) Weakness and weakness
e) Gas and bloating

56. Which of the following is not most likely to be associated with gluten disease?
a) Type 2 Diabetes Mellitus
b) Epilepsy
c) Sjögren's syndrome
d) Autoimmune Thyroid Diseases
e) Down Syndrome

57. Which of the following is not one of the causes of acute pancreatitis?
a) Drugs such as Sulfonamides and Salicylates
b) Alcohol
c) Hypocalcemia
d) Triglyceride elevation
e) Scorpion stings

58. Which of the following cannot be said for hepatic encephalopathy?


a) It can be seen in acute or chronic liver disease
b) Patients who develop hepatic encephalopathy are candidates for liver transplantation
c) It is a poor prognosis indicator
d) There is usually a precipitating factor
e) Hepatic encephalopathy causes irreversible brain damage
59. Which of the following is not one of the goals of chronic hepatitis B treatment?
a) HBV DNA loss
b) Anti-Hbc IgG loss
c) Hbe Ag loss
d) Normalization of ALT
e) Reduction in liver necroinflammation

60. Which of the following tests does not need to be performed in a patient newly diagnosed with
chronic hepatitis B?
a) HBV DNA
b)AFP
c) Abdominal USG
d) Antinuclear antibody (ANA)
e) Anti-HCV

61. Which of the following is not a risk factor for esophageal squamous cancer?
a) Folic acid deficiency
b) Vitamin A deficiency
c) Use of acetyl salicylic acid
d) Tylosis
e) Achalasia

62. Which of the following can be used to correct the abnormal viscerosomatic reflex?
a) Peppermint oil
b) Microbiota modification
c) Pinaverium
d) Biofeedback
e) Otilonium

63. Which of the following is not true for gastroesophageal reflux disease?
a) Endoscopy is required for diagnosis
b) It may be related to dental erosions
c) It may be related to asthma
d)Obesity is a risk factor
e)Esophagus may be normal in endoscopy

64.Which of the following is not one of the relative risk factors for the development of malignancy
in pancreatic cystic neoplasia?
a) Pancreatic duct diameter is larger than 10 mm
b) Thickened cyst wall
c) Recurrent acute pancreatitis
d) Increased CA 19-9 level
e) Cyst size larger than 3 cm
65. A patient diagnosed with extensive ulcerative colitis presents to the emergency clinic with
complaints of fever and abdominal pain while receiving azathioprine and mesalazine treatment.
Physical examination reveals hypotension, tachycardia, abdominal tenderness, distension, and
decreased bowel sounds. Rebound is not detected. What is your most likely diagnosis for this
patient? What would be your approach?
a) The probable diagnosis is Toxic megacolon. To confirm this diagnosis, the patient should have
a colonoscopy urgently.
B)Possible diagnosis is Toxic megacolon. To confirm this diagnosis, it would be good to have the
patient take an urgent CT scan or a direct standing abdominal radiograph to see that the
transverse colon diameter is more than 6cm.
c) The probable diagnosis is Toxic megacolon. There is no need for additional examination for
diagnosis. Physical examination is sufficient. It is necessary to undergo urgent surgery.
d) Possible diagnosis is Toxic megacolon. After the diagnosis is confirmed by USG, diarrhea
should be reduced with antidiarrheal drugs.
e) The probable diagnosis is colon perforation. Colonoscopy should be performed to confirm the
diagnosis.

66. Which of the following are true for gastrointestinal polyposis syndromes? 1. Peutz-Jeghers
syndrome is a variant of Familial Adenomatous polyposis (FAP). The primary syndrome of the
central nervous system is one of the hamartomatous polyposis syndromes. It is inherited in an
autosomal dominant manner. 2. It may be associated with Turcot tumors (medullablastoma,
glioblastoma multiforme). 3. In Attenuated Familial Adenomatous Polyposis (FAP), colon cancer
develops at an earlier age than in classical FAP. 4. In classical Familial Adenomatous polyposis
(FAP), adenomas begin to form at the age of 10-12, so annual sigmoidoscopy should begin at this
age.
a) 1,2,4 is correct
b) only 3 is correct
c) all are true
d) only 1 and 2 are correct.
e) only 2 and 4 are correct

67. Which of the general principles of approach to nonvariceal upper gastrointestinal system
bleeding is incorrectly stated?
a) It is used to determine the clinical prognosis of the patient with appropriate scoring in order to
decide on the appropriate approach to the patient. and Rockall score are predictable. The most
commonly used scorings are: Glasgow-Blatchford Score (GBS) is the pre-endoscopy part.
b) Second-look endoscopy is important to prevent compulsory rebleeding of the principles of
approach to nonvariceal upper GI bleeding.
c) Intravenous PPI administration is recommended for patients with predicted NVUGIB. It is
recommended to ensure hemostasis. The target may be different in patients with comorbidities.
d) In patients requiring erythrocyte transfusion, targeting HB 7-9g/dl after transfusion has been
shown to reduce recurrence.
e) H. Pylori status should be investigated and if positive, it should be eradicated. Peptic ulcer
bleeding of erection
68. Type II Gastric neuroendocrine tumor may be associated with which of the following
syndromes?
a) Multiple endocrine neoplasia type 1
b) Tuberous sclerosis
c) Von Hippel Lindau Syndrome
d) Neurofibromatosis Type I
e) Cushing's syndrome

69. Which of the following is the most common solid benign liver lesion?
a) Hepatic adenoma
b) Focal nodular hyperplasia
c) Caroli disease
d) Hemangioma
e) Nodular Regenerative Hyperplasia

70. Which of the following is the autoantibody used for the diagnosis of primary biliary
cholangitis?
a) Antinuclear antibody
b) Anti-mitochondrial antibody
c) Anti Smooth muscle antibody
d) Anti Liver kidney microsomal antibody
e) Anti soluble liver antigen antibody

71. A 30-year-old, 27-week pregnant patient complains of persistent itching that worsens at night.
He does not describe fatigue or abdominal pain. Laboratory findings ALT 52 AST 57 GGT 230
bilirubin total 1.7. Which of the following is not an expected finding for this patient?
a) Risk of recurrence in other pregnancies
b) Having a similar pregnancy history in the family
c) Immediate recovery in the postpartum period
d) Increased ammonia level in the blood
e) Serum bile acids > 10 μmol/L

72. Which of the following can be considered the best finding that distinguishes Crohn's disease
from Ulcerative colitis?
a) Perianal fistula
b) Continuous colonic involvement in colonoscopy
c) Pseudopolyps
d) Extraintestinal involvements
e) Diarrhea with bloody mucus

73. Which of the following is true about portal hypertension and cirrhosis?
a) Portal pressure (HVPG) is considered normal up to 10 mm Hg
b) Varicose vein development is a sign of decompensation in cirrhosis.
d) Increased nitric oxide in the liver is also important in the development of portal hypertension.
c) Defenestration of sinusoids is important in the pathophysiology of portal hypertension
e) Somatostatin analogs are used for the prophylaxis of variceal bleeding.
74. Which of the following statements is NOT true about biochemical, function tests and
metabolic diseases of the liver?
a) Prothrombin time (PT), INR and serum albumin indicate liver functions.
b) The first step in evaluating a slightly elevated serum aminotransferase level is to perform repeat
tests to confirm the persistence of the elevated value.
c) Macrovesicular fat accumulation and Mallory equivalent bodies in liver biopsy define the
characteristics of Hemachromatosis.
d) Wilson disease is an autosomal recessive disease with the defective gene ATP7B on
chromosome 13.
e) D-penicillamine, trientine and zinc are used in the treatment of Wilson's disease.

75. Which of the following information about peptic ulcer, functional dyspepsia, and alarm markers
is NOT true?
a) The main risk factors of peptic ulcer are Helicobacter pylori infection and nonsteroidal
anti-inflammatory drug (NSAID) use.
b) The most common cause of dyspepsia in the gastroenterology department is functional
dyspepsia.
c) A tear of 5 mm or larger in the mucosa and submucosa layer in the gastrointestinal system is
defined as an ulcer, and a tear of less than 5 mm is defined as erosion.
d) Upper gastrointestinal endoscopy should be performed on a 71-year-old male patient with
epigastric pain and weight loss for 4 months.
e) Diagnostic criteria of alarm features; It includes postprandial bloating, early satiety, epigastric
pain and epigastric burning.

76.A 67-year-old female patient complains of jaundice in the sclera and itching on the skin, which
started 4 weeks ago. On physical examination, jaundice appears on the skin. Laboratory values are
as follows: ALT 108 U/L AST 92 U/L Alkaline phosphatase 479 U/L (25-100) GGT 182 U/L (10-50)
Total bilirubin 4.9 mg/dL Direct bilirubin 2.1 mg/dL Albumin and INR-PTT is normal. What is the
next most appropriate step in diagnosis?
a) Liver biopsy
b) Endoscopic retrograde cholangiopancreatography (ERCP)
c) Abdominal Ultrasonography (USG).
d) Computed Tomography (CT)
e) Endoscopic ultrasonography (EUS)

77.A 1x1.2 cm, stage 3 pressure injury was detected in the sacral area in a 76-year-old female
patient. Which of the following is not appropriate to approach this patient?
a) If the patient is bedridden, an air mattress should be provided and a full position change should
be made at least every 2 hours.
b) If possible, lifting plates should be used to eliminate the effect of friction force.
c) Necrotic tissue should be allowed to heal on its own and should not be debrided.
d) The head of the bed should be kept at 30 degrees and wedge-shaped head pillows should be
used.
e) If risks are detected in other areas, barrier cream should be used.
78. A 67-year-old female patient has no known disease other than hypertension. He uses
lercanidipine 10 mg for hypertension. In the tests performed within the scope of comprehensive
geriatric evaluation, Katz basic activities of daily living scale: 6/6 Lawton-brody instrumental
activities of daily living: 8/8 Mini-nutritional assessment: 12/14, Geriatric Depression scoring 3/15
and Mini-Mental test score: 30. /30 is detected. Which of the following approaches is correct
regarding this patient? I. Pneumococcal and influenza vaccination is recommended. II. Even if
bone mineral density is normal, annual DEXA monitoring is recommended. III. Routine screening
tests are not required because its functionality and life expectancy are not good.
It is not appropriate to do so. IV. Colorectal cancer screenings are recommended if they have never
been done before.
a) I-II-III-IV
b) I-III
c) I-II-IV
d) I-IV
e) I-III-IV

79. Which of the following is not an intrinsic cause that may cause falls in the elderly?
a) Cognitive disorders
b) Vitamin D deficiency
c) Having a history of falling in the past
d) Heart rate and rhythm disorders e) Unsuitable grounds

80. A 73-year-old female patient applies to the geriatric clinic with complaints of high blood
pressure and sugar. It is determined that he has a history of Diabetes Mellitus for 15 years, and in
the complication screening, he is being checked for neuropathy, nephropathy and retinopathy. In
the tests performed within the scope of comprehensive geriatric evaluation, Katz basic activities
of daily living scale; 6/6 Lawton brody instrumental daily living activities; 8/8 Mini nutritional
assessment; 12/14, geriatric depression scoring 7/15 and Mini-Mental test score 30/30. System
interrogations reveal that the patient complains of severe diabetic neuropathy pain and has a
complaint of urinary incontinence. He says that the incontinence complaint is minor with coughing
and sneezing. Which option gives the correct options for the case given above?
1. A full history and physical examination should be performed on the patient, first the causes of
temporary incontinence should be excluded.
2. The patient's incontinence type is urge incontinence.
3. The patient was found to be in a depressed mood in the depression screening performed with
the GDS score.
4. The first treatment to be given to the patient will be medication administration.
5. In treatment, behavioral methods should be tried first. Kegel exercise and regulating fluid intake
are among them.
6. The most appropriate drug treatment option for the patient is duloxetine.
7. The most appropriate drug treatment option for the patient is darifenacin.
a) II, III, IV, VI
b) I, II, III, V
c) I, III, V, VII
d) I, III, V, VI
e) I, II, III, V, VII
81. Which of the following is one of the most commonly used inappropriate medications in the
elderly?
a) Antihypertensives
b) Antidiabetics
c) Statins
d) Sedatives
e) Iron replacement

82. An 82-year-old male patient who applied to the geriatric outpatient clinic with a complaint of
forgetfulness is diagnosed with Alzheimer's dementia. Which of the following does not play a role
in Alzheimer's dementia pathology?
a) Cortical atrophy in the brain
b) Amyloid Beta plaques
c) Inclusion bodies
d) Neurofibrillary tangles
e) Having the apolipoproein E-84 allele

83. Which of the information given about delirium is incorrect?


a) Acetylcholine deficiency may play a role in its pathophysiology.
b) Wavy course and lack of attention are significant for the diagnosis of delirium.
c) Pain and emotional stress can lead to delirium.
d) Hyperactive type delirium is most commonly observed.
e) Antipsychotics can be used in the symptomatic treatment of delirium.

84. A 73-year-old male patient without any disease applies to the geriatric clinic with high blood
pressure. Treatment is planned for the patient whose blood pressure varies between
150-165/65-75 mmHg at home. Which of the following statements about hypertension is
incorrect?
a) In the elderly, systolic blood pressure increases with age and diastolic blood pressure plateaus.
b) Isolated diastolic blood pressure elevation is rare in the elderly.
c) Alpha blockers can be used as single agents when there is no other indication.
d) Calcium channel blocker can be used as initial medication.
e) Lifestyle changes are effective in elderly patients.

85. Which of the following types of lymphoma shows an indolent course?


a) Burkitt Lymphoma
b) Follicular Lymphoma
c) Angioimmunoblastic T-cell lymphoma
d) Diffuse large B cell Lymphoma
e) Peripheral T-cell Lymphoma

86. Which of the following is true in allogeneic stem cell transplants?


a) HLA class I compatibility between the stem cell recipient and donor is sufficient.
b) Sinusoidal obstruction syndrome is one of the acute complications.
c) There must be blood group compatibility between the recipient and the donor.
d) Only bone marrow stem cells are used.
e) There is no need for antimicrobial prophylaxis in allogeneic stem cell transplantation.
87. A 28-year-old male patient visits the emergency department due to nosebleeds. In the blood
count, PLT: 5x103 cells/uL, Hemoglobin: 8gr/dL, WBC: 130x109/L. Chest radiography shows wide
mediastinum. In the peripheral blood smear, small immature cells with narrow cytoplasm and
dense chromatin are observed. In flow cytometry, CD3 and CD7 expression is detected in these
cells. What is your preliminary diagnosis?
a) T-ALL
b) B-ALL
c)APL
d)AML M4
e) AML M7

88. Which of the following is true for Polycythemia Vera?


a) Erythrocyte Sedimentation rate increased.
b) Erythropoietin level increased.
c) Cytoreductive therapy is used in all patients.
d) The most important cause of morbidity is thromboembolic complications.
e) BCR-ABL mutation is positive.

89. Which of the following blood components is used to prevent allergic reaction?
a) Red blood cell suspension with reduced leukocyte count
b) Irradiated erythrocyte suspension
c) Washed erythrocyte suspension
d) Fresh frozen plasma
e) Cryoprecipitate

90. Which of the following is not true for Immune Thrombocytopenia (ITP)?
a) ADAMTS 13 level is normal.
b) Prothrombin Time is normal.
c) D-Dimer level increased.
d) First-line treatment is glucocorticoids.
e) IVIG can be used in emergency situations.

91. What is the diagnostic immunophenotype in classical Hodgkin lymphoma?


a) CD15+, CD30+
b) CD10+,CD20+
c) CD5+,CD23+
d) CD5-,CD20+
e) CD5-,CD23-

92. Which is not a hereditary cause of thrombophilia?


a) protein c deficiency
b) FVleiden mutation
c) prothrombin gene mutation
d) venous insufficiency
e) protein s deficiency
93. Which one(s) are used in the treatment of tumor lysis syndrome? 1- hydration 2- allopurinol 3-
azacytidine 4- hydroxyurea 5- rasburikas
a) 1,2,3
b) 1,2,4
c) 3.4
d) 1,2,5
e) 4.5

94. Mr. Ali works in a factory. That day, the Red Crescent's mobile "Blood Donation Center" comes
to the factory to collect blood. The factory manager tells his employees that donating blood would
be a good move. Mr. Ali, as one of the employees of the factory, is pleased to donate blood
immediately. What type of donor is Mr. Ali?
a) Voluntary donor
b) Autologous donor
c) Specific donor
d) Referred donor
e) Commercial donor

95. A 70-year-old male patient applies to the hospital to get a medical report. On physical
examination, the patient's blood pressure was found to be 180/110, and no other pathological
findings were detected. The hemogram of the patient, whose biochemistry tests are normal,
shows: Hemoglobin is measured as 14 g/dl, platelets are measured as 220 x 10^9/1 and
leukocytes are measured as 250 x10^9/1. What is the Rai staging of the patient diagnosed with
chronic lymphocytic leukemia?
a) Stage 0
b) Stage I
c) Stage II
d) Stage III
e) Stage IV

96. Medicines that are effective in the treatment of multiple myeloma have been given. Choose the
wrong one.
a) Lenalidomide
b) Bortezomib
c) Nilotinib
d) Daratumumab
e) Cyclophosphamide

97. Choose the wrong one from those given regarding the storage conditions of blood
components before use.
a) Whole blood should be stored at +4 degrees
b) Erythrocyte suspension should be stored at +4 degrees
c) Platelet suspension should be stored at +20-24 degrees
d) Cryoprecipitate should be stored at -4 degrees
e) Fresh frozen plasma should be stored at temperatures lower than -18 degrees.
98. Which of the following is not true for iron deficiency anemia?
a) Hypochromic and microcytic anemia
b) Transferrin saturation increased
c) Reactive thrombocytosis may be seen
d) In cases where inflammation is not accompanied, measuring ferritin <15 ng/ml is diagnostic.
e) It is a hypoproliferative anemia

99. A 62-year-old female patient applies to the outpatient clinic due to fatigue and decreased
exercise capacity. The patient's complete blood count reveals Hb 9 g/dl, MCV 110 fl, leukocyte
5400/mm3, platelet 546,000/mm3, reticulocyte 1% and serum Epo 680 u/l. In the peripheral blood
smear of the patient, whose nutritional deficiency was not detected, the leukocyte formula was
normal and no blasts were detected. It is noteworthy that the patient's bone marrow biopsy was
hypercellular, there was 30% dysplasia in the erythroid series, the blast rate was <5%, and
micromegakaryocytes were found. In the karyotyping performed on the patient's bone marrow, 8
out of 20 metaphases showed deletion of the long arm of chromosome 5.
It is observed that he suffered. What is the most appropriate first-line treatment for this patient?
a) Luspatercept
b)Lenalidomide
c)Recombinant erythropoietin
d)Dulotinib

100. Which of the following is incorrect regarding the conventional cytogenetic examination used
in the diagnosis of acute leukemias?
a) It is the examination of number and structure anomalies of chromosomes
b) It allows numerical determination of DNA copy number and mRNA level
c) Live and dividing cells are required for analysis.
d) The presence of the same abnormality in at least 2 cells for chromosome structure anomalies
and chromosome increases, and the presence of the same abnormality in at least 3 cells for
chromosome losses is considered "clonal".
e) For a healthy cytogenetic result, the metaphase of at least 20 cells should be examined.

You might also like